You are on page 1of 178

Republic rif the Philippines

PROFESSIONAL REGiULATION COMMISSION


Manila
BOARD OF MEDICINE

PHYSICIAN Licensure Examination


Saturday, September 9, 2017 08:00 a.m. - 1O:0O a.m.

BIOCHEMISTRY SET A

INSTRUCTION: Select the correct answer for each of the following questions.
rcforeachitembymarkingtheboxcorrespondingtotheletterofyourchoice
pencilno.
on the answer sheet provided. STRICTLY Np ERASURES ALLOWED. Use 2 only.
'I

MULTIPLE CHOICE:
1. A S5-year-old woman together with her neighbors collected jungle mushrooms and had its
soup for dinner. After approximately 12 hours after ingestion, she began experiencing stomach
ache, burning throat, vomiting and diarrhea innumerable times. Her neighbor also staded
complalning of stomach upset during the day. The village physician diagnosed them cholera
and treated them with saline at home. Then 72 hrs. post-digestion of Amanita phalloides,
all developed jaundice and renal shutdovvn. They all died after a week. Which selective
inhibitor of q,-Amanitin makes it a deadly toxin?
A. DNA primase
B. RNA nuclease
C. DNA ligase
D. RNA polymerase
2. A2}-month-old boy was admitted to the Department of Pediatrics because of musculoskeletal
pain, irritability, fever and pallor. His past medical history was remarkable for psychomotor
developmental delay. He presented with a three-week history of increasing.bilaterdl knee pain,
and there was no history of trauma, but bleeding of the gums and pallor were reported. The
patient had a history of poor orat dietary intake he existed on a diet of milk products, and there
was a lack of fruits and vegetables. On physical examination, the patient weight was below
the third percentile for his age group. Vitamin C analysis was performed and serum ascorbic
acid concentration was found to be extremely low, under 3 micro-mole/l (reference range: 26.1
to 84.6 micro-mole/l). The diagnosis of scurvy was confirmed, and the child was treated with
500 mg of vitamin C daily. His mother was educated about dietary modification then two weeks
after vitamin C administration, the child's paih and the general condition of the patient
improved. Which among the statements about the synthesis of vitamin C is incorrect?
A. Hydroxy proline and hydroxy lysine ard important for stabilizing collagen cross linking
B. Vitamin C requires cofactor prolyl hydrbxylase and lysyl hydroxylase
C. Deficiency is primarily due to excessive crosslinking of tropocollagen
D. Defective collagen fibrilogenesis impairs wound healing
3. A77-year-old male was presented with spontaneous onset of edema, excruciating pain, and
inflammation at the metatarsal-phalangealjoint of the great toe. The acute flare of his gout
resulted from his overconsumption of high purine content foods and consumption of beer
plus hard liquors. He self-medicated with lbuprofen which only gave partial relief. ln the
succeeding days, the patient's situation continued to worsen and radiographic examination
revealed changes consistent with gouty arthritis. Allopurinol and uricosurics were prescribed
to lower his levels of uric acid. Question: Diseases associated with purine degradation with
overproduction of uric acid is often the result of the decrease salvage of:
A. inosine 5'-monophosphate dehydrogenase
B. hypoxanth ine-g uan ine phosphori bosyltransferase
C. S'-phosphorib.osyl pyrophosphate amidotransferase
D. adenine phosphoribosyltransferase
4. A 21-year-old man with sickle cell disease presented with URI symptoms for 2 daysand fever
to 38.8 "C. The URI symptoms consist of a stuffy nose, no rhinorrhea and a dry cough, which
has interrupted his sleep. Diagnostic tests were performed and management decisions were
made. H.is oral intake has been decreased, but adequate. He has been given paracetamol and

Continued on Page 2
PHYSICIAN Licensure Examination
Saturday, September 9,2017 - 08:00 a.m.i- 10:OO a.m.
BIOCHEMISTRY SET A

over the counter cold medications. He also takes daily prophylactic antibiotic. People with sickle
cell disease have abnormal hemoglobin S in their red blood cells and the mutation causing sickle
cell dse. is a single nucleotide substitution in the codon for amino acid at 6th position. This change:
A. converts a glutamic acid codon to a phenylalanine codon
B. converts a glutamic acid codon to an isoleucine codon
C. converts a glutamic acid codon to an arginine codon
D. converts a glutamic acid codon to a valine codon
5. A child born and raised in the U.S. planned to spend the summer on a relative's fruit farm and
to help with the harvest. The summer passed uneventfully, but several days after the harvest
began the child became jaundiced and very sick. Upon admission to the hospital the following
clinical findings were made: ln addition to the expected hyperbilirubinemia the patient
was hypoglycemic, had a markedly elevated rise in blood fructose concentration, and was
hyperlactiacidemic. Further history-taking revealed that during the harvest it was customary
for the family to indulge in fruit-filled meals and to snack freely on fruit while carrying out the
harvest. The elevated blood fructose was due to:
A. defective liver hexokinase
B. defective liver fructokinase
C. defective liver fructose-1-phosphate aldolase
D. defective liver fructose-1,6-bisphosphate aldolase
6. An infant delivered at 30 weeks and weighs 1,7OO grams appears to be normal at birth but
later showed cyanotic and breathing spells with grunting noise. The child cried inconsolably
for 5 hours and refused feeding. By next morning the child was working hard to breath. Chest
x-rays revealed dense lungs with significant atelectasis. Your diagnosis is respiratory distress
syndrome. Which of the following measurements would be most useful in making this
diagnosis?
A. phosphatidylglycerol C. phosphatidylinositol
B. phosphatidylcholine D. phosphatidylethanolamine
7. of bloating and diarrhea. His
A young black man entered his physician's office complaining
eyes were sunken and the physician noted additional signs of dehydration. The patient's
temperature was normal. He explained that the episode had occurred following a birthday party
at which he had participated in an ice cream eating contest. The patient reported prior episodes
of similar nature following ingestion of a significant amount of dairy products. This clinical
picture is most probably due to a deficiency in:
A. salivary o-amylase C. lactase
B. sucrase D. pancreatic o-amylase
8. A 20 year old female was brought to the emergency department with respiratory distress. She
gave a history of fever from the previous few days. She was a known alcoholics but used to
take alcohol only during episodes of depression. On examination she was pale, malnourished,
agitated and acutely ill. She was in acute respiratory distress. Her pulse was 1 1S/minute, there
was marked tachycardia and a systolic murmur was heard along the left sternal edge. Bilateral
crepitations were felt in the lungs. Neck vein were engorged and liver was enlarged, although
non-tender. Bilateral foot drop was also noted. lf the patient suffers from inadequate
micronutrient quantities, which of the following enzymatic activities would you expect to be
decreased in vitamin B1 deficiency?
A. pyruvate carboxylase
B. isocitrate dehydrogenase
C. o-ketoglutarate dehydrogenase
D. lactate dehydrogenase
9. The patient is a 2S-year old vegetarian who exercises regularly and does not smoke. His blood
pressure is 110fl0 and his body mass index (BMl) is 20. He has no medical complaints, but is
concerned about his family history of coronary heart disease; his father died of a heart attacl<
at age 50. He notes that his father had high cholesterol, which he attributes to his father being
overweight, eating a high-fat diet, and never exercising. The patient seeks reassurance that his

Continued on Page 3
PHYSICIAN Licensure Examination Page 3
Saturday, September 9,2017 - O8:OO a.m. -- 1O:OO a.m.
BIOCHEMISTRY SET A

healthy lifestyle will protect him frorn also having a heart attack at a young age- Which one of
the following dietary components most strongly influences the risk of coronary artery disease?
A. cholesterol
B. saturated fat
C. polyunsaturated fat
D. trans fatty acids
10. An entire family found themselves experiencing flu-like symptoms over several weeks. A week
prior to the emergency call, the daughter suffered convulsions and seizures, often symptoms
of CO poisoning. One morning, the father of the family woke extremely ill. He tried arousing
other members of the household without success. Completely disoriented and passing in and
out of consciousness he searched for a phone & luckily after several hours later he managed
to dial emergency medical services before again passing out. Emergency workers found him
collapse over the phone. The entire family received treatment at a local emergency room. The
culprit in this case was the furnace, which during a remodeling project, was enclosed in a
small room with no combustion air openings. With the above correlative case, which of the
following statements about carbon monoxide (CO) is incorrect?
A. lt is a chemical asphyxiant that disables the biochemistry of cellular respiration even in the
presence of adequate oxygen level in the blood.
B. lt is a simple asphyxiant that displaces oxygen in the ambient atmosphere causing oxygen
deprivation in those who are exposed leading to unconsciousness and death.
C. The binding of CO will shift the oxygen-hemoglobin curve to the left.
D. Carbon monoxide binds lightly to the hemoglobin iron forming carboxyhemoglobin.
i

11. An 8 month child presents with a history of poor growth and chronic cough. He was the product
of a 21 yr old G2P1 mother and was born art 41 weeks of gestational age. Soon after birth he
developed respiratory distress and was admitted to the neonatal intensive care unit where he
was mechanically ventilated for 1 day and discharged after 5 days. He was initially breast-fed,
but due to frequent vomiting and LBM, he was changed to formula feeding. Despite trials of
different types of formulas, his clinical cours;e was remarkable for bloating, diarrhea and failure
to thrive. He developed a daily cough and some respiratory difficulty. At the age of 5 months
he was hospitalized for respiratory distress and was diagnosed as having asthma. He
continued to have loose, large, greasy, foul - smelling stools and failure to thrive. An
iontophoresis of pilocarpine sweat test revealed positive result. lf the diagnosis of this child's
condition is cystic fibrosis, this disease is caused by which genetic mechanism?
A. hexosaminidase A
B. p-glucuronidase
C. glucocerebrosidase
D. transmembrane conductance regulator
12. A 4}-year-old white woman, similarly in good health, with an indoor occupation, moderate
recreational UV exposure, and no family of skin cancer, had a scaly papule on the dorsum of
her right hand. Biopsy (hematoxylin-eosin) revealed a squamous cellcarcinoma in situ
and stage 3 stages of Mohs surgery were required to clear the tumor. The area healed by
secondary intention. The patient had a 1S-year history of twice-monthly UV nail light exposure
to dry her nail polish and set her acrylic nails. Question: Which of the fOllowing types of
physical mutagens produces thymine dimer mutation?
A. ionizing radiation
B. x-ray
C. microwave
D. ultraviolet light
13. While waiting for his baby sister to be born, a 4-year-old child fell off a stairs in the hospital
waiting area striking the edge of a glass-topped table on the way to the floor. The impact result
to the child's loss of consciousness with 2 open cuts near his eyebrow. He was brought to the
ER by the nurses and an attending doctor from adjacent ward for further evaluation and
management. ln the succeeding days, the child started having convulsive seizures. He was

Continued on Page 4
PHYSICIAN Licensure Examination Page 4
Saturday, September 9,2017 - 08:00 a'm' - 1O:OO a.m.
BIOCHEMISTRY SET A

started on anti-epilepsy medication Phenobarbital and sent home advised. With a diagnosed
case of medical refractory epilepsy, what do you think is the best diet plan for this boy?
A. gluten free diet C. elementaldiet
B. ketogenic diet D. diabetic diet
14. A 3g-month-old child exhibiting disproportionate short-trunkdwarfism, jaws that extend out
farther than normal and cloudy cornea undenruent histological and electron-microscopy
examination of the long bone growth plate. These result indicated marked irregularities
in chondrocytes orientaltion within the growth plate, disruption of the_normal columnar
architecture, and vacuolization and enlargement of the cellular border. The child's physical
stature and the ultrastructural analysis of bone development indicate the child is suffering
from a mild form of mucopolysaccharidosis type I called
A. Hurler's syndrome
B. Scheie syndrome
C. Sanfilippo syndrome
D. Hunter's syndrome
15.A44-year-oldwomanpresentedwitha4-weekhistory of progressive visual loss and was
noted to have bilateral retrobulbar optic neuropathy. No other clinical abnormality was noted.
lnvestigations revealed severe folate defic:iency. Her alcohol and tobacco consumption was
moderite and subsequent correction of folate levels with oral supplementation has led to
improvement in her visual acuity. What other vitamin co-exist with folic acid deficiency leading
to FOLATE TRAP?
A. vitamin 81 C. vitamin 86
B. vitamin 83 D. vitamin 812

'16. A nine-year-otd boy was presented with multiple yellowish plaques consistent with tuberous
xanthoma in different parts of body, including auricles, back of the elbows, gluteal regions
and leg. The lesions were present at birth, and increased number over time. A systemic
examination of the cardiovascular system revealed otherwise normal findings. There was no
organomegaly revealed. There was no history of premature coronary artery disease or death
in the family. His maternal grandfather underwent coronary artery bypass (CABG) at the age
of 60. His father, grandfathers (both paternal and maternal) had xanthelasmata. However, his
two siblings had no such lesions. Lipid profile performed initially and repeated on several
occasions. The clinical features and biochemical parameters in this nine year old boy are
consistent with Familial Hypercholesterolemia without cardiovascular sequel. Treatment with
with cholestyramine and atorvastatin proved no benefit. Niacin or plasmapheresis may be
considered before liver transplantation. ln biochemical context through'mechanismsonly
partially understood, levels of Apolipoprotein B (Apo B) involved in atherosclerosis and
cardiovascular diseases is often seen abundant in:
A. chylomicron C. LDL
B. IDL D. HDL

17. This is a case of a77-year old female with a past medical history of hypertension and type 2
diabetes mellitus who had recently been prescribed rmetformin (3 g/day), perindopril and
acetylsalicylic acid. She was admitted to the emergency department two weeks later with
abdominal pain and psychomotor agitation. Physical examination revealed only signs of poor
perfusion. Laboratory evaluation revealed hyperkalemia, elevated creatinine and blood urea
nitrogen and mild leukocytosis. Arterial blood gases showed lactic acidemia. She was
admitted to the intensive care unit. Vasopressor and ventilatory support was initiated and
continuous venovenous hemodiafiltration was instituted. Twenty-four hours later, full clinical
recovery was observed, with return to a normal serum lactate level. The patient was
discharged from the intensive care unit on the sixth day. What acid-base imbalance is
is indicated in the above case?
A. respiratory acidosis C. metabolic acidosis
B. respiratory alkalosis D. metabolic alkalosis

Continued on Page 5
PHYSICIAN Licensure Examination Page 5
Saturday, September 9,2017 - O8:OO a.m. - 1O:OO a.m.
BIOCHEMISTRY SET A

of mature coconuts harvested from


18. Coconut oil is an edible extract from the kernel or meat
the coconut palm. lt has several benefits such as skin care, hair care, weight loss, improving
digestion and immunity against a host of infections and diseases. The majority of coconut oil
ftjilJ; }6X* l5,r"l;*o
ratty acid c. tons chain saturated and unsaturated ratty acid
B. medium chain saturated iatty acid D. very long chain unsaturated fatty acid
19. A 3O-year old male jewelry worker accidentally ingested silver potassium cyanide and was
Urougnt to the emeigency department in alstate of shock and profound metabolic acidosis.
The patient was managed hypothetically with use of injection thiopentone sodium
intravenously untilthe antidote was received. Cyanide is a highly cytotoxic poison and it
rapidly reacis with the trivalent iron of cytochrome oxidase thus paralyzing the aerobic
respiration. The result is severe lactic acidosis, profound shock, and its fatal outcome. The
patient dies of cardio-respiratory arrest secondary to dysfunction of the medullary centers. lt
is rapidly absorbed, symptoms begin few seconds after exposure and death usually occurs in
<30 min. The average tethal dose for potassiurn cyanide is about 250 mg. Question: Cyanide
acts on which part of the electron transpoft chain?
A. complex ll C. complex lV
B. complex lll D. complex V
20. A SO-year-old woman presented to the OPD with right upper abdominal pain and backache
that has lasted 10 days. She had no palpitation, sweating, orhypertension (BP = 121177 mm
Hg). The patient's history revealed that she had a similar incidence of abdominal pain two
months prior, which was a "dull ache" in nature and somewhat associated with headache. The
pain was relieved using a mild analgesic drug. Abdominal ultrasound showed a right adrenal
mass, and both CT and MRI imaging of the adrenal glands confirmed a right adrenal mass
consistent with adrenal pheochromocytoma. The patient undergone surgery and showed full
recovery at follow-up after three months. Patient with this condition will have an increased
level of which of the following metabolites in the urine?
A. epinephrine C. vanillylmandelicacid
B. dopamine D. metanephrine
21. This is a case of a 4 day old, 3.2 Kg, appropriate for gestational age female infant born at term
lo a 24 year old blood type A+ primiparous woman with gestational diabetes. The pregnancy
was otherwise uneventful. Labor was augmented with Oxytocin. The baby was discharged
home on day of life 2 at which time her weight was down 4Yo from birth weight and she had
mild facialjaundice. ln the hospital, she was breast fed every 3 hours and had 2wet diapers
andonemeconiumstool over24hr. period.Onday3,her parpnts gave her water on two
occasions as she appeared hungry despite regular and frequent breast feeding attempts. ln
addition, they noted an increase in the degree of jaundice, but failed to address it after being
reassured by them family members that jaundice is common. They also had an appointment
to see their pediatrician the following day. ln the office, on day 4, mother report that she
is breastfeeding the baby eveq/ three hours and that there have been 2 wet diapers
per day. The urine is described as dark yellow in color and the stools appear dark green.
Question: Bilirubin comes from the degradation of:
A. cytochrome C. heme
B. haptoglobin D. albumin

22. A 2o-year-old man presented for consideration of repair of a rectal prolapse that had been
present since birth. Physical examination revealed hyperelasticity of the skin with easy
bruisability, hypermobility of the joints, and a rectal prolapse of about 15 cm in length, which
was easily reducible. Results of laboratory test and chromosome studies were normal. Results
of a skin biopsy showed disorganization of collagen bundles in the dermis. None of the
patient's family members had the disease or similar findings on physical examination. No
evaluation for vascular abnormalities was performed. Surgery was deferred, given the mild
nature of the rectal prolapse and concern that the patient was at increased risk for surgical
complications. He received conservative treatment and has done well for more than 6 years.
Question: What connective tissue disease was presented by the affected patient?

Continued on Page 6
PHYSICIAN Licensure Examination Page
Hage 6
o
Saturday, September 9,2017 - 08:00 d.Irl. * 10:00 a.m.
BIOCHEMTSTRY SET A

A. Marfan syndrome
B. Peyronie's disease
C. Ehlers-Danlos syndrome
D. Osteogenesis imperfecta

23. Rhabdomyolysis is a syndrome characterized by muscle necrosis and the release of


intracellular muscle contents into systemic circulation. lt is most often associated with severe
or intense exercise. The manifestations of this syndrome range from asymptomatic elevation
of serum muscle enzymes to life - threatening cases associated with extremely high
enzyme levels, electrolyte imbalances and acute renal failure. The clinical presentation of
rhabdomyolysis includei myalgia, muscle weakness and brown or "tea-colored urine. ln
addition to a recent history of excessive exercise and clinical signs and symptoms, laboratory
findings such as elevated serum creatinine kinase and myoglobinuria confirrns the diagnosis
of rhaldomyolysis. The standard of care for rhabdomyolysis includes urine alkalization,
aggressive intravenous fluids and in some cases, short-term dialysis- ln addition to the known
tinli Uetween excessive exercise and rhabdomyolysis, rehabilitation specialists have recently
been interested in the risk of developing rhabdomyolysis from statin medications (HMG-CoA).
Statins are the most effective and widely prescribed drugs currently available for the reduction
of low-density lipoprotein (LDL) cholesterol, which is critical for primary and secondary
prevention of cardiovascular disease. Rhabdomyolysis is the most severe form of myotoxicity,
which can occur in all statins, either monotherapy or in combination therapy. Following statin
intake, which of the following is expected to be excreted in great amount in feces?
A. cholesterol
B. triacylglycerol
C. free fatty acids
D. bile salts
24. Themother of a four month old female baby attended in the well-baby clinic with the complaint
of black staining of the diaper after few minutes of urination. The baby was born of a non-
consanguineous marriage, healthy and breastfed. Mother noticed that stain first at the age of
two and half month. The urine kept in a test tube for two hours turned black. The patient was
diagnosed as alkaptonuria. Laboratory examination of urine revealed increased concentration
of:
A. tryptophan
B. tyrosine
C. homogentisic acid
D. melanin
25. A child develop chronic diarrhea and liver inflammation in early infancy when the mother
begins using formula that includes corn $yrup. Evaluation of the child demonstrates sensitivity
to iructose in the diet. Which of the folloWing glycosides contain fructose and therefore should
be avoided when feeding or treating infant?
A. sucrose C. lactose
B. rutinose D. maltose
26. A lighter skin tone has been considered a superior trait in most races, especially in wornan of
Asiin or African descent who have Fitzpatrick skin types lV-Vl. The higher prevalence of
pigmentary disorders in these skin types adds to the woes of the patients. ln relatively
conservative societies such as Philippines, many people are obsessed with the desire for a
fair complexion for themselves as well as their spouse. Such traditions motivate the patient to
desire fair complexion and sometimes seek it even against their will. Realizing this growing
need for fair skin, many pharmaceuticat companies focuses on oral and intravenous
glutathione market for skin-lightening and dermatologists are highly knowledgeable about
glutathione:its efficacy,the mechanism of hypopigmentary effects,pharmacokinetics, evidence
leveland safety profile. Question: What mineral help promote the production of glutathione
and considered important co-factor for the enzyme glutathione peroxidase?
A. zinc C- selenium
B. chromium D. molybdenum

Continued on Page 7
PHYSICIAN Licensure Examination Page 7t
Saturday, September 9,2017 - O8:OO a.m. - 1O:OO a.m.
BIOCHEMISTRY SET A

27. This is a case of a 21-year-old female, married for 5 years, educated, a homemaker from an
upper middle class family and living with her husband's family, was brought to a tertiary
care center with comptaints of gradual loss of weight, recurrent episode of vomiting, from a
period of 2 years, menstrual irregularities from 1 year and amenorrhea since 6 months, with
a probable precipitating factor being husband's critical comment about her weight. Patient
was reported to be dull and inactive most of the times since her marriage able to carry out her
activities of daily living adequately. With symptoms of weight loss and amenorrhea, she was
evaluated by a physician. A series of investigations were conducted in the background of
suspected tuberculosis, anemia for evaluation and abdominal tumors. However, all the
investigations were well within normal limits except low hemoglobin. lnformation was elicited
thru her husband which revealed an incident during their early days of marriage when he had
casually remarked of her being slightly heavy near her flanks and thighs and that she would
look more beautiful if she reduced it. Since then her intake of food decreased. She followed
a change in the diet pattern with complete avoidance of all foods with high caloric value. She
graduatly began to skip breakfast and would have minimal lunch. She began to avoid eating
in front of other family members. At times hide and eat, and/or would secretly go into the
bathroom and induce vomiting. The clinical featur'es and laboratory parameters in this 25-year
old housewife are consistent with anorexia nervosa. A multidisciplinary approach was
employed. Psycho education with regard to the disorder was given. Nutritional rehabilitation
was planned, where she was asked to maintain a diary about her intake of food. She was
encourage to eat food with high caloric diet. Question: What metabolic fuel provides the
energy during prolonged starvation?
A. glucose
B. ketone bodies
C. amino acids
D. lactate
28. A SO-year-old man presents to his general practitioner with complaints of fatigue for 2 months.
The patient also notes distension of his abdorrien and shortness of breath beginning 2 weeks
ago. His wife reports that the patient has been having episodes of confusion lately. The patient
his a significant medical history of chronic heavy alcohol consumption of about half a pint of
vodka daily for around 2O years. On physical exam the patient is noted to have scleral icterus,
tremors of both hands, and spider angiomata on the chest. There is abdominal distension
with presence of shifting dullness . fluid waves, and splenomegaly. Laboratory examinatioh
shows low hemoglobin, and high PT and lNR. Ultrasound of the abdomen shows liver
hyperechogenicity, portal hypertension, splenomegaly, and ascites. QUESTION: Since
alcohol is known to modulate the levels of serotonin in the central nervous system where it
functions as a neurotransmitter, which of the following statements about serotonin is correct?
A. associated with anxiety and depression
B. degraded via methylation by monoamine oxidase w/c also degrades the catecholamines
C. released by activated platelets
D. synthesized from tyrosine in a two-step process that utilizes a tetrahydrobiopterin-requiring
hydroxylase and a pyridoxal phosphate-requiring carboxylase

29. ApH of 7.O, inthe middle of the scate, is neutral. Blood is normally slightly basic. Acid-base
imbalance occurs when a significant insult causes the blood pH to shift out of the normal
range. A doctor evaluates a person's acid-base balance by measuring the pH and levels of
carbon dioxide (an acid) and bicarbonate (a base) in the blood. An excess of acid in the blood
is called acidemia and an excess of base is called alkalemia. The proqess that causes the
imbalance is classified based on the etiology of the disturbance (respiratory or metabolic) and
the direction of change in pH (acidosis or alkalosis). There are 4 basic processes: metabolic
acidosis, respiratory acidosis, metabolic alkalosis and respiratory alkalosis. One or a
combination may occur at a given time. Question: What is the normal pH of blood?
A. 7.2
8. 7.4
c. 7.6
D. 7.8

Contiuued on Page 8
PHYSICIAN Licensure Examination Page
Saturday, September 9,2017 - 08:0O a.m. - 10:O0 a.m.
BIOCHEMISTRY SET A

30. A 48 - year - old moderately obese man was brought to the medical emergency in an
unconscious state. He was breathing deeply and rapidly. The breath did not smell of acetone
or alcohol. Signs of mild dehydration, such as dry tongue, soft eyeballs, weak and rapid pulse
and low blood pressure were present. Examination of the past medical record showed that
the patient was diagnosed as having type 2 diabetes mellitus six years back. lnitial treatment
with Sulfonylureas proved effective in controlling blood glucose levels, and the patient
apparently remained in good health all these years. However, about four months back,
elevated blood glucose levelwas detected on 2 separate,occasion. lncreasing Sulfonylureas
did not help. Biguanide therapy was initiated, after the renal and hepatic functions were found
normal. Adequate control of diabetes was achieved with Metformin in three daily doses of 1.0
gram each. For the last 2 days, the patient had hectic sessions of physical activity when he
went to a hill station for recreation. He had rich meals during this period, often accompanied
by drinking till late night. While driving back home early in the morning, the family members
noticed that he was disoriented and his speech was incoherent. Soon afterwards he
lost consciousness and was rushed to the nearest hospital. Lactic acidosis frequently
occurs during strenuous exercise in healthy people, bearing no consequence. However,
development of lactic acidosis in disease states is ominous, often indicating a critical illness
of recent onset. Therefore, a careful history should be obtain to evaluate the underlying
pathophysiologic cause of shock that contributed to lactic acidosis. Furthermore, a detailed
history of ingestion of various prescription drugs or toxins from the patient or a collateral
history from the patient's family should be obtained. Question: What cause the elevated lactic
acid level in blood?
A. inefflcient utilization of lactate for glucose synthesis
B. abnormal conversion of lactate to pyruvate
C. accelerated glycogenolysis
D. lack of glucago
31. A 4$-year-old male was presented to the department of physical medicine and rehabilitation
with a long standing history of pain, swelling and deformity of small and large joints of both
hands and feet approximately 12 years. The patient reported with the history of polyarthralgia
which had improved over a week's time. This was followed by frequent intermittent episodes
of arthritis of small and large joints hands, knees, wrists, ankles and feet without morning
stiffness. The disease had been treated symptomatically with non-specific non-steroidal anti-
inflammatory drugs (NSAlDs) leading to the improvement over a period of time. Thereafter,
patient developed nodules on hand and feet. He was put on Allopurinol for 6 months and
dietary restriction of protein on account of hyperuricemia with serum uric acid levels of 12.43
mg/dL. Patient intentionally interrupted the treatment following his betterment of health.
Question: ln addition to arthritis, what other condition is most often associated with primary
gout?
A. hypoglycemia
B. urolithiasis
C. dementia
D. biliary atresia
32. Cassava is eaten widely in Africa, South America and some parts of Southeast Asia. lt must
be very thoroughly cooked before being eaten. Chew it raw, and enzyme in the tuber converts
into cyanide. Question: Cyanide blocks electron transport through complex IV. Which of the
following will occur if cyanide is added to cells?
A. The rate of citric acid cycle will increase
B. ATP synthesis will increase
C. NADH and FADH2 will be depleted
D. 02 consumption willdecrease
33. Many patients will read that some drugs increases the level of uric acid in the blood. Which of
the following drug should gout sufferers avoid?
A. Ketoprofen
B. Aspirin
C. lndomethacin
D. Colchicine
Continued on Page 9
PHYSICIAN Licensure Examination Page
Saturday, September 9,2017 - O8:OO a.m. - 10:OO a.m.
BIOCHEMISTRY SET A

34. On the day of admission, 2 sewerage workers (worker 1 and worker 2) entered a catch basin
of sludge in an underground enclosed sewer tank with a third co-worker. About 5 minutes
after entry, the men became dizzy and decided to leave the tank. While climbing out of the
tank, worker 1 fainted and fell 5 feet onto worke r 2, and both remained unconscious until they
were rescued. The remaining coworker was able to climb out of the tank and call for help.
Experts measured the air quality in the tank and found it contained 24O ppm of H2S, 0 ppm
of methane, and 18.57o oxygen. After 1S-minute extrication, the 2workers were transferred
to the local emergency hospital. The toxicity of hydrogen sulfide is thought to be due
primarily to reversible inactivation of the respiratory enzyme, cytochrome oxidase, with
resultant inhibition of aerobic metabolism. Patients with severe hydrogen sulfide (H2S)
poisoning and with profound metabolic acidosis are treated successfully with nitrites and
orygen. The nitrite induced methemoglobin, by competitively binding the toxic hydrosulfide
anion untildetoxified, presumably reactivated and protected cytochrome oxidase and thereby
aided the patient's recovery by enhancing aerobic metabolism. Rapid recovery adds clinical
support to the efficacy of nitrite therapy in sulfide poisoning. Therefore, practitioners
recommend that severe cases of sulfide poisoning be treated with nitrite - induce
methemoglobinemia in addition to vigorous supportive care. Question: Hydrogen Sulfide
(H2S) acts on which part of the electron transport chain?
A. complex ll C. complex lV
B. complex lll D. complex V
35. A 4-year-old presented to the clinic with megaloblastic anemia and failure to thrive. History
revealed that the child was born normally. The RBC count was 2.55 million/mm3 and
hemoglobin was 6 g/dl. She was given antibiotics and transfusion. Despite that the anemia
worsened. There was no response following treatment with vitamin 812, folic acid or
pyridoxine. A prominent feature of the child's urine was a crystalline sediment, which was
iound to be oiotic acid. Orotic acid in amounts as high as '1500 mg (9.6 mmol) was eXcreted'
daily (Norm al 1 .4 mg/day, 9 umol). Enzyme rrleasurement of WBC revealed a deficiency of
pyrimidine biosynth6sis Lnzyme. The diagnosis of orotic bciduria was rnade. Orotic aciduria
type l, also known as hereditary orotic aciduria, is a condition that occurs when the body
cannot make compound called UMP synthetase, which is an important enzyme that produces
the chemicals that make up DNA. The main symptoms of this disease are megaloblastic
anemia, as well as delays in physical and intellectual development. Orotic aciduria is
caused by inherited changes (mutations) in the UMPS gene. Treatment consists of taking
supplements of a compound called uridine. Question: Which of the following enzyme is lacking
in orotic aciduria?
A. hypoxanthine-guanine phosphoribosyl transferase enzyme
B. carbamoyl phosphate synthetase ll enzyme
C. orotodylic pyrophosphorylase - orotodylic decarboxylase enzyme
D. phosphoribosl pyrophosphate synthetase enzyme
36. A 1.S-year-old boy presented with a three week-history of erythematous scaly plaques on his
extremities, diaper-region, and face. Thr.ee months later similar lesions appeared over much
of his body, but resolved spontaneously; the patient developed cracked lips, mucosal ulcers,
thinning of the hair, and 2 plus non-pitting edema on both extremities, two weeks after the
skin lesions. He was a confirmed case of phenylketonuria since the 11th month of his life. His
medications include Clonazepam and Phenobarbital for seizure disorder. He had also
been prescribed folic acid and feroglobin. The child was a full-term infant, born without
complications. While his past medical history was significant for neurodevelopmental delay
and visual inattention, his older brother was and is healthy. On examination, the patient was
irritable and diffusely edematous and had a "chubby" appearance. His skin manifestations
include multiple annular plaques with hyperkeratotic border and desquamated centers and
were confluent over the extremities, buttock, and face, in addition he has angular cheilitis.
Neurological exam revealed poor head control, microcephaly, and poor eye contact.
Laboratory studies revealed a hemoglobin of 6.1 and hematocrit 18.3, white blood cell count
18200(poly: 59%) and platelet count 285000 .total protein and serum albumin were 4.9 mg/dL
and 2.1mg/dl, respectively. BS, BUN, Cr, Ca, Ph, and transaminases were in normal ranges.
Phenylalanine levelwas 0.4 mg/dl. The child was hospitalized and put on a special diet,

Continued on Page 10
PHYSICIAN Licensure Examination Page'10
Saturday, September 9,2017 - 08:00 a.m. -- 10:0O a.m.
BIOCHEMISTRY SET A

multivitamins, zinc, folate, albumin, and appropriate antibiotic. The skin lesions improved
dramatically using a combination of Mupirocin, Hydrocortisone, and Clotrimazole; oral
lesions responded to Nystatin and Metronidazole. Q,uestion: Which of the following statements
regarding PKU is not correct?
A. A Guthrie test can check the necessiry lab values.
B. The urine has a high concentration of phenylpyruvic acid.
C. Mental deficit are often present with PKU.
D. The effects of PKU are reversible.
37. A 56-year-old female presents with difficulty opening her eyelids, as wellas inability to raise
herself from sitting position. She is diagnosed with "myasthenia gravis", a disease of extreme
fatigue, due to decrease concentration of Acetylcholine in her muscles. She has been
prescribed Physostigmine, a drug that increases the amount of available Acetylcholine, by
completelyinhibiting acetylcholinesterase. Which of thefollowing statements is not true of
competitive in hibitor?
A. Vmax remain the same
B. Apparent Km is increased
C. lnhibitor binds covalently to the enzyme
D. lncreasing concentration of substrate can reverse the changes
38. A 1S-year-old girl is presented with abdominal pain and diarrhea for 3 days. She became
jaundiced and presumptive diagnosis of infective hepatitis was made, but serological tests
were negative. She subsequently died of fulminant liver failure. At post-mortem her liver
copper concentration was found to be grossly increased. A brown ring at the edge of the iris
was observed. What is the probable diagnosis?
A. Wilson disease C. Acrodermatitis enteropathica
C. Hemochromatosis D. Glucose-6-Phosphate dehydrogenase deficiency
39. Squalene is a natural 3O-carbon onganic compound originally obtained in high amounts from
shark liver oil. Recently more plant sources of squalene are available as squalene is also
abundant in olives and in lesser amount in amaranth seeds, rice bran, and wheat germ.
Yeast cells have been genetically engineered to produce commercially useful quantities of
"synthetic" squalene. Question: Which commitment cholesterol biosynthesis action of this
enzyme willshow decrease cholesterol synthesis and decrease plasma triglycerides level?
A. Squalene synthetase
B. HMG-CoA synthetase
C. HMG-CoA dehydrogenase
D. HMG-CoA reductase
40. A 2-week-old child s brought to the pediatrician office for his well-child visit. The child was
born overseas and did not receive the standard post-natal testing that is performed in the
Philippines. While in the office, the mother states that she has noticed a musty odor in the
child's diaper, and that the child has begun to develop a rash on his forearms. The pediatrician
is immediately concerned that the child may be suffering from an accumulation of which of
the following amino acids?
A. Leucine C. Citrulline
B. Phenylalanine D. Ornithine
41, Epinephrine, norepinephrine and dopamine are all derive from which amino acid?
A. serine C. glycine
B. tyrosine D. threonine
42. Clinical sign of "cabbage-like odor" is associated with:
A. ihcreased branched-ohain amino acid
B. increased homocysteine
C. increased methionine
D. increased phenylalanine

Continued on Page 11
PHYSICIAN Licensure Examination
Saturday, September 9,2017 - OB:O0 a.m. - 1O:OO a.m.
BIOCHEMISTRY SET A

43. The term pH is defined as:


A. The ion product of pure water C. The negative logarithm of [HA]
B. The logarithm of [H+1 and [OH-] D. The negative logarithm of [H+]
44. A 3o-year-old woman of Northern European ancestry presents with progressive dyspnea
(shortness of breath). She denies the use of cigarette. Family history reveals that her sister
also has problems with her lungs. Which of the following etiologies most likely explains this
patient's pulmonary sYmPtoms?
A. deficiency in dietary vitamin C c. decrease elastase activity
B. deficiency of o1-antitrypsin D. increased collagenase activity

45. There exists a fairly rare group of genetic disorders that have unfairly branded many sufferers
with the term "vampire". These poor souls are extremely,sensitive to sunlight that can easily
result in burns and abrasions, and so they prefer darkness. They suffer from acute attacks of
abdominal pains, vomiting and loose stools. Their urine may have a purplish-red color leading
some to wrongly believe that it results from drinking blood. Those afflicted may have increased
hair growth, and with repeated damage, their skin tightens and shrinks. When this occurs
around the mouth, the canine teeth appear to be more prominent, and suggestive of fangs. At
other times, it causes depression and affects the brain to produce peculiar behavior. lt is
probably no surprise that garlic makes all the symptoms worse. Porphyria is a misunderstood
condition. lt is very difficult to diagnose and symptoms mimic those of a hundred other
conditions. These symptoms make for a good vampire story and Count Dracula fits the
description. Fiction writers of the 18th and 19th centuries described Dracula as being deathly
pale in some accounts, but others described vampires as having flushed red faces due to their
intake of blood. They were repulsed by garlic and although immortal, could not reflect in a
mirror. Question: ln which of the following porphyrias, cutaneous hypersensitivity is not
observed in:
A. Variegate Porphyria
B. Congenital Erythropoietic Porphyria
C. Acute intermittent Porphyria
D. Hereditary Coproporphyria
46. The chief role of glycogen stored in the liver is to:
A. sustain adequate energy reserve for at least 3 days
B. abort hyperglycemia after a meal
C. keep blood glucose levels between meals
D. furnish energy to the skeletal muscles during physical activity
47. A7-year-old boys suffers from mental retardation and self-mutilation and has an increased
levels of serum uric acid. These symptoms are characteristic of Lesch Nyhan syndrome
which is due to defective:
A. Salvage pathway for pyrimidine biosynthesis
B. Denovo synthesis of pyrimidines
C. Xantine oxidase
D. HGPRT (Hypoxanthine Guanine Phospho Ribosyl Transferase)
48. A novel drug is developed that specifically acts only on the intestinal lumen. This new drug is
found to prevent the absorption of glucose and galactose from the lumen, but does not affect
the absorption of fructose. Based on the new drug's effects, which of the following is the most
likely mechanism of action?
A. it targets the GLUT-4 transporter
B. it targets the GLUT-2 transporter
C. it targets insulation regulation of glucose transporter
D. it targets the sodium-potassium pump within intestinal cells

49. Why can't the enzyme in saliva work in the stomach?


A. they get broken down by bacteria C. it is too dark
B. the stomach is moving too much D. conditions are too acidic

Continued on Page 12
n Page 12
3 :,I :J :J:S*',"""J ; : :", 33 # 1' "&, o o a m - 1 o :o o a. m
BIOCHEMISTRY SET A

50. The DNA triplet corresponding to the messenger RNA triplet CGU:
A. ACT C. GCT
B. TAC D. GCA
51. Which "system" is most likely to have the highest entropy?
A. alpha helical protein C. plasma membrane
B. ice cubes D. sugar molecules in hot tea
52. ldentify the abbreviation and symbol of the amino acid responsible for heme synthesis:
A. Gly=6 C. Trp = 1nt
B. Tyr=Y D. Arg-R
53. A 35-year old Mediterranean man receiving antibiotic therapy for UTI has self-limiting episode
of hemolysis, back pain and jaundice. Blood analysis showed Heinz bodies around his
peripheral RBC. His son also suffer the same fate for eating fava beans. Which of the
following enzymes is most likely deficient in this case?
A. Pyruvate kinase
B. Glucose-6-phosphate dehydrogenase
C. Dihydrofolate reductase
D. Transketolase
54. Which of the following metabolic processes occurs in the mitochondria?
A. cholesterol synthesis C. pentose phosphate pathway
B. glycolysis D. fafty acid B-oxidation
55. This type of enzyme inhibition is associated with a decrease in Vmax without a change in Km?
A. competitive C. uncompetitive
B. non-competitive D- irreversible
56. Pyruvate is LEAST likely to be used to synthesize which of the following?
A. acetyl CoA C. lactate
B. ethanol D. purine nucleotide
57. Aspartafne (Nutrasweet) is:
A. a modified sugar C. a carbohydrate
B. a dipeptide D. an amino acid
58. Which of tlre following binding interactions is likely to be the most important initial interaction
when a drug enters a'binding site?
A. hydrogen bond C. van der waals interaction
B. ionic bond D. induce dipole-dipole interactions
59. Which of the following is INCORRECTLY paired?
A. Vitarnin 81 - Wernicke Korsakoff Syndrome
B. Vitamin 83 - Casal Necklace
C. Vitamin C - Frankel's sign
D. Vitamin 86 - Cheilitis
60. Which of the following is NOT true about an o-helix?
A. the side chain extend radially outward from the helix axis
B. it is held together primarily by hydrogen bonds
C. it usually involves multiple polypeptide chains
D. the peptide backbone is on the inside of the helix
61. In which one of the following tissues is glucose transport into the cell insulin dependent?
A. adipose C. liver
B. brain D. red blood cells

Continued on Page 13
PHYSICIAN Licensure Examination
Saturday, September 9,2017 - O8:OO a.m. -' 1O:OO a.m.

BIOCHEMISTRY SET A

62. Many antimicrobials inhibit protein translation. Which of the following antimicrobials is correctly
paired with its mechanism of action?
A. Erythromycin binds to the 6O5 ribosomal subunit
B. Puromycin inactivates EF-2
C. Streptomycin binds to the 3OS ribosomal subunit
D. Tetracycline inhibit peptidyltransferas,e
63. Glutathione is a powerful antioxidant that inhibits the formation of free radicals. lt is a strong
reducing agent and works both outside cells and inside cells. Unlike a number of antioxidants
it can wend its way into mitochondria and protect it. lt repairs damage caused by stress,
pollution, the sun, radiation and cigarette smoke. Glutathione, which is a tripeptide, is present
in numerous foods. ln pill form it is not as easily absorbed as it is when consumed with foods.
Those with debilitating illnesses and poor diets might use it as a supplement. Question: What
vitamin helps to increase the absorption of Glutathione and act as protectant of Glutathione
in the blood and liver?
A. Vitamin A C. Vitamin D
B. Vitamin C D. Vitamin E
64. The key mechanistic failure in xeroderma pigmentosum patients involves which one of the
following?
A. loss of proofreading capacity
B. inability to excise a section of UV damaged DNA
C. mutation on one of the mismatch repair components
D. inability to synthesize DNA across the damaged region
65. All cjf the following are polysaccharides, e:<cept:
A. glycogen C. maltose
B. cellulose D. starch
66. Long term use of corticosteroid leads to the deficiency of all of the following, except:
A. Vitamin D C. Calcium
B. Vitamin K D. Zinc
67. All of the following neurological disorders are caused due to expansion of the repeating triplet
CAG, except:
A. Fragile X syndrome c. Huntington's disease
B. Mytonic dystrophy D. Kennedy's disease

68. Glycolysis is regulated by all of the following enzymes, except:


A. hexokinase C. aldolase
B. phosphofructokinase D. pyruvate kinase
69. Which one of the following statements concerning gluconeogenesis is correct?
A. it is an energy-producing (exergonic) process
B. it is important in maintaining blood glucose during a fast
C. it is inhibited by a fall in the insulin-to-glucagon ratio
D. it occurs in the cytosol of muscle cells
70. Which of the following is least likely to be impaired if a patient has liver disease?
A. fat digestion
B. bile production
C. hematocrit levels
D. albumin concentration
71. For excretion the drug isoniazid, used in the treatment of tuberculosis, is subjected to:
A. conjugation with glutathione
B. sulphation
C. methylation
D. acetylation
Continued on Page 14
PHYSICIAN Licensure Examination
Saturday, September 9,2017 - 08:00 a.m. - 10:OO a.m.
BIOCHEMISTRY SET A

72. \Nhal term is used to describe the process by which DNA is copied to produce two daughter
DNA molecules?
A. reproduction
B. replication
C. translation
D. transcription
73. Zellweger's syndrome, a rare inherited disorder characterized by accumulation of C26-C2A
polyenoicacidsinbraintissueiscaused<Juetotheabsenceof:
A. lysosomes
B. peroxisomes
C. microsomes
D. endosomes
74. Caramelization of sucrose yields:
A. glucose and galactose
B. galactose and fructose
C. glucose and fructose
D. maltose
75. Pulmonary hypertension is a serious complication of a number of lung and heart diseases that
is characterized by peripheral vascular structural remodeling and loss of vascular tone. Nitric
oxide can modulate vascular injury and interrupt elevation of pulmonary vascular resistance
selectively; however, it can also produce cytotoxic oxygen radicals and exert cytotoxic and
antiplatelet effects. The balance between the protective and adverse effects of nitric oxide is
determined by the relative amount of nitric oxide and reactive radicals. Nitric oxide has been
shown to be clinically effective in the treatment of congenital heart disease, mitrial valvular
disease combined with pulmonary hypertension and in orthotropic cardiac transplantation
patients. Additionally, new therapeutic modalities for the treatment of pulmonary hypertension,
phosphodiesterase inhibitors, natriuretic peptides and aqueous nitric oxide are also effective
for treatment of elevated pulmonary vascular resistance. Question: From which amino acid is
Nitrous Oxide generated?
A. arginine C. asparagine
B. aspartic acid D. lysine
76. Substrate-level phosphorylation is catalyzed by which of the following enzyme?
A. hexokinase
B. glycerol kinase
C. pyruvate kinase
D. galactokinase
77. Which of the following pairs is interconverted in the process of mutarotation?
A. D-glucose and L-glucose
B. D-glucose and D-fructose
C. o-D-glucose and B-D-glucose
D. D-glucose and D-galactose
78. Which of the following statements concerning liver in the fed state is correct'7
A. Fructose 2,6-biphosphate is elevated
B. lnsulin stimulates the uptake of glucose
C. The oxidation of acetyl coenzyme A is increased
D. The synthesis of glucokinase is repressed
79. The mitochondria of brown fat contain a unique protein thermogenin in their inner membranes
which:
A. uncouples oxidative phosphorylation
B. facilitate electron transport
C. shuttles NADH
D. shuttles FADH
Continued on Page 15
PHYSICIAN Licensure Examination
Saturday, September 9, 2017 - O8:OO a.m. - 1O:OO a.m.
BIOCHEMISTRY SET A

80. What term is used to describe the process by which a segment of DNA is copied to produce
a molecule of messenger RNA?
A. reproduction
B. replication
C. translation
D. transcription
81. Which of the following is a precursor for gluconeogenesis?
A. glycogen
B. glycerol
C. glucagon
D. galactose
82. An 8O-year-old man presented with impairment of higher intellectual function and alterations
in mood and behavior. His family reported progressive disorientation and memory loss
over the last 6 months. There is no family history of dementia. The patient was tentatively
diagnosed with Alzheimer disease. Which one of the following best describes Alzheimer
disease?
A. it results from accumulation of denatured proteins that have random conformations
B. it is associated with the accumulation of amyloid precursor protein
C. it is associated with the deposition of neurotoxic amyloid B peptide aggregates
D. it is caused by the infectious B-sheet form of a host-cell protein
83.A 1S-year-old Type I diabetic faints after injecting himself with insulin. He administered
glucagon and rapidly recovers consciousness. Glucagon induces activity of:
A. glycogen synthase
A. gtycogen phosphorylase
C. glucokinase
D. hexokinase
84. During starvation, the major source of blood glucose is:
A. hepatic glycogenolysis
B. gluconeogenesis
C. iruscle glycogenolysis '

D. dietary glucose from intestines


85. Glucagon stimulates release of fatty acids from adipocytes because it:
A. activate transporter of fatty acid
B. activates lipoprotein lipase
C. activates hormone-sensitive lipase
D. inhibits uptake of glucose
86. Which of the following is NOT an intermediate of the citric acid cycle?
A. acetoacetate C. oxalosuccinate
B. citrate D. succinyl-CoA
87. The only step in TCA cycle in which substrate level phosphorylation takes place is catalyzed
by:
A. isocitrate dehydrogenase
B. succinyl CoA synthase
C. fumerase
D. malate dehydrogenase
88. ln gram negative cell walls, the peptidoglycan layer is linked to the outer membrane by:
A. lipopolysaccharide
B. lipoprotein
C. phospholipid layer
D. teichoic acid

Continued on Page 16
PHYSICIAN Licensure Examination " Page 16
Saturday, September 9,2017 - O8:OO a.m. - 1O:OO a.m.
BIOCHEM]STRY SET A

89. A 4S-year-old woman is diagnosed with breast cancer. The oncologist orders a positron
emission topography scan of the head to rule out metastasis. The imaging modality covalently
links a radioactive isotopes most commonly to Glucose to appreciate highly active areas in
the body such as tumor. Which of the following traps the tracer in the cell?
A. lnsulin C. GLUT-4
B. GLUT-1 D. PFK-1
90. The major glycolytic product produced under normal circumstances by erythrocytes required
for unloading of oxygen to the peripheral tissues is:
A. 2,3 biphosphoglycerate
B. 1,3 biphosphoglycerate
C. lactate
D. pyruvate
I

91. Histones least tightly bound to chromatin are:


A. H1
B. H2A
C. H3
D. H4
92. A24-year-old woman is getting training for her first marathon. Her coach instructed her to
keep a pace that allows her to stay below her anaerobic threshold^ Under such conditions,
pyruvate does not accumulate as it is converted to:
A. ethanol
B. lactic acid
C. acetyl CoA
D. oxaloacetate
93. What term is used to describe the process by which proteins are synthesized from a genetic
code?
A. reproduction
B. replication
C. translation
D. transcription
94. Triple stranded DNA is a feature of:
A. A-DNA
B. B-DNA
C. H-DNA
D. Z-DNA
95. Deficiency of lysosomal 1-4 and 1-6 gluccisidase results in:

B. Pompe's disease
C. Here's disease
D. Tauri's disease
96. All of the following are ketone bodies, except:
A. acetone
B. acetoacetate
C. o-ketoglutarate
D. B-hydrory butyrate
97. Which of the following origin of the majority of the ATP used in the pathway gluconeogenesis?
A. p-oxidation of fatty acid
B. breakdown of amino acid
C. degradation of glycogen
D. oxidation of fructose-6-phosphate

Continued on Page 17
PHYSICIAN Licensure Examination Page 17
Saturday, September 9,2017 - O8:OO a.m. - 1O:OO a.m.
BrocHEMrsTRY SET A

98. Point of difference between substrate level phosphorylation and oxidative phosphorylation?
A. Oxidative phosphorylation is the synthesis of ATP in the Electron Transport Chain
B. Substrate level phosphorylation takes place exclusively in the cytosol
C. Oxidative phosphorylation produces a mole of ATP per mole of substrate oxidized
D. Substrate level phosphorylation is more energy efficient per mole of substrate converted
to product

99. Proofreading by DNA polymerase involves the removal of:


A. only the mismatched base on the old strand of DNA
B. only the mismatched base on the newly-synthesized strand of DNA
C. the mismatched base pair on both strands of DNA
D, several bases on the newly-synthesized strand of DNA
100. There are some absurd science questions that need to be answered like: "does physically
taking boogers out of your nose and putting them in your mouth and swallowing boost your
immune system?". The short answer is probably not. You ingest your snot all the time
needing to channel it through your mouth. So, if there is a benefit here, you get it without
needing to munch your nose nuggets. That said, there are a couple medical professional
willing to comment on the benefits of mining from green candy, particularly touting benefits
to one's immune systenr. One of the more credible sounding proponents of the habit is Scott
Napper, a professor of biochemistry who made waves around the world's media outlet in
2013 when he half-heartedly proposed to a group of his students that eating one's boogers
allows our bodies to safely develop anti-bodies to the weakened pathogens present in our
snot and noses. He also suggested that the reason boogers have a sugary taste is to entice
children to eat them, thus helping bolster their immune systems. lt's evolution. You can't fight
it. While he mostly just trying to get students interested in doing science by an unconventional
proposition, Napper's hypothesis, thanks largely to media, has since morphed into many
seeming to think that he actually did some'sort of study on this, and that there is evidence to
support it. The truth is that to date no such study has been done. Question: The sweetness
taste of boogers is primarily due to the prbsence of what?
A. monosaccharides
B. oligosaccharides
C. glucitol
D. rylulose

*** END ***

WARNING: Failure to submit your Test Questions (Complete) set will cause the cancellation
of your Test-Result for this subject.
BIOCHEMISTRY 2017
ANSWER KEY

1. D 26. C 51. D 76. C


2.C 27. B 52. A 77. C
3,8 28. C 53. B 78. A
4.D 29. B 54. D 79. A
5.C 30. A 55. B 80. D
6.8 31. B 56. D 81. B
7.C 32. D 57. B 82. C
B.C 33. B 58. B 83. B
9.8 34. D 59. D 84. B
10. B 35. C 60. c 85. C
11. D 36. D 61. A 86. A
12. D 37_C 62. C 87. B
13. B 38. A 63. B 88. B
14. B 39. A 64. B 89. B
15. D 40. B 65. C 90. A
16. C 41. B 66. B 91. A
17. C 42. A 67. D 92. C
18. B 43. D 68. C 93. C
19. C 44. B 69. B 94. C
20. c 45. B 70. c 95. B
21. C 46. C 71. D 96. C
22. C 47. D 72. B 97. A
23. B 48. D 73. B 98. A
24. C 49. D 74. C 99. D
25. A 50. D 75. A 100. B
Seat No.
Republic of the Philippines
PROFESSIONAL REGULATION COMMISSION
Manila
BOARD OF MEDICINE
PHYSICIAN Licensure Examination
Saturday, September 9, 2017
ANATOMY AND HISTOLOGY SET A

INSTRUCTION: Select the correct answer for each of the following questions.
Mark onlv one answer for each item by marking the box corresponding to the letter of your choice
on the answer sheet provided. STRICTLY NO ERASURES ALLOWED. Use pencil no. 2 only.
MULTIPLE CHOICE:
1. Apo Whang-Od (Fang-Od) is the oldest Kalinga mambabatok (traditional tattoo artist) in the
Philippines. She has been tattooing head hunters and women of indigenous people of Kalinga
for more than B0 years. The legendary tattoo method used in Kalinga culture, known as batok,
requires very little. The ink is made of soot and water and it is applied by tapping the ink
into the skin using thorns or needle - like glass, The rhythmic application is of course
more painful than the machine method we are familiar with. The most common tattoo is of
centipedes, representing protection and spirituat guidance. lnked individuats used their tattoos
to communicate with one another for generations. Tattoos represented who they were, where
they came from and what they stood for-marking their tribe's history with ink. Question: What
layer/s of the skin is tattoo ink injected into?
A. epidermis
B. dermis
C. deep subcutaneous tissue
D. all layers of the skin
2. Female genital mutilation (FGM), also known as female genital cutting (FGC), is the ritual
cutting or removal of external female genitalia. The practiced is found in dozens countries,
most commonly in Africa, but also in parts of the Middle East, Eastern Europe and South
America. lt is practiced by some Christians, Ethiopian Jews, some Muslim groups, and
followers of certain traditional African religions. That's why the United Nation views it as a
cultural practice, rather than a religious one. Other groups consider female genital mutilation
a brutal form of violence against women and girls. Some countries criminalizes the practice,
which is widely condemned as both harmful to the physical and mental health of women and
girls and a violation of their human rights. There are 4 types of genital mutilation (Type I - lV)
as defined by the World Health Organization. Question: What type of FGM defined as the
excision of the prepuce, with or without total or partial excision of the clitoris?
A. Type I FGM
B. Type ll FGM
C. Type lll FGM
D. Type lV FGM !

3. A71-year-old man with history of coronary artery disease and pacemaker treatment was
transferred from tertiary care hospitalwith a diagnosis of mitral valve endocarditis. He agrees
to mitral valve repair and removal of his pacemaker. Subsequently to the surgery, the man
suffers a left pleuraleffusion - a buildup of fluid around the lungs. He undergoes thoracentesis-
a procedure to have the fluid removed. During this procedure, he suffers a punctured artery,
a life-threatening hemothorax which requires an emergency thoracotomy, and emergency life-
saving measures are done at his bedside. The man files a medical negligence lawsuit against
the tertiary care hospital and the doctors who cared for him. He alleges that they violated the
standard of care by failing to properly perform thoracentesis, resulting in puncture or injury to
his artery, pneumothorax, and hemothorax, as well as other injuries. Question: What is the
highest level at which thoracentesis might safely be done without injuring the lung?
A. 5th ICS space C. 7th ICS space
B. 6th ICS space D. 8th ICS space

Continued on Page 2
Page 2

ANATOMY AND HISTOLOGY SET A

4. Venipuncture is the process of obtaining intravenous access for the purpose of intravenous
therapy or for blood sampling of venous blood procedure. This is performed by medical
practitioners, dialysis & medical laboratory technicians, some EMTs, paramedics,phlebotomist
and other nursing staff. Blood is most commonly obtained from the superficial veins of the
the upper limb (antecubital fossa). The most common vein for blood collection is the:
A. Basilic vein C. Median antecubital vein
B. Cephalic vein D- Lateral antecubital vein
5. A 66-year-old man was shown to have a prostate-specific antigen (PSA) level of 8 ng/ml
in a routine evaluation. His physical exam was normal and the digital rectal examination
-of prostate. Prostatic biopsy
revealed a slightly enlarged revealed a Gleason score 7 (4 + 3)
adenocarcinoma- in O fZ specimeris. His pasi medical history was unremarkable.
Laboratory data showed the following: hemoglobin level 15 g/ml, hematocrit 437o, white
blood cell count (WBC) 7,500 mm3, normal differential, platelets 250,000, blood urea nitrogen
15, creatinine 1.0, and normal alkaline phosphatase and liver function tests. Chest X-ray,
bone scan, and abdominal CT scan were negative. He underwent a radical retropubic
prostatectomy and the final pathology revealed a Gleason score 8 (4 + 4) adenocarcinoma
involving approximately 60% of the gland, established capsule penetration, and right seminal
vesicle involvement. ln addition, 1 of 5 lymph nodes on the right side demonstrate metastatic
adenocarcinoma. Three months after surgery his PSA is undetectable. Question: Which lobe
of the prostate is cancer most enmmonly found?
A. anterior lobe
B. posterior lobe
C. lateral lobe
D. median lobe
6. A 67-year-old woman presented with chronic symptoms of recurrent constipation and
abdominaldistention which she had been experiencing over 5 years. The constipation had
improved over the years through use of an enema. However, during the last six months, her
constipation had worsened and she had also suffered abdominal distention associated with
colicky pain. She had no hematochezia, had not undergone any previous surgery. Clinical
examination showed no abdominal mass on palpation but the abdomen was grossly
distended with visible peristalsis. Digital rectal examination revealed good anal sphincteric
tone with a high rectal fecal load. Routine laboratory evaluation, including a complete blood
count, urinalysis, and blood chemistry testing, were all within normal limits. The barium
enema showed a grossly dilated, large colon, with a transition zone located at the middle
one-third of the rectum. Computed tomography (CT) with three-dimensional reconstruction
showed a dilated sigmoid colon and descenaing bol5n. Seven days after bowel preparation,
the patient underwent low anterior resection with colorectal anastomosis (Rehbein
procedure). Histologicalstaining showed an aganglionic narrowed segment with a normal
distal rectum. The patient followed up for one and half years without any constipation or
other complications. Which of the following statements best describes the above case?
A. The most common type of intussusception is ileocolic. .

B. Chron's disease is associated with ulcers and fistula of the colon and ileum.
C.Volvulusmaybeseenatthedescendingpartofthecolon.
D. Hirschsprung's disease is characterized by the absence of parasympathetic Auerbach
plexus

7. Nerve injury can result from injections of commonly used antibiotics and other agents into the
buttock, especially in young infants. Such injection injury occurs in the hands of those
experienced in the care of infants and children. ln most of these a paralytic drop foot
resulting from nerve injury has been misdiagnosed as a congenital lesion or the result of
unrecognized poliomyelitis. Demonstration of sensory loss and the absence of sweating over
the distribution of nerve branches is the most valuable clue to the diagnosis. Surgical
exploration of the buttock revealed marked scarring in and about the nerve. Recovery has
been poor in this important, preventable, type of peripheral nerve injury which probably
occurs more commonly than is usually supposed. The midanterior thigh is the preferred
site for all intramuscular injections in infants and young children. Question: lntramuscular
Continued on Pagq 3
P HYSICIAN Licensure Examination
Saturday, September 9,2017 - 1 1:00 a.m. - 01:00 p.m.
ANATOMY AND HISTOLOGY SET A
injection should be given in the upper outer quadrant of the buttocks to prevent damage to
which of the following nerves?
A. Sciatic nerve C. Superior gluteal nerye
B. Obturator nerve D. Lateral femoral nerve
8. An Australian news correspondent who has been covering the battle between the Philippine
military and the lslamic State Maute militants has been struck by a stray bullet in the neck
while working in the besieged city of Marawi. lt all started when he was opening the back door
of a car to get food and water when he felt pain in the left side of his neck. He thought he'd
been hit by a bit of shrapnel. The blood flow was enough to suggest he should go to the
hospital. Doctors covered the wound - just under his left ear * with gauze. The x-ray showed
intact bullet lodged behind his neck, closed to his spine and narrowly missed a major blood
vessel arlery by 1 centimeter. What artery is this?
A. subclavian artery C. internal thoracic artery
B. carotid artery D. brachiocephalic artery
9. A 19-year-old college freshmen complained of generalized aches, chills, bacl<pain, and
urine discoloration. At first he said he hurt himself in an accident, but later admitted that for
the preceding three months he'd undergone fraternity hazing that included 700 to 1,000 hard
blows to the buttocks area with wooden paddles. The hazing was performed in a wooded
area off-campus between the hours of 1O p.m. to 3 a.m. Aside from obvious bruising, his
laboratory abnormalities included blood urea nitrogen of 89 mg/dL, creatinine of 137 mgidL
and other out-of-range values. Biopsy showed "focal acute tubular injtrry with occasional
muddy red-brown casts". The organ most likely to be traumatized in the above case is the:
A. spleen C. kidney
B. liver D. pancreas
10. A suspected drug personality was shot several times at the left upper abdomen after he
reportedly exchanged fire with police officers out to arrest him for drug trafficking. The
abdominal injuries caused by gunshot wound resulted to crushing of tissues and penetration
into his vital organs. Unfortunately, he died of recurrent bleeding 3 days after ICU discharge.
Question: Whichof following organs belowis not seen in the left upper quadrant of the
abdomen?
A. spleen C. gallbladder
B. kidney D. pancreas
1'1. Metal implants have been used in total hip replacement surgeries. The orthopedic surgeon
will select a hip replacement device for the patient based on their body structure, medical
history, and lifestyle. Many different types of designs and materials are currently used in
artificial hip joints. lt is made of a highly polished strong metal or ceramic material and a
durable cup of plastic or polyethylene. All hip implant devices can have complications. The
most common complications include implant dislocation and device wear. Researchers are
continuing to study different device surfaces in order to address the problems of wear debris
and osteolysis. Question: The hip joint is an example of:
A. hinge C. saddle
B. ball and socket D. condyloid
12. A S5-year-old man complained of abdominal pain in the right upper quadrant. Abdominal
ultrasound indicated acute cholecystitis and a single extremely large pear-shaped gallstone
(16.8 cm long, and 7.8 cm and its widest point and 4.1 cm at its narrowest point). Emergency
classical cholecystectomy was recommended. ln cholecystectomy, one of the most
important portions of the procedure is the identification of the Triangle of Calot since one of
its critical structure may not be found in the triangle and needs to be ligated before separating
the gallbladder from the liver beds and removed through one of the small incisions. Which
one of the following structure inside the Triangle of Calot need to be identified first?
A. cystic artery C. hepatic duct
B. cystic duct D. hepatic artery

Continued on Page 4
PHYSICIAN Licensure Examination
Saturday, September 9,2017 - 1 1:00 a.m. - 01:OO p.m.
ANATOMY AND HISTOLOGY SET A

13. A 1-month-old infant was examined at the Philippine Children's Hospital. Clinical symptoms
included bulging of anterior and posterior fontanelles and wider than normal sutures between
adjacent skull bones. He was diagnosed as having hydrocephalus. lt was determined that
there was a blockage in the ventricular system of the baby's brain between the third and
fourth ventricles. The blockage therefore must have involved the:
A. Cerebral aqueduct C. Foramen of Magendie
B. Foramen of Luschka D. lnterventricular foramen
14. Which of the following statements is incorrect?
A. The left kidney is slightly more higher than the right kidney due to the larger size of the
liver on the right side of the body.
B. The man's left testicle hang lower than the right testicle due to the way the blood drains
from the testes. The left testis drains into the renal vein at an angle while the right testis
drains directly to the inferior vena cava.
C. Foreign objects are more likely to lodge in the right primary bronchus because it is shorter,
larger and straighter than the left primary bronchus.
D. The left lung is higher and heavier than the right lung due to the compression of the
and the position of the heart.

15. The thoracodorsal nerve innervates the latissimus dorsi muscle which facilitates in certain
movements of the arm. Moreover, it can be used as a nerve graft in long thoracic nerye
injury whether in trauma or surgery. An injury to the thoracodorsal nerve might affect which
of the following movements?
A. lateral movement of the arm
B. flexion of the arm
C. rotation of the scapula
D. extension of the arm
16. A 42-year-old blackAmerican woman with a7-year history of hypertension first diagnosed
during her last pregnancy. Herfamily history is positive for hypertension, with her mother
dying at 56 years of age from hypertension-related cardiovascular disease (CVD). ln addition,
both her maternal and paternal grandparents had CVD. On her first clinic appointment, she
presented with complaints of headache and generalweakness. She reported that she has
been taking many medications for her hypertension in the past, but stopped taking them
because of the side effects. She could not recall the names of the medications. Currently she
is taking '100 mg/day atenolol and 12.5 mg/day hydrochlorothiazide (HCTZ), which she
admits to taking irregularly. Despite this antihypertensive regimen, her blood pressure
remains elevated, ranging from 150to 155/110to 114 mm Hg. ln addition, she admits that
she has found it difficult to exercise, stop smoking, and change her eating habits. Findings
from a complete history and physical assessment are unremarkable except for the presence
of moderate obesity (5 ft 6 in., 150 lbs), minimal retinopathy, and a Z5-year history of smoking
approximately one pack of cigarettes per day. lnitial laboratory data revealed serum sodium
138 mEq/L (135 to 147 mEqlL); potassium 3.4 mEq/L (3.5 to 5 mEq/L); blood urea nitrogen
(BUN) 19 mg/dl (10 to 2O mg/dt-); creatinine O.9 mg/dL (0.35 to 0.93 mg/dl); calcium 9.8
mg/dL (8.8 to 10 mg/dl); total cholesterol 268 mg/dl (. Z+S mg/dl); triglycerides 230 mg/dl
(<160mg/dL); andfastingglucosel05mg/dl(70to110mg/dl). Question: Which of the
following vessels is the prime controller of'blood pressure?
A. large artery C. arteriole
B. medium size vein D. medium size artery
17. A S9-year-old man with coronary artery disease was scheduled for coronary artery bypass
grafting. He had a myocardial infarction,S months ago. He was taking nitroglycerine, digoxin,
propranolol, isosorbide dinitrate (lsordil) and nifedipine. His blood pressure was 120180 mm
Hg and his heart rate 60 beats per minute. Question: Blockage of which of the following
arteries would lead to ischemia of the apex of the heart?
A. anterior interventricular C. right marginal
B. left circumflex D. right coronary

Continued on Page 5
PHYSICIAN Licensure Examination Page 5
Saturday, September 9,2017 - 11:00 a.m. - O'1:0O p.m.
ANATOMY AND HISTOLOGY SET A

18. ASo-year-oldmalewithacuterightflankpainpresented to the emergencydepartment. A


ureteric calculus with associated hydronephrosis was identified and he was prescribed pain
medications and discharged to pass the stone naturally. One day later, he returned to the
emergency department with severe pain and was referred to urology. He was managed
with a temporary ureteric stent and antibiotics. Question: Renal calculi are most commonly
trapped in which of the following anatomic narrowing of the ureter?
A. renal pelvis
B. as the ureter enters the bladder
C. at the level of the pelvic brim
D. at the level of the iliac crest
19. A7l-year old male was admitted to a hospital with a three-week history of jaundice,
pruritus, pale stools and dark urine. He had a history of alcohol abuse, a stable angina and
was on medication for hypertension. The differential diagnoses of hepatitis or gallstones or
cancer of the pancreas and gallbladder was made. His biochemical values were as follows:
a low serum albumin (25 g/l), raised total bilirubin (206 pmol/l), raised conjugated bilirubin
(173 pmol/l), raised gamma-glutamyl transferase (1 356 u/l), & raised alkaline plrosphatase
(1 127 u/l). During hospitalization (preoperatively), he was placed on a full ward diet.
On average, he consumed 6oolo of his food, and sometimes complained of vomiting after
meals. He also presented with blood glucose values ranging from 6-16.8 mmol/|. After a
computed tomography (CT) of the abdomen, the diagnosis of pancreatic carcinoma with
obstructive jaundice was made. Aorta calcifications were also noted. The patient underwent
a standard Whipple's operation. Extensive unresectable spreading of the tumor was noted.
Question: What is the most common site of pancreatic cancer?
A. head of the pancreas
B. neck of the pancreas
C. body of the pancreas
D. tail of the pancreas
20. A 45-year-old man comes to the physician because of right shoulder pain that began after he
chopped wood 2 days ago. Examination of the right upper extremity shows no obvious bone
deformities or point tenderness. The pain is reproduced when the patient is asked to externally
rotate the shoulder against resistance; there is no weakness. ln addition to the teres minor,
inflammation of which of the following tendons is most likely in this patient?
A. infraspinatus
B. pectoralis
C. supraspinatus
D. trapezius
21. Find the mismatched pair among the following choices:
A. retroperitoneal - ureters
B. intraperitoneal - jejunum
C. infraperitoneal - bladder
D. subperitoneal - adrenal glands
22. Vilamin A deficiency, often presenting with nyctalopia, has been described in,a number of
patients with malabsorption as a result of intestinal bypass surgery and, more recently,
bariatric surgery. ln these reports, vitamin A deficiency developed within several years
of gastric or intestinal surgery. Cases of patients who developed decreased vision
from vitamin A deficiency more than 18 years after their intestinal surgery are reported.
Vitamin A deficiency affects vision by inhibiting the production of rhodopsin, the eye
pigment responsible for sensing low light situation. Rhodopsin is found in tl're:
A. retina
B. cornea
C. iris
D. sclera

Continued on Page 6
PHYSICIAN Licensure Examination Page 6
Saturday, September 9,2017 - 1 1:00 a.m. - O1:00 p.m.
ANATOMY AND HISTOLOGY SET A

23. A Z2-year-old male presents to the emergency department with abdominal pain, anorexia,
nausea, and low-grade fever. Pain started in the mid-abdominal region 6 hours ago and is
now in the right lower quadrant of the abdomen. The pain was steady in 'nature and
aggravated by coughing. Physicalexamination reveals a low-grade fever (38"C; 100.5"F),
pain on palpation at right lower quadrant (McBurney's sign), and leukocytosis (12 x 10^9/L
or l2,OOOlmicrolitre) with 85% neutrophils. Question: ln appendectomy, the base of the
appendix can be identified during surgery by following the convergence of this slructure:
A. appendices epiplocae
B. sacculations
C. taenia coli
D. haustra
24 A 3S-year-old, right handed, elite, male bodybuilder presented with a two week history of
gradual onset of spontaneous right wrist drop. There was no evidence of recent trauma to
the brachial plexus, upper arm, or forearm to account for the condition. Past medical history
included left sided carpal tunnel syndrome, which resolved spontaneously in six months.
Physical examination showed considerable wasting of the brachioradialis muscle, loss of the
extensor carpi radialis and extensor carpi ulnaris, loss of digital extensorS, and loss of the
extensor pollicis longus and abductor polllcis longus. The triceps were preserved clinically.
Plain radiography did not show any sinister bony and soft tissue lesion. A nerve conduction
study showed severe acute motor denervation of muscles suggesting a lesion around the
spiral groove. ln view of spontaneous recovery of a previous compressive neuropathy without
surgical intervention, a similar approach of conservative treatment was started with non-
steroid anti-inflammatory drugs and physiotherapy. He was reviewed again after three
months; no improvement was found. Magnetic resonance imaging of the right arm did not
show any abnormal soft tissue mass, with a normal appearance of the humeral cortex and
marrow signal. As no recovery was noted after four months, exploration of the right affected
nerve was planned. The patient was placed in the lateral decubitus position, and a posterior
longitudinal incision was made. The affected nerve was identified after entering the interval
between the long and lateral heads of the triceps. The raphe was incised ,in the usual manner
to expose the nerve. The course of the nerve was traced proximally. There was a notable
extensive venous engorgement of the comitantes vessels. The nerve wad dissected
proximally to the inferior edge of the teres major, which appeared to be constricting the
underlying nerve. The aponeurotic edge of the muscle was incised 1.5 cm proximally allowing
free passage of the examining digit into the triangular space until no extrinsic compression
was felt. The lateral head of the triceps was extremely irritable to the point of spontaneously
twitching after the decompression was performed. The long head and medial head of the
tricepsweresparedbecauseof thehighoriginofthe branches of the affected nerve. This
finding showed that the level of entrapment was at the triangular space, which compromised
the nerve supply to the lateral head of the triceps, which has a lower origin. Recovery was
uneventful. He was started immediately on a physiotherapy regimen to mobilize his right arm.
Question: Based on the vignette above, which nerve is currently affected?
A. ulnar nerve
B. lateral cutaneous nerve
C. median nerve
D. radial nerve
25. A 61-year-old woman was presented with a chief complaint of a hand tremor that has
worsened over the past three years. The tremor occurs when she uses her hands for
activities such as slicing vegetables, sewing, putting on her lipstick, or typing at her
computer. Her handwriting has become messy and sprawling and very difficult to read. She
has worked for 23 years as an executive assistant; as a result of her tremor and the
associated difficulties, she is considering retirement. She notes that on the rare occasion
that she consumes alcohol, the tremor is somewhat improved. Family history is positive for
two grandparents who also "had the shakes" later in life. Her medications include an
antihypertensive, a statin, and an estrogen patch. Her BP is 120162. Height is 5'6", weight
139 lbs. On physical examination, mental status, cranial nerves, sensation, muscle
strength, tone, and deep tendon reflexes are all normal. There is a mild tremor bilaterally in

Continued on Page 7
PHYSICIAN Licensure Examination Page 7
Saturday, September 9,2017 - 1 1:O0 a.m. - 01:0O p.m.
ANATOMY AND HISTOLOGY SET A

both hands as she writes her name. No tremor at rest. There is no bradykinesia or rigidity
noted. Voice is somewhat tremulous. There is a slight involuntary back-and-forth horizontal
rotation of the head. Lab data, including lipids, TSH and FBS are all within normal limits'
Question: This slowly progressing neurologic disorder caused by a loss of secreting cells are
commonly seen in which part of the brain?
A. globus pallidus
B. red nucleus
C. substantia nigra
D. putamen
26. A 19-year-old marine was brought to the infirmary after passing out during basic training in
Mindinao. He had repeatedly complained of severe weakness, dizziness, and sleepiness
during the preceding 4 weeks of boot camp. ln a previous episode 3 weeks earlier, he had
drowiiness and generalized tiredness, and was brought to the infirmary, where after lV
administration oisaline, he was returned to duty with the diagnosis of dehydration. Upon
questioning, he reported unquenchable thirst, and the repeated need to urinate. A'lthough he
ate all of his rations as well as whatever he could get from his fellow trainees, he had lost 19
pounds. (Baseline body weight was 150 pounds, height 5'8"). On the last day, he complained
of vague abdominal pain, which wasworse on the morning of admission. He had vomited
once. During examination, he was oriented but tachypneic. He appeared paLer, dehydrated
with dry muCous membranes, and poor skin turgor. His respiratory rate was 36lminute with
deep, llborious breathing; his heart rate was 138/minute regular, and his blood pressure was
90/60. His chest was cle-r, heart tones were normal. There was an ill-defined :generalized
abdominal tenderness, which was otherwise soft to palpation and showed no rebound. There
was a generalized muscular hypotonia; his deep tendon reflexes were present but very weak.
Laboratory, on admission, showed glucose of 560 mg/dl, sodium 154, potassium 6.5, pH 7 .25,
bicarbonate 1OmM/liter,chloride 90, BUN,38mg/dl, creatinine-2,5mg/dl. (Normal values:
glucose, 70-114 mg/dl; Na = 136-146; K, 3.5-5.3: Cl, 98-108; GO2, 2}-32i1att in mMll]; BUN,
1-22mgldl; creatinine,0.7-1.5 mg/dl). A urine sample was 4+ for glucose and had "large"
acetone. HbAl c was 14o/o (n=4-6.2o/o). Serum acetone was 4+ undiluted, rand' still positive at
the 4h dilution. Beta-Hydroxybutyrate level was 2O millimols/liter (normal=O.O-0.3 mM/l).
He was treated with insulin and saline l.V. By the 4th hour of treatment, potassium chloride
was added to the lV at a rate of 15 mEq/hour. Sixteen hours later, he was active, alert, well
hydrated and cheerful, indicating he felt extremely well. He requested that his lV be
discontinued. His physician decided to switch his insulin to subcutaneous injections and to
start a liquid diet. He was later put on a diabetes maintenance diet and treated with one
injection of Human Lente insulin in the morning. Although his blood sugars the next morning
were 100-140mg/dl, he had frequent episodes of hypoglycemia during the day, and his
HbAl c was 97o. Eventually, he was put on 3 injections of regular insulin/day, and a bedtime
intermediate duration (Lente) insulin. Question: Which of these cells in the pancreas can
lead to increase blood glucose once attacked or destroyed by the immune system over a
long period of time?
A. q-alphacells
B. B-betacells
C.6-deltacells
D.8-epsiloncells
27. A six-month-old male presents to the emergency department with a history of tethargy. He
was seen 3 days ago with fever and URI symptoms, diagnosed with otitis media and treated
with oral amoxicillin.. This morning he had become irritable and was less active than usual.
He has vomited three times and his urine output is noticeably decreased. He has no diarrhea.
Vital signs: Temp = 40.0, Pulse = 90, RR = 30 (irregular), BP = 12Ol9O, weight = 8kg. He is
lethargic and arousable only to painful stimuli. His anterior fontanel is full and tense, and he
has questionable neck rigidity. His tymoanic membrane are red and bulging. His pupils are
reactive, but his eyes do not focus w'ell on his parents. His heart, lung s and abdomen
are normal. His color and perfusion are good. He has no petechiae. He moves all his
extremities weakly and his DTRs are hyperactive. A CBC, blood culture and chemistry
panel are drawn. An lV is started. Since an increased ICP (intracranial pressure) is

Continued on Page I
PHYSIiIAN Licensure Examination
Saturday, September 9,2017 - 1 1:00 a.m. - 01:00 p.m-
ANATOMY AND HISTOLOGY SET A

suspected, a lumbar puncture (LP) is initially delayed and he is immediately given 500 mg
of ceftriaxone lV. A siat CT scan of the brain is normal, so an LP is done and the CSF
(cerebrospinal fluid) is visibly hazy. An infectious disease consultant is called to inquire
about lV dexamethasone and vancomycin. Both are reco'mmended and given. The CSF
results return t hour later showing 450 WBCs, 957o segs, 57o monos, total protein 75'
glucose 25 mg/dl. Gram stain of the CSF shows many WBCs with few gram positive cocci.
He was admitted to the pediatric lCU. Question: While doing a spinal tap, the spinal needle
is inserted between the:
A. Ll and L2 vertebrae
i

B. L2 and L3 vertebrae r

C. L3 and L4 vertebrae
D. L4 and L5 vertebrae
28. A16-year-oldgymnastpresentstotheemergency department after landing awkwardly on
her ankle. She is diagnosed with a sprained ankle. Which of the following ligaments is most
commonly injured in an ankle sprain? ir

A. anterior talofibular ligament


B. calcaneofibular ligament
C. talonavicular ligament
D. tibiotalar ligament
29. The following nerve structures passes thru each corresponding foramina. Which of the
following pairs is a mismatched?
A. vagus nerve - jugular foramen
B. mandibular nerve 0/3) - foramen ovale
C. abducens nerve - foramen lacerum
D. otfactory nerye - cribiform plate of ethmoid
30. A 1S-year-old boy is brought to the emergency department because he has pain in his hand
following a fist fight. The physician tells the patient that he has broken his hand. Which of the
following is the most likely site of this patient's fracture?
A. distal radius C. metacarpals
B. hamate D. phalanges
31. An unconscious S7-year-old man is brought to the emergency department by ambulance with
massive, bright red emesis. On arrival, his blood pressure is 80/40 mm Hg and his heart rate
is 124lmin. He appears jaundiced with multiple spider angiomas on his chest and arms. He
has an enlarged abdomen that is dull to percussion and positive for a fluid wave. He
has splenomegaly and muscle wasting'in his extremities. Which of the following vessel
anastomoses i- responsible for the patientrs bleeding?
A. left gastric artery and left gastric vein l

B. left gastric vein and azygos vein


C. portal vein and inferior vena cava
D. splenic vein and left renal vein
32. During dental procedures, it is possible that smallfragments may be aspirated into the trachea
andcauseaspirationpneumonia. lfthepatientissittingupright during the procedure, which
of the following is the most common site of aspiration pneumonia?
A. left lower iobe C. right lowei lobe l

B. left upper lobe O. rignt upper lobe I

33. During autopsy of a 65-year-old woman, the liver is examined, revealing multiple tumors of
various sizes throughout both lobes. This patient, along with the fact that most tumors found
in the liver are metastases, leads the pathologist to suspect that a primary tumor exists in
another organ. Which of the following is most likely location of primary tumor?
A. breast C, kidney
B. colon D. lungs

Continued on Page 9
PHYSICIAN Licensure Examination Pa$e.9
Saturday, September 9,2017 - 1 1:OO a.m. - 01:OO p.m.
ANATOMY AND HISTOLOGY SET A

34. Prior to histology laboratory session, a first-year-medical student described the following
structure under the low power objective compound microscope: The structure of the
muscularis mucosa, submucosa and muscularis externa and serosa is typical. The
external longitudinalmuscle layer is modified that the muscle is arranged in three, thick,
flat bands witn tittte or no muscie between them. Villi are absent. The free surface appears
smooth and is indented at close intervals by long tubular intestinal glands. The structure in
details is the histological appearance of the large intestine. Question: Which of the following
is NOT characteristic of the large intestine?
A. presence of Taeniae coli
B. presence of Crypts of Lieberkuhn
C. presence of Valves of Kerckring
D. presence of Haustration
35. Paracentesis (tapping of the abdomen below the umbilicus) may be necessary to withdraw
excess peritoneal fluid. lf the cannula is inserted in the midline of the anterior abdominal wall,
it will pass through the following anatomic structure, except the
A. skin and fascia
B. linea alba
C. transversalis fascia, extraperitoneal fat and parietal peritoneum
D. rectus abdominis
36. Slide No. 1: The lining epithelium of the cavity of the organseen as a dark narrow area on
the free surface differentiated from the underlying tunica propria by the row of deeply stained
nuclei. The presence of cilia on the distal end of the columnar cells. They appear short and
fine, visible when the fine adjustment is turned up and down. The centrosomal granules at
the distal end are not visible. You may see in the specimen an oval clear spaces within
theepithelium.Thesearegobletcells. Atthedistalhalf oftheliningepithelium which is just
cytoplasm devoid of nuclei. ln the proximal half of the epithelium you will notice that the nuclei
of the cells of the epithelium appear in several layers. The slide you are looking at is a slide
of the trachea. Question: What is the lining epithelium of this organ?
A. stratified squamous, non-keratinized
B. simple cuboidal
C. pseudostratified columnar, ciliated
D. simple squamous
37. A 4O-year-old female with end-stage renal disease (ESRD) by hypertension on hemodialysis
at the Kidney Hospital since 2014 had 6een experiencing anemia, resistance to recombinant
human efihropoietin (rHuEPO) and need for blood transfusion twice a week in the last four
months. She had not provided any history of hematuria, gastrointestinal, cardiac, liver
orcollagen disease. Medication include intravenous iron (lOOmgweekly), subcutaneous
THuEPO (484 lU/kg/wk), oral folate, B complex, amlodipine, atenolol, sevelamer and
omeprazole. Over the preceding 2 months, the patient complained of progressive asthenia
and weakness. She experienced an episode of hematemesis and hypotension (Mean
Arterial Pressure: 49 mm Hg) during dialysis session. On physical examination, mucosal
pallor and tachydyspnea were observed. There were no stigma of chronic liver disease
and rectal examination was positive for melena. There was no Raynaud's phenomenon.
Hematologic tests showed a severe decrease in hemoglobin (Hb) from 9.6 g/dl to 4.8 gldL,
hematocrit from 31o/o to 15.67o and reticulocytosis (2.8%). Decreased Mean Corpuscular
Volume (72 me1, normal Mean Corpuscular Hb (25 pg) and Mean Corpuscular Hb
Concentration (29o/o) are also seen. Whereas that RDW 16.20/0, normal platelet count
(221.OOO mm"), activated partial thromboplastin time (28 seconds) and lnternational
Normalized ratio (lNR1.1). The patient was transferred to the intensiveecare unit for
hemodynamic instability and required 4 units of packed red blood cells (RBC) transfusions
to treat anemia and crystalloid solution (2.5 L saline). Question: ln the kidney, which cells
produce erythropoietin?
A. mesangialcells C. podocytes
B. J-G cells D. macula densa cells

Continued on Page 1O
PHYSICIAN Licensure Examination Page 10
Saturday, September 9,2017 - 11:OO ?.ffi,: - O1:OO p.m.
ANATOMY AND HISTOLOGY SET A

38. Which of the following statements is incorrect?


A. The paired cranial bones include parietal bones and temporal bones.
B. The corpora spongiosum are paired sponged-like regions of erectile tissues which
contains most of the blood in the penis during an erection'
C. The left lungs is divided into two lobes, an upper and a lower, by the oblique fissure.
D. There are 2 floating ribs which are attached only to the vertebrae and not to the sternum
or cartilage of the sternum.

39. Slide No. 2: The slide was focus under low power and noted the following division of the
gland into an outer cortex and inner medulla. CORTEX: this portion is divided into 3 ill-defined
i-ayers or zones that are recognized by the specific arrangement of the parenchymal
celts in each zone. Zona glomerulosa - a narrow area beneath the capsule consisting of
columnar cells arranged in spherical groups or hooked. Zona fasciculata - the widest zone
of the cortex showing columnar cells arranged in radiating columnar fashion. These are fat
droplets in the cytoplasm giving "foamy" appearance hence these cells are sometimes called
spongiocytes. Zona reticularis-the thinnest portion of the cortex, passing to the medulla.
The small, polygonal shaped cells are arranged in very short anastomosing cords.
MEDULLA: lrregular boundary with the cortex. Small polygonal dark staining cells arranged
in spherical groups or short anastomosing cords containing chromaffin granules. Large thick
walled central veins. Autonomic ganglion cells in the connective tissue of the stroma.
The structure in details is the histological appearance of the adrenal glands. Question: Each
of the three main differentiated zones secretes a specific profile of steroid hormones, which
of the following is a mismatched?
A. Zonaglomerulosa - aldosterone
B. Zonafasciculata cortisol
C. Zona reticularis androgens
D. Allare correctly matched
40. Most commonly fractured bone in the body?
A. scapula
B. clavicle
C. rib
D. humerus
41. Slide No. 3: Allfour layers - mucuous membrane, submucosa, muscular and outermost fibro-
serous coats are identified. Aggregations of confluent lymph nodules in the tunica propria of
the organs found below the level of the intestinal villi. These are Peyer's patches. The type
of lining epithelium - simple columnar cells with a striated cuticular border and goblet cells.
Small simple tubular glands, few in number, found among the lymph nodules. These are
crypts of Leiberkuhn or intestinal glands. Over the dense mass of lymphoid tissue, there are
few or absent intestinalvilli. The Peyer's patch produce a bulging upwards towards the lumen
and downward in the direction of the submucosa. The structure in details is the histological
appearance of the small intestine. Question: Peyer's patches are absent in the following
structures, except the
A. jejunum
B. duodenum
C. ileum
D. all of the above
42. Traumatic injury to the perineum in the male may rupture the bulb of the penis or penile
urethra. The resulting leakage of blood or urine may be found in all of the following , except:
A. anterior abdominal wall
B. ischio rectal fossa
C. scrotum
D. superficial perineal pouch

Continued on Page 11
PHYSICIAN Licensure Examination Page 11
Saturday, September 9, 2017 - 11:OO a.m. - 01:OO p.m.
ANATOMY AND HISTOLOGY SET A

43. A brain tumor of unknown origin was surgically removed from an elderly male. Radiation
therapy was administered to ensure that tumor cells not removed do not repopulate the brain.
Which-of the following cells would be decreased in number after radiation therapy?
A. microglial cells
B. fibrous astrocytes
C. oligodendrocytes
D. neurons
44. Melatonin which regulates the day and night cycle is secreted by what endocrine gland?
A. adrenal gland C. Pituitary gland
B. pineal gland D. thYroid gland
45. All of the following commonly occur on inhalation, except
A. the diaphragm descends
B. the external intercostal muscles contract
C. the abdominat muscles contract and push the abdominal viscera cranially
D. the ribs are raised
46. Destruction of these cells will lead to lods of intrinsic factor in the gastric mucosa:
A. parietal cells C. G cells
B. 4ymogenic cells D. mucous cells
47. Which of the following muscle depresses the mandible?
A. lateral pterygoid muscle
B. medial pterygoid muscle
C. masseter muscle
D. temporalis muscle
48. To test trapezius muscle paralysis, you would ask the patient to:
A. flex the arm fully
B. adduct the arm against resistance
C. push against a wall with both ands
D. shrug the shoulder
49. All of the following cranial nerves have parasympathetic component, except the
A. trigeminal nerve
B. oculomotor nerve
C. facial nerve
D. vagus nerve
50. A 3S-year-old man injures his lateral pterygoid muscle. Which of the following activities will
be adversely affected by his injury?
A. smiling C. whistling
B. swallowing D. yawning
51. The following structures are transmitted by the aortic opening, except the
A. a4ygous vein
B. inferior vena cava
C. thoracic duct
D. aorta
52. The spermatic cord contains the following structure, except
A. scrotal arteries and veins
B. vas deferens
C. pampiniform plexus
D. testicular atlery

Continued on Page 12
PHYSICIAN Licensure Examination Page 12
Saturday, September 9,2017 - 1 1:OO a.m. - 01:OO p.m.
ANATOMY AND HISTOLOGY SET A

53. The primary flexor of the elbow:


A. brachialis
B. bicep branchii short head
C. bicep branchii long head
D. brachioradialis
54. Which of the following tissues would most likely yield the highest percentage of
mucus-secreting cells?
A. parotid gland
B. sublingual gland
C. submandibular gland
D. esophageal mucosa
55. The celiac artery arises from the abdominal aorta at the level of:
A. T6
B. T8
C. T1O
D. T12
56. Agranular leukocytes with kidney-shaped nucleus and extensive frosted glass cytoplasm:
A. basophils
B. monocytes
C. neutrophils
D. eosinophils
57. The following veins drain directly into the inferior vena cava, except the:
A. hepatic veins
B. renalveins
C. lumbar veins
D. inferior mesenteric veins
58. What is the most powerful flexor muscle of the thigh?
A. vastus lateralis muscle
B. sartorius muscle
C. iliopsoas muscle
D. rectus femoris muscle
59. The auscultatory triangle is the site on the back where breath sounds may be most easily
heard with a stethoscope. Which of the following does not formed the triangle of auscultation?
A. medial border of the scapula
B. teres major muscle
C. latissimus dorsi muscle
D. trapezius muscle
60. The following except one are examples of primary receptive areas of the brain. Areas outside
the primary receptive areas are called association areas. Which one of the following is
considered an association areals:
A. Brodmann area 4
B. Brodmann areas 3,1,2
C. Areas 41 and 42
D. Areas 18 and 19
6'1. The bulk of muscles that make up the floor of the oral cavity:
A. genioglossus
B. stylohyoid
C. mylohyoid
D. digastric

Continued on Page 13
PHYSICIAN Licensure Examination Page 13
Saturday, September 9,2017 - 1 1:OO a.m. - 01:OO p.m.
ANATOMY AND HISTOLOGY SET A

62. The cephalic vein is normally a tributary to the:


A. brachialvein
B. axillary vein
C. medial brachialvein
D. subclavian vein
63. Arthrocentesis is most difficult to carry out in which joint?
A. wrist
B. elbow
C. ankle
D. knee
64. Which of the following arteries is most likely eroded in perforated peptic ulcer disease?
A. gastroduodenal
B. retroduodenal
C. splenic
D. hepatic
65. The part of the male reproductive tract which carries only semen within the prostate gland is
the:
A. prostatic urethra
B. membranous urethra
C. ductus deferens
D. ejaculatory duct
66. The following veins drain into the right atrium, except
A. coronary sinus
B. superior vena cava
C. anterior cardiac
D. great cardiac
67. Which of the following is not a muscle of the posterior triangle of neck?
A. superior belly of omohyoid : :

C. splenius capitus
D. anterior scalene
68. The small intestine receives blood supply from the:
A. splenic artery
B. left gastric artery
C. inferior mesenteric artery
D. superior mesenteric artery
69. Which part of the brain is responsible for sensation and perception?
A. frontal C. parietal
B. temporal D. occipital
70. Which of the following is a membranous labyrinth?
A. semicircular canal
B. utricle
C. vestibule
D. cochlea
71. The most common site for palpation of tlre pulse in the upper extremity:
A. axillary artery
B. radial artery
C. ulnar artery
D. brachial artery

Continued on Page 14
PHYS ICIAN Licensure Examination Page 14
Saturday, September 9,2017 - 1 1:OO a.m. - 01:OO p.m.
ANATOMY AND HISTOLOGY SET A

72. lnjury to the common peroneal nerve results in an:


A. inability to invert the foot
B. inability to plantar flex the ankle
C. inability to evert the foot
D. inability to plantar flex the big toe
73. Considered the widest segment of the colon:
A. rectum C. sigmoid
B. cecum D. transverse
74. When passing a needle through the chest wall into the pleural cavity in the midaxillary line,
the following structures will be pierced exeept the:
A. skin
B. parietal pleura
C. levator costarum
D. internal intercostal muscle
75. Which of the following contains the MOST actin and myosin?
A. endometrium C. mYometrium
B. perimetrium D. muscularis serosa
76. The lesser omentum contains the following important structure, except
A. portalvein C. common bile duct
B. hepatic artery proper D. inferior vena cava
77 . Each of the following muscles form the boundary of the axilla, except
A. serratus anterior muscle
B. subscapularis muscle
C. pectoralis major muscle
D. supraspinatus muscle
78. Cells found in the Crypts of Lieberkuhn that secretes antibacterial enzymes are called:
A. goblet cells
B. paneth cells
C. argentaffin cells
D. clara cells
79. The smallest of all the carpal bones and classified as a sesamoid bone:
A. trapezium C. lunate
B. pisiform D. traPezoid
BO. The following structures pass through the esophageal hiatus in the diaphragm, except
A. the left vagus nerve
B. branches of the left gastric artery
C. the left phrenic nerve
D. a tributary of the Portal vein
81. The rotator cuff consist of the following muscles, except
A. subscapularis muscle
B. teres minor muscle
C. teres major muscle
D. supraspinatus muscle
82. Kyphosis is an accentuated or abnormal curvature of which region of the spine?
A. cervical
B. thoracic
C. lumbar
D. sacral

Continued on Page 15
PHYSICIAN Licensure Examination Page 15
Saturday, September 9,2017 - 1 1:O0 a.m. - O1:OO p.m.
ANATOMY AND HISTOLOGY SET A

83. A 3-year old girl ruptured her eardrum when she inserted a pencil into her ear. Her mother
took her to the emergency department after noticing that the child was crying and complaining
of pain in her ear with a few drops of blood in the external auditory meatus. The attending
doctor examined the child for possible injury to a nerve that runs across the eardrum. The
most likely nerve to be injured is the:
A. auricular branch of the vagus
B. chorda tympani
C. glossopharyngeal
D. lesser petrosal
84. Which of the following bones is classified as flat bone:
A. humerus C. scapula
B. talus D. clavicle
85. The following nerve structures participates in the reception of sound, except
A. trigeminal nerve C. facial nerve
B. lateral lemniscus D. medial lemniscus
86. The right posterior veins drains mostly into which of the following veins?
A. hemiazygos vein C. azygos vein
B. accessory a4ygos vein D. inferior vena cava
87. A previously healthy 36-year-old Japanese woman presented with blood pressure that was
difficult to control after she had undergone an emergency cesarean section at 29 weeks of
gestation because of severe hypertension and fetal growth restriction. During pregnancy, her
blood pressure had been more than 180/1OO mm Hg, and it continued to be more than 160/90
mm Hg after detivery. The patient began taking 40 mg of nifedipine, and the blood pressure
level stabilized at approximately 140/8O mm Hg. Computed tomographic (CT) imaging
revealed a completely disrupted abdominal aorta, severe stenosis of the renal arteries
bilaterally, and development of numerous collateral arteries. A diagnosis of Takayasu's
arteritis was made, and renat-artery stenosis was presumed to be causing secondary
hypertension. Question: Takayasu's arteritis also called pulseless disease is a form of
g ranulomatous vasculitis affecting:
A. small sized arteries
B. small and medium sized arteries
C. medium and large sized arteries
D. large sized arteries
88. The dorsalis pedis pulse is derived from which artery?
A. femoral artery
C. posterior tibial artery
D. poplitealartery
89. Which one of the following contributes to the blood supply of the pancreas?
A. left gastroepiploic artery
B. inferior mesenteric artery
C. hepatic proper airtery
D. splenic artery
90. A 1S-year-old boy was admitted to the emergency room for having large bowel obstruction
resulting from left-sided indirect inguinal hernia. The most likely intestinal segment involved
in this obstruction is the:
A. ascending colon
B. cecum
C. descending colon
D. sigmoid colon

Continued on Page 16
PHYSICIAN Licensure Examination Page 1,6
Saturday, September 9,2017 - 1 1:OO a.m. - 01:OO p.m.
ANATOMY AND H]STOLOGY SET A

91. Which of the following structures passes through both greater and lesser sciatic foramens?
A. pudendal nerve C. inferior gluteal nerye
B. superior gluteal nerye D. sciatl'c nerve
92, A muscle that protrudes the tongue is the:
A. styloglossus muscle
B. hyoglossus muscle
C. genioglossus muscle
D. pataloglossus muscle
93. A "pulled elbow" in a young child results when the radial head is dislodged from the:
A. annular ligament
B. insertion of biceps brachii
C. olecranon process
D. radial collateral ligament
94. To what vein does the left ovarian vein drain into?
A. renal vein C. Portal vein
B. inferior vena cava D. iliac vein
95. lnjury to the sciatic nerve would affect the action of this muscle:
A. adductor magnus C. gracilis
B. obturator externus D. adductor brevis
96. The spinal cord in the adult usually ends inferiorly at the following vertebral level:
A. at the upper border 51
B. at the lower border of L1
C. at the lower border of 54
D. at the upper border of cocclx
97. Where is the dangerous area of the face?
A. root of the nose and two angles of the mouth
B. tip of the nose and two angles of the mouth
C. root of the nose and two angles of the jaw
D. tip of the nose and two angles of the jaw
98. The widest part of the fallopian tube and the most frequent site of fertilization is the:
A. infundibulum C. ampulla
B. isthmus D. intramunal
99. The midbrain is a derivative of what secondary brain vesicle?
A. telencephalon C. mesencephalon
B. diencephalon D. metencephalon
100. Talaria are winged sandals worn by the gods Mercury (Roman messenger) and Hermes
(Greek messenger). The wings are depicted closed to the:
A. ankle bone
B. heel bone
C. metatarsal bone
D. phalangeal bone
*** END ***

WARNING: Failure to submityour TestQuestions(Complete)set will cause the cancellation


of your Test-Result for this subject.
ANATOMY & HISTOLOGY 2017
ANSWER KEY

1. B 26. B 51. B 76. D


2.C 27. D 52. A 77. D
3.D 28. A 53. A 74. B
4.C 29. C 54. B 79. B
5.8 30. c 55. D 80. c
6.D 31. B 56. B 81. C
7. A 32. C 57. D 82. B
8.8 33. B 58. C 83. B
9.C 34. C 59. B 84. C
10. c 35. D 60. D 85. D
11. B 36. C 61. C 86. C
12. A 37. A 62. B 87. C
13. A 38. B 63. C 88. B
14. D 39. D 64. A 89. D
15. D 40. B 65. D 90. D
16. C 41. C 66. D 91. A
17. A 42. B 67. A 92. C
18. C 43. D 68. D 93. A
19. D 44. B 69. C 94. A
20. A 45. C 70. B 95. A
21. D 46. A 71. B 96. B
22. A 47. A 72. C 97. A
23. C 48. D 73. B 98. C
24. D 49. A 74. D 99. C
25. C 50. D 75. C 100. A
Republic of the Philippines
PROFESSTONAL REGULATION COMMISSION
Manila
BOARD OF MEDICINE
PHYSICIAN Licensure Examination
Saturday, September 9, 2017
MICROBIOLOGY AND PARASITOLOGY SET A

INSTRUCTION: Select the correct answer for each of the following questions.
Mark onlv one answer for each item by marking the box corresponding to the letter of your choice
on the answer sheet provided. STRICTLY NO ERASURES ALLOWED. Use pencil no. 2 only.
MULTIPLE CHOICE:

1. After bathing at a hot spring resort, a7l-year--old man presented to the emergency department
because of seizure - like attack with loss of consciousness. The fatal disease may
have been contracted from contaminated water during bathing, A case of primary amebic
meningoencephalitis (PAM) was diagnosed based on the clinical presentation and
detection of actively motile trophozoites in his cerebrospinalfluid using wet-mount smear
and Liu's stain. The amoeba was further confirmed by PCR and gene sequencing.
Question: Which of the following should not be included in the treatment of PAM caused by
Naegleria fowleri?
A. Amphotericin B
B. Miltefosine
C. Azithromycin
D. Paromomycin
2. A 55-year-old male had a history of four weeks cough with blood - tinged expectorate.
Sputum examination was done which revealed the presence of operculated ova. Name
the intermediate host of this parasite:
A. Gyraulus convexiusculus
B. Sundathelphusa philippina
C. Oncomelania quadrasi
D. Ctenopharyngodon idellus
3. A25-year-old school teacher presented to the emergency room with the following complains
of headache, neckstiffness, vomiting, high-grade fever, photophobia and petechial rashes.
P.E. reveals an ill-appearing woman unable to flex her neck without eliciting pain and diffuse
petechial rashes in her extremities. Kernig and Brudzinski signs are positive. CSF analysis
reveals low glucose, increased protein and increased neutrophils. Which of the following is
the most likely causative agent?
A. Escherichia coli
B. Listeria monocytogenes
C. Neisseria meningitidis
D. Haemophilus influenzae
4. A12-year-old boy presentedwith acute onset of sore throat, fever to 38.9"C and painful
anterior cervical lymphadenopathy. On exam, the pharynx is red and swollen and the tonsils
are covered with yellow-white exudate. The child has halitosis. What set this infection apart
from other pathogenic bacteria is its potential to cause delayed non-suppurative complications
like acute rheumatic fever and acute glorrrerulonephritis. Question: Which one of the following
organism is most likely responsible for this boy's infection?
A. Streptococcus pyogenes
B. Streptococcus agalactiae
C. Staphylococcus saprophyticus
D. Staphylococcus epidermidis

Continued on Page 2
PHYSICIAN Licensure Examination Page'2
Saturday, September 9,2017 - O2:O0 p.m. - O4:OO p.m.
M ICROBIOLOGY AND PARASITOLOGY SET A

5. A 26-year-old seaman who works with an oil-exploration company in West Africa comes to
the STD clinic with a S-day history of burning on urination and a 3-day history of non-purulent
urethral discharge. He is sexually active with many female partners and does not use condom.
There is no history of prior sexually transmitted diseases. Laboratory finding from endourethral
exudate shows abundant intracellular diplococci in neutrophils. What organism is the most
likely etiology of the illness?
A. Treponema pallidum
B. Neisseria gonorrhea
C. Haemophilus ducreyi
D. Chlamydia trachomatis
6. A 23-year-old woman present to the physician with vaginal itching and burning. On
examination, she has a frothy, foul-smelling, greenish vaginal discharge. A swab sample is
taken and a slide is prepared. Microscopy results showed negative for clue cells, pseudo-
hyphae and D-shaped egg. Which of the following gynecologic infections most likely the cause
of the woman's illness?
A. Candida albicans
B. Gardnerella vaginalis
C. Trichomonas vaginalis
D. Enterobius vermicularis
7 . A 3-year-old child who spends 5 days a week at daycare center while her parents work
developed a mild grade fever of 38.3oC. Tlre fever persisted for the next 4 days. She is also
drowsy and complains that her head ahd tummy hurt and she does not want to eat. The
parents call the doctor and he recommends antipyretic medication to keep the fever down
and increase oral fluid intake to keep the child's body hydrated. By the fifth day, her parents
noticed that she has developed a rash on her head and chest that is spreading to her back.
The lesions are 1 to 4 mm in diameter. Her parent's become concern and bring her to their
primary care physician for diagnosis. Which of the following is the most likely diagnosis of
this child's illness?
A. Measles
B. Roseola
C. German measles
D. Chicken pox
8. 47S-year-old patient with an episode of acute urinary retention was catheterized. Three
days later, he developed fever and suprapubic pain. Culture of the urine revealed a thin film
of bacterial growth over the entire blood agar plate, and urease test was positive. Which of
the following is the most likely organism to cause this infection?
A. Proteus mirabilis
B. Escherichia coli
C. Serratia marcescens
D. Pseudomonas aeruginosa
9. A 9-month-old baby is rushed to the emergency complex because of severe cough and
fever. On physicalexamination, rales and high-pitched wheezes are heard on his lung fields.
The child also presents with acute otitis media and bluish discoloration of skin due to lack of
oxygen. X-ray result showed bilateral patchy consolidation in a predominantly pre-hilar
distribution extending to the bases. The diagnosis given is pneumonia. Which of the following
viruses is the most likely cause?
A. Rhinovirus c. Adenovirus
B. Respiratory syncytial virus D. Coronavirus
10. A 4 day old male neonate born to 23 years old G2P1 mother with term pregnancy with no
sepsis by cesarean delivery was transferred to maternity ward. Baby was average birth
weight (2.9K9) and ready to accept breast feeding. ln first 48 hours, there was less secretion
of milk. lnspite of counseling, formula feed was given in first 48 hours. On 4th day of life, he
developed loose watery stool, 10-15 times, curdy white and had fishy odor with 6-7 episodes

Continued on Page 3
PHYSICIAN Licensure Examination Page 3
Saturday, September 9,2017 - O2:OO P.m. - O4:OO p.m.
MtcF,oBlolocy ANp pARASlroLoGy
:
:FT "
of vomiting. There was a history of abdominal distension, reduced urine output, lethargy and
refusal to feeds. On further inquiry, grandparent totd that water was used from restaurant for
dilution of formula. On examination, baby was irritable with sunken eyes, depressed anterior
fontanelle and dry oral mucus membrane. There was prolonged capillary filling time and
feeble peripheral pulses with HR-168/min and RR-64/min. Blood investigation showed
evidence of leucocytosis and hyperglycenria with no azotemia and dyselectrolytemia. CRP
and blood culture was negative. Stool sarnple was sent for hanging examination and culture.
Stoolexamination showed darting motility on hanging drop method and stoolculture revealed
growth of bacteria on TCBS medium. Baby was immediately resuscitated with Ringer lactate
and was treated with Ciprofloxacin. Which of the following organisms is the most likely cause
of this child's diarrhea?
A. Vibrio cholera C. Escherichia coli
B. Giardia lamblia D. Salmonella typhi
11. A19-year-oldwomanwasreferredtothehospital withcomplaints of productive cough and
massive hemoptysiS. The patient was a young housewife originally from the northern part of
the Philippines. Her illness started 30 days ago with fever, night sweats, weakness, and
weight loss of four Kg, that followed with productive cough and hemoptysis, about 20 cc of
blood in her sputum for 3 times. A chest X-ray was done and showed cavitary lesions and
nodules in both lungs. Direct smear of sputum for acid fast bacilli was positive three times.
Histopathological study showing epithelioid granuloma, Langhan's giant cell and caseous
necrosis consistent with tuberculous granuloma. Question: Which of the following categories
of hypersensitivities is consistent with the above case?
A. Type I Hypersensitivity Reaction
B. Type ll Hypersensitivity Reaction
C. Type lll Hypersensitivity Reaction
D. Type lV Hypersensitivity Reaction
12. A sexually active 35-year-old businessm€rn comes to the physician for evaluation because
of local non-tender ulcer that have appeared on his genitalia 2 weeks ago. He was also
presented with fever, headache and body malaise. He has no history of allergies and takes
no other medications. His only medical problem in the past was being treated with gonorrhea
about a year ago. On exam, he has no palpable cervical, axillary or inguinal adenopathy. His
body has no erythematous or maculopapular lesions, including his palms and soles of his
feet. Question: Which of the following laboratory tests would best confirm the diagnosis of
syphilis infection in the above patient?
A. Darkfield Microscopy C. VDRL
B. RPR D. FTA-ABS
13. Thisviruscausemononucleosissyndromeinyoung adults which is characterized by fever
and pharyngitis. lt can also cause severe infection in immunocompromised patients, which is
characterized by retinitis, pneumonia and even death. For infected pregnant mothers, it
can result in teratogenesis of developing fetus. Questions: What etiologic agent should you
suspect and what antiblotic is best for the above presentation?
A. Herpes Simplex Virus treated with Acyclovir
B, Cytomegalovirus treated with Foscarnet
C. Epstein Barr Virus treated with Ganciclovir
D. Human lmmunodeficiency Virus treated with Zidovudine
14. On his return trip to South America, a 43-year-old man was presented with chest pain,
difficulty of swallowing, and frequent regurgitation of food and liquids. A radiological exam
was requested based on the patient's clinical sign and symptoms which showed tapering
"bird's beaks sign" in the inferior esophagus consistent with Achalasia diagnosis. Which of
the following is the most likely protozoan agent linked to this case?
A. Leishmania donovani
3 ii)",Iffi3:?"':T;:,
D. Trypanosoma gambiense

Continued on Page 4
PHYSICIAN Licensure Examlnation Page 4
Saturday, September 9, 2017 - O2:OO p.m. - O4:OO p.m.
M ICROBIOLOGY AN D PARASITOLOGY SET A

15. A 1S-year-old teenager is brought to the office for evaluation of a cough and fever. His illness
began several days ago with low-grade fever, headache, myalgias, and fatigue, and has
slowly worsened. He now has a persistent cough. He has tried multiple over-the-counter cold
and cough medications without relief. He has no significant medical or family history. No
family members have been ill recently, but one of his good friends missed several days of
school about 2 weeks ago with "walking pneumonia". On examination, he is coughing
frequently but is not particularly ill-appearing. His temperature is 38.5 oC, pulse 95 beats per
minute, and respiratory rate of 23 breaths per minute. His pharynx is injected. Head and neck
exam is normal. His lung exam is notable only for some scattered rhonchi. The remainder of
his examination is normal. A chest x-ray shows sonle patchy infiltration. A sputum Gram stain
shows white blood cells but no organism.'What is the most likely etiology of this infection?
A. Klebsiella pneumoniae
B. Mycoplasma pneumoniae
C. Streptococcus pneumoniae
D. Legionellapneumophila
16. A 4-year-old child presents with fever, conjunctivitis, photophobia, cervical lymphadenopathy,
and coryza. Red lesions with a white center are present on the buccal mucosa. A generalized,
blanching, erythematous rash is also noted. What is the most likely,diagnosis?
A. Rubella
B. Roseola
C. Rubeola
D. Varicella
17. Ferdinand, Vilma and OJ, first year undergraduate medical students, finds it difficult to
identify malarial histology slides in the laboratory. even though they have a good theoretical
knowledge about them. Their first validated thin blood smear slide on loan shows Maurer's
dot as stippling in erythrocytes containing a larger form of plasmodium parasite. The
slide simply explain about the blood protozoan:
A. Plasmodium vivax C. Plasmodium falciparum
B. Plasmodium ovale D. Plasmodium malariae
18. A 3O-year-old man comes to the ER department because of dysphagia and diplopia that
developed after he consumed a homemade strawberry jam. While in the emergency room,
he developed generalmuscle weakness and requires intubation due to sudden respiratory
muscle paralysis. Which of the following pathogens is most likely responsible for this patient's
condition?
A. Clostridium botulinum C. Clostridium tetani
B. Clostridium perfringens D. Clostiidium difficile
19. A 33-year-old African drug addict male with aortic and mitral valve prosthesis and history of
tooth extraction 2 months ago came with high fever and chest pain and was found to
have infective endocarditis. Culture was positive for Streptococcus spp. on three different
occasions. Transesophageal echocardiogram (fOE) showed two dangling vegetations on
prosthetic mitralvalve. The patient received intensive inpatient treatment with antibiotics and
underwent another successfulprosthetic valve replacement. Question: Which of the following
groups of streptococci is implicated in infective endocarditis?
A. Group B streptococci
B. Group D streptococci
C. Viridans streptococci
D. Al of the above
20. A 48-year-old man presents for the evaluation of a 2-month history of upper abdominal pain
associated with nausea. lt is made worse when he drinks soda, coffee, or alcohol. He has
denies significant use of nonsteroidal anti-inflammatory drugs (NSAlDs). His general physical
examination and vital signs are normal. His abdominal examination is notable for epigastric

Continued on Page 5
PHYSICIAN Licensure Examination Page 5
Saturday, September 9,2017 - O2:OO p.m. - O4:OO p.m.
MICROBIOLOGY AND PARASITOLOGY ; S.E-LA

taken multiple over-the-counter antacid medications that provide temporary retief. He admits
to a 20-pack-year smoking history and drinking one or two alcoholic beverages a week but
tenderness without the presence of masses, rebound tenderness, or guarding. A rectal
examination reveals his stoot to be heme positive. A CBC shows a mild hypochromic'
microcytic anemia. He is referred to a gastroenterologist for an upper Gl endoscopy, which
shows diffuse gastritis and a gastric ulcer. Question: What organism is most likely to be
visualized on histologic evaluation of a gastric biopsy specimen?
A. Escherichia coli
B. Helicobacter pYlori
C. Bacteroides fragilis
D. Salmonella enteritidis
21. A S5-year-old boy scout master developed a case of diarrhea and stomach cramps during
the second week of camping trip in the mountains of Los Baios Laguna. Trophozoites with
pear-shaped features and cysts were observed in his stool specimen. A duodenum string
test was also positive for trophozoites. The stools appears fatty but without any blood or
mucus. Question: Which of the following was the most likely culprit causing the disease?
A. Giardia lamblia
B. Trichomonas hominis
C. Balantidium coli
D. Entamoeba histolytica
22. An 1$-year-old teenager was presented with closed comedones (white heads), open
comedones (blackheads) and multiple red papules and pustules on the face, back and upper
arms. The diagnosis of acne vulgaris based on the history and physical examination was
established. Her attending doctor prescribed her with oral isotretinoin to decrease sebum
production and reduce inflammation of her oily skin. What is the clinicat hallmark of acne
vulgaris?
A. Papules C. Combdones
B. Pustules D. Nodul65
23. AZ2-year-old male, had an outing with his friends and developed fever of 38.5 oC, diarrhea
and vomiting following eating chicken salad 24 hours back. Two of his friends developed the
same symptoms. Most probable diagnosis is:
A. Salmonella enteritis poisoning
B. Bacillus cereus
C. Staphylococcus aureus
D. Vibrio cholera
24. A 41-year-oldman comes to the physician because of a2day history of sore throat. Current
medications include an inhaled corticosteroids for asthma. His temperature is 37 oC. A KOH
preparation of a scraping from tongue showed budding yeast. Which of the following is the
most appropriate pharmacotherapy for this patient?
A. Amphotericin B C. ltraconazole
B. Fluconazole D. Nystatin
25. When you're driving around on the road, you see all sorts of different Vpes of vehicles.
You've got SUVs, minivans, pickups, motorcycles and 18-wheelers. ln the end, they are all
a type of vehicle regardless of their shape, size and color. Likewise, viruses have many
different shapes as well, and some of their shapes are unique to certain viruses and the
things they infect. These shapes aren't just for looks. Like certain types of automobiles,
they actually confer a function. Question: Which of the following viral organism is correctly
matched with its morphological shape?
A. rabies virus - needle shape
B. chikungunya virus - wagon wheel shape
C. small pox virus - brick shape
D. rotavirus - bullet shape
Continued on Page 6
PHYSICIAN Licensure Examination
Saturday, September 9,2017 - O2:00 p.m. -. 04:OO p.m.
MICROBIOLOGY AN D PARASITOLOGY SET A

26. A S2-year-old man presents for the evaluation of diarrhea and abdominal pain, which have
been'worseningoverthepastweek. He is now having 8-9 watery stoolg 3 q?y and mild
crampingpain.-Hedeniesvomiting,fever,ill contacts,orhaving had blood in his stool- He
has no n'lstory- of gastrointestinal diseases. He states that about 1O days ago he completed
a course of am6xicillin/clavulanic acid for pneumonia. On examination, he is mildly ill
appearing, but his vitalsigns are normal. His abdomen is soft, has hyperactive bowelsounds,
ahb is diffusely, mildly tender. A stoolsample is negative for blood but positive for leucocytes.
A stool culture is negative, but Toxin A and B assay is positive. Which of the following
organisms is the moSt likely cause of this man's enterocolitis?
A. Salmonella typhi C. Clostridium difficile
B. Helicobacter pylori D. Vibrio cholera
27. A2-year-old boy who plays in a sandbox in a nursery that also has a family of cats for children
to play with preients with wheezing, hepatosplenomegaly, and prominent peripheral blood
eosinophilia. Wnicn of the following diagnoses is most likely in this child?
A. Ascariasis
B. Pinworn infestation
C. Visceral larva migrans
D. Loeffler's syndrome
28. A4-year-oldboyisbroughtto the pediatrician because of abdomindl pain, vomiting, and
mucus and blood. The child has a fever of 39.4 oC. On stool culture, the
diarihea containing
causative organism is shown to be a non-lactose-fermenting and non-hydrogen-sulfide
producing bacterium. Which of the following is most likely responsible for the child's illness?
A. Escherichia coli
B. Vibrio cholera
C. Salmonella spp
D. Shigella specie
29. A 68-year-old woman was admitted to the emergency room after eating sushi and sashimi
at a fancy Japanese restaurant. The patiertt showed symptoms of vomiting and epigastric
pain. When admitted to the hospitalflve hours after eating the contaminated raw seafood,
she complained of severe epigastric pain, chest pain and vomiting, and then immediately
fell into a state of mental stupor. A total blood count, chemical examination of blood and
urinalysis, and ECG were normal. A gastro endoscopic examination was done showing four
thread-like worms penetrating the gastric mucosa in the great curvature of the middle body
and fundus. Another one long-white nematode was found at the great curvature side of
upper body during an endoscopic examination performed on the following day. The gastric
mucosa of the body and antrum showed diffuse hyperemia. No more worms could be found
during a third endoscopic examination carried out on two days after the initial examination.
The state of patient improved immediately after removing the worms and she was
discharged after receiving supportive care. Which of the following organism is most likely
responsible for this woman's condition?
A. Pfiesteria
B. Anisakis
C. Vibrio vulnificus
D. Vibrio parahaemolyticus
30. A 2O-year-old man presents to the physician with a non-tender indurated mass over his
mandible. He has had this mass for 4 months and decided to come to the physician because
the mass started to ooze a thick yellow exudate. Yellow granules are seen on microscopic
examination of the discharge. Which of the following organism is most likely responsible for
this man's lesion?
A. Actinomyces israelii
B. Nocardia asteroides
C. Staphylococcus aureus
D. Trichinella spiralis

Continued on Page 7
PHYSICIAN Licensure Examination
Saturday, September 9, 2017 - O2:OO p.m. - O4:00 p.m.
MICROBIOLOGY AND PARASITOLOGY SET A

31. A 2-year-old child presents to the physician's office with her mother. The mother states that
for the past few days the child has had a low-grade fever, upper respiratory tract symptoms,
and has been tugging at her right ear. An examination of the right ear with the pneumatic
otoscope reveals a hyperemic, opaque, bulging, tympanic membrane with poor mobitity.
The physician makes the diagnosis of right otitis media. Which of the following organisms is
the most likely causative pathogen?
A. Group B Streptococci
B. Staphylococcus pyogenes
C. Streptococcus pneumoniae
D. Haemophilus influenzae
32. Diffuse erythematous macules that may appear confluent is called:
A. Morbilliform
B. Scarlatiniform
C. Koebner phenomenon
D. Satellite lesion
33. A7-year-old girl s brought to the pediatrician because of fever of 37.9 oC, trouble swallowing,
and drooling. Within a few minutes of arriving at the office, she develops inspiratory stridor
and respiratory distress. An x-ray of her neck from the side showed a'thumbprint sign"
presentation. Which of the following is most likely responsible for the patient's condition?
A. Haemophilus influenzae
B. Mycoplasma pneumoniae
C. Parainfluenza virus
D. Respiratory syncytial virus
34. A 13 - year- old child is brought to the pediatrician because of a purulent yellow
discharge oozing from his eyes. On physical examination, both eyelids are swollen,
and the conjunctivae appear inflamed. Under microscopic view, scraping from the
conjunctival surface show epithelial cells with basophilic inclusions in the cytoplasm,
called Halberstaedter-Prowazek bodies. Which of the following organism is most likely
responsible for this patient's condition?
A. Adenovirus
B. Chlamydia trachomatis
C. Neisseria gonorrhea
D. Treponema pallidum
35. Disorder caused by roundworm infestation of Necator americanus:
A. can cause malabsorption enteropathy to man
B. manifested by nocturnal anal pruritus
C. associated with typicalcreeping eruption
D. produces aut6infection to man
36. A 3O-year-old man presents with an erythematous patch on his arm. The patch is warm,
swollen, and tender. Margins are not very distinct. Vital signs are normal. What diagnosis
should you suspect?
A. erysipelas
B. cellulitis
C. folliculitis
D. furunculosis
37. The virulent strain E. coli 0157: H7 which causes Hemolytic Uremic Syndrome in cases of
bacterial gastroenteritis is closely related to Shigella dysenteriae type 1. Both are capable of
producing a toxin celled:
A. Exotoxin
B. Staphylotoxin
C. Verotoxin
D. Enterotoxin

Continued on Page 8
PHYSICIAN Licensure Examination
Saturday, September 9,2017 - O2:0O p.m. - O4:00 p.m.
MICROBIOLOGY AND PARASITOLOGY SET A

38. A 2S-year-old man came to the ER department because of fever, jaundice, severe myalgia
and conjunctival suffusion. 7 days ago, he participated in the esteros clean-up drive spear-
headed by the barangay council. The canals are filthy, stagnant and have become breeding
grounds for rats and diseases. What does the patient have?
A. Typhoid fever
B. Malaria
C. Cholera
D. Weil's syndrome
39. A 4o-year-old abaca farmer from Northern Samar was brought to the regional health hospital
because of gross swelling on his right leg for more than 4 years. He was walking by his own
without support and swelling was not reversible at night. On examination, the whole leg was
edematous, bluish in color, deep skin fold at junction of middle and lower one third, non-
discharging sinus above the fold anteriorly, absence of nodules or warts and lrairs. Non-
pitting in nature. On admission, x-ray revealed calcified filariae. He was labelled as a case of
lymphatic filariasis. Question: How is elephantiasis transmitted to human?
A. penetration of the human skin host by filariform larvae
B. infected by drinking water containing copepod's rhabditiform larvae
C. propagated primarily through the bite of an infected mosquito
D. ingestion of infective egg from fecal contaminated vegetables
40. Life Cycle of Hoolorvorms:
A. Adult * Egg - Miracidiutn + First lntermediate Host * Sporocyst * Redia
-
lntermediate Host --+ Definitive Host
Cercaria
- Second *
B. Egg -'Soil----+ Larva Hatches --+ Rhabditiform
- Filariform Skin + Circulation
----+

Heart & Lung --' EsophaguS -+ Small lntestine


C. Small lntestine .-* Egg Laid in Typical Female---+ Embryonation
in Water 3-5 Days
-
lngested by Freshwater Fish + Man lngested Raw Fish Larva
*
D. Large intestine Female Migrates to Perianal Region Deposited Embryonated Egg
-
lngested or Inhales -' Larva Hatch in Duodenum -' Migrate to Large lntestine
41. Which injury does not predispose a person to gas gangrene?
A. Surgical incision
B. Puncture wounds
C. Diabetic ulcers
D. Dislocated shoulder
42. The characteristic itch of scabies is:
A. intermittent
B. continuous
C. diurnal
D. nocturnal
43. The spirochetes that is tightly coiled, thin, and has a hook at one end that looks like a question
mark:
A. Leptospira interrogans
B. Borrelia recurrentis
C. Treponema pallidum .

D. Borrelia burgdorferi
44. The localization and distribution of the skin findings seen in herpes zoster are distinctive,
unilateral, does not cross the midline, and localized to a single dermatome of a single
sensory ganglion. The most common sites are the thoracic nerve and:
A. ophthalmic nerve (Vl )
B. maxillary nerve (V2)
C. mandibular nerve (V3)
D. glossopharyngeal nerve (lX)

Continued on Page 9
PHYSICIAN Licensure Examination Fage 9
Saturday, September 9,2017 - O2:OO p.m. -- 04:OO p.m.
MICROBIOLOGY AN D PARASITOLOGY SET A

45. lnAsia,Wuchereriabancroftiarewidely distributed over much of the Southeastern region.


Among specific population settings, there exist a "rural" and "urban" strain transmitted by
mosquitovectors. Giuestion: The"-ruial" form found in thejungle of Palawan is primarily
transmitted bY:
A. Aedes (FinlaYa) Poicilius
B. Anopheles minimus flavirostris
C. Culex quinquefasciatus
D. Ochlerotatus (FinlaYa) niveus
46. Staphylococcal enterotoxicosis is assocrated with eating all but which of the following foods?
A. Custard
B. Ham and processed meat
C. Hamburger
D. Chicken salad
47. 'fhefollowing are considered hermaphroditic or monoecious trematodes (flukes), except
A. Fasciola hePatica
B. Clonorchis sinensis
C. Opisthorchis viverrini
D. SchistosomajaPonicum
48. Penicillin is the drug of choice for infections caused by all of the following, except
A. Treponema pallidum
B. Bacteroides fragilis
C. Streptococcus Pyogenes
D. Clostridium perfringens
49. Find the mismatched pair among the following arthropods:
A. Fleas Pthirus pubis
B. Ticks Rickettsia ricketttsii
C. Lice Pediculus humanus
D. Mites Sarcoptes scabiei
50. Salmonella infections are characterized by all of the following, except
A. Typhoid fever
B. Gastroenteritis
C. Bacillary dysentery
D. Enteric fever
51. How do you prevent Diphyllobotrium latum infection?
A. thorough cooking of all suspected freshwater fish
B. control of mosquitoes by destruction r:r removal of water plants
C. abstaining from eating raw, freshly salted or inadequately cooked crabs or crayfish
D. periodic deworming of infected dogs and cats
52. Ulcer of the ileum over enlarge Peyer's patches are a characteristic finding in:
A. Tuberculosis of the intestine
B. Shigella dysentery
C. Typhoid fever
D. Salmonella gastroenteritis
53. A 4-month-old infant is brought to the pediatrician by his mother because he has not been
feeding well. Physical examination of the mouth reveals curd-like plaques on the tongue and
buccal mucosa that do not scrape off easily. Another associated physical finding might be
seen in which of the following?
A. ears C. scalp
B. nose D. perineum

Continued on Page 10
PHYSICIAN Licensure Examination
Saturday, September 5,2017 - 02:00 p.m. - 04:00 p.m.
M ICROBIOLOGY AND PARASITOLOGY SET A

54. A 19-year-old sexually active woman presents with a desquamating rash involving her palms
and soles, diarrlrea, fever, hypotension, headache, myalgias, and red eyes. Her last menstrual
period was 4 days ago. Which of the following is the most likely diagnosis?
A. Rocky Mountain SPotted Fever
B. Toxic Shock Syndrome
C. Scarlet Fever
D. Kawasaki's Disease
55. Early detection of parasitic ova using microscopy plays a crucial role in the treatment and
prevention strategies.Which of the following morphological combination is a mismatched?
A. Whip worm FootballshaPe ova
B. Thread worm - Chinese lantern shape egg
C. Pin worm D shaPe ova
D. Hook worm Peanut shaPe ova

56. Bacteria are cleared from the circulation nlainly by:


A. lung capillaries
B. spleen and liver macroPhages
C. circulating neutroPhils
D. renal capillaries
57. Dumdum fever is a potentially fatat parasitic disease of the visceral organs due to infection
cause by the protozoan:
A. Leishmania donovani
B. Leishmania tropica
C. Trypanosoma cruzi
D. Trypanosoma gambiense
58. A young woman with secondary lesions gave birth to a child with congenital syphilis. All of
the following are stigmata of congenital.syphilis, except
A. Chancre
B Clutton's joints
C. Hutchinson's teeth
D. Higoumenaki's sign
59. Proven to be the most sensitive diagnostic test for typhoid fever:
A. stool culture
B. culture of intestinal secretions
C. blood culture
D. bone marrow culture
60. Which of the following is not a characteristic of the agents of spongiform encephalopathies?
A. highly resistant
B. are naked fragments of RNA
C. associated with tangled protein fibers in the brain
D. are chronic
61. Which of the following diseases is not likely to have acquired through a mucus-membrane
associated port of entry?
A, Tetanus C. Gonorrhea
B. Tuberculosis D. Cholera
62. Ot the following bacteria, the one which is LEAST likely to be the cause of abscess formation:
A. Escherichia coli
B. Proteus vulgaris
C. Pseudomonas aeruginosa
D. Staphylococcus aureus

Continued on Page 11
PHYSICIAN Licensure Examination Page 11
Saturday, September 9,2017 - O2:OO p.m. - O4:OO p.m.
MICROBIOLOGY AND PARASITOLOGY SET A

63. ln making the diagnosis of infective endocarditis, which of the following is most important?
A. complete blood count (CBC)
B. microscopic analysis
C. blood cultures
D. efihrocytes sedimentation rate (ESR)
64. The virus that has been implicated in the pathogenesis of nasopharyngeal carcinoma:
A. Human Papilloma Virus C. Cytomegalo Virus
B. Epstein-Barr Virus D. Human T Cell Leukenria Virus

65. The beneficial in vivo effect of Hyperbaric Oxygen Therapy in the treatment of gas gangrene
is that:
A. 02 is bacteriocidal for Clostridial organism at tensions of 250 mm Hg
B. 02 is bacteriocidalfor Clostridial organism at tensions at above 14OO mm Hg
C. 02 stops alpha toxin production by Clostridial organism at tensions of 250 mm Hg
D. 02 stops alpha toxin production by Clostridial organism at tensions of 1400 mm Hg
66. What is the mechanism of action of paralytic shellfish poisoning?
A, The saxitoxin acts as a selective sodium channel blocker preventing impulse-generation
in peripheral nerves and skeletal muscle leading to paralysis of the affected region.
B. The spasmogenic toxin binds to the presynaptic membrane of the neuromuscular junction
and is internalized and transported retroaxonally to the spinal cord.
C. The tetrodotoxin inhibits the firing of action potentials in neurons by binding to the
voltage-gated sodium channels in nerve cell membranes and blocking the passage of
sodium ions into the neurons.
D. The verotoxin acts on the lining of the blood vessels, the vascular endothelium. The
breakdown of the lining can lead to hemorrhage and eventual bloody diarrhea.

67. A 9-year-old boy had his semiannual dental cleaning. The dentist found some tooth decay.
Which bacteria are frequently associated with dental caries?
A. Streptococcus pneumoniie
B. Streptococcus pyogenes GrouP A
C. Streptococcus mutans
D. Peptostreptococcus
68. Virulence of a microorganism can be best described as:
A. the ability to penetrate into the host tissue
B. the ability to colonize the host
C. the ability to produce a pathological symptoms
D. the ability to utilize the machinery of the host
69. Which of the following clinical specimens should be collected to confirm the diagnosis of
Y aricella-Zoster i n fecti on ?
A. saliva c. vesicle fluid
B. blood D. cerebrospinal fluid

70. All of the following are modified penicillin resistant to destruction by staphylococcal beta-
lactamase, except
A. oxacillin C. cloxacillin
B. ampicillin D. methicillin
71. Malignant neoplasm which occur in increased incidence in patients with AIDS include all of ,

the following, except


A. Burkitt's lymphoma
B. Hairy cell leukemia
C. Kaposi's sarcoma
D. Primary CNS lymphoma

Continued on Page 12
PHYSICIAN Licensure Examination Page 12
Saturday, September 9,2017 - O2:O0 p.m. '- 04:O0 p.m.
M ICROBIOLOGY AN D PARASITOLOGY SET A

72. fhe Tsetse fly is a transmission factor for which of the following protozoan diseases?
A. Loa loa
B. Leishmaniasis
C. Trypanosomiasis
D. Chagas
73. Historically, there are six "classical" infectious childhood exanthem that have been
recognized. Four of which are viral. The numbers were provided in the year 19O5. Which of
of the following numerical diseases and viruses are matched up incorrectly?
A. First Disease Measles
B. Second Disease - Scarlet Fever
C. Fourth Disease Staph Scalded Skin Syndrome
D. Fifth Disease Roseola lnfantum

74. The cause of antigenic drift of influenza viruses is:


A. mixing of double-stranded DNA genome
B. re-assortment of genome segments during mixed infections
C. accumulated point mutations in the hemagglutinin gene
D. phenotypic masking
75. A S-year-old boy was bitten by wasps. Later, he developed wheals in the skin. Which blood
cell leucocyte is responsible for the wheal forination?
A. neutrophils C. basophils
B. eosinophils D. lymphocytes
76. All of the following statements about Trichuris trichura are true, except
A. commonly known as whipworm
B. diagnosis is best achieved by finding the adult parasite in the feces
C. no animal reservoir exist
D. no serological test are available

77. PrimaryinfectionscausedbyStreptococcus pyogenes are due to the organisms invading


what two body sites?
A. nose and vagina
B. eyes and gastrointestinal tract
C. mouth and urethra
D. skin and throat
78. Which of the following fungal organism demonstrate a very distinctive thick walled
barrel-shape arthroconidia?
A. Blastomyces dermatitidis
B. Histoplasma capsulatum
C. Aspergillus fumigatus
D. Coccidioides immitis
79. Granulomatous lesion is an inflammatory reaction found in many diseases which are form
when the immune system attempts to wall off substances it perceives as foreign but is unable
to eliminate. lt may be seen in all of the following conditions, except
A. Leprosy C. Amoebiasis
B. Schistosomiasis D. Cryptococcocis
80. Laryngeal papillomatosis is commonly associated to this human papilloma virus serotype:
A. HPV 1 and 4
B. HPV6 and 11
C, HPV 16 and 18
D. HPV 8 and 10

Continued on Page 13
PHYSICIAN Licensure Examination Page 13
Saturday, September 9,2017 - O2:OO p.m. - O4:OO p.m.
MICROBIOLOGY AND PARASITOLOGY SET A

81 . The following are inclusion bodies with the disease that it causes. Which of the following is
a mismatched pair?
A. Babes-Ernst Bodies DiPhtheria
B. Henderson-Paterson Bodies Herpes Simplex
C. Levinthal-Colle-Lillie Bodies Psittacosis
D. Guarnieri Bodies Small Pox

82. Which of the following protozoa is known only in the trophozoite stage?
A. Entamoeba histolytica i

B. Giardia lamblia
C. Balantidium coli
D. Trichomonasvaginalis
83. A child after consuming food in a party complains of vomiting and diarrhea within 1-5 hours.
Most probably he h6s got the infection due to:
A. Staphylococcus aureus
B. Streptococcus
C. Clostridium perfringens
D. Clostridium botulinum
84. What is the most prevalent antibody circulating throughout the tissue fluids and blood?
A. lgG
B. lsA
C. lgM
D. lgE
85. Chemical agent that is applied directly to body surfaces, wounds, and surgical incisions to
inhibit vegetative pathogens
f :,#];::
B. fungicides
C. antiseptics
D. disinfectants

86. What do you call the phase of bacterial groMh where cells are most sensitive to chemical
agents?
A. lag phase
B. log phase
C. stationary phase
D. death phase
87. What is the key test that separates Staphylococcus aureus from other staphylococci?
A. Catalase test
B. Coagulase test
C. Urease test
D. Susceptibility to novobiocin
88. All of the following statements about cestodes are true, except
A. adult tapeworms are composed of se'gments called proglottids
B. adult tapeworms have no digestive tract
C. each mature proglottid contains both male and female reproductive organs
D. there are no intermediate hosts in their life cycle
89. The usual culture medium for Haemophilus influenzae is:
A. blood agar
B. chocolate agar
C. EMB agar
D. brain-heart infusion agar

Continued on Page 14
PHYSICIAN Licensure Examination Page 14
Saturday, September 9,2017 - 02:OO p.m. - 04:OO p.m.
MICROBIOLOGY AND PARASITOLOGY SET A

90. The Escherichia coli strain identified to be the major cause of traveler's diarrhea:
A. EAEC (Enteroaggregative E. coli)
B. EHEC (Enterohemorrhagic E. coli)
C. EIEC (Enteroinvasive E. coli)
D. ETEC (Enterotoxigenic E. coli)
91. Fever is an effect of:
A. cytolysin
B. endotoxin
C. exotoxin
D. capsule
92, The most frequent clinical manifestation of Listeria morrocytogenes is:
A. endocarditis
B. abortion
C. meningitis
D. neonatal septicemia
93, All of the following viruses contain an envelope, except
A. bunyavirus
B. coronavirus
C. rotavirus
D. togavirus
94. Congenital infections often characterized by hepatosplenomegaly, jaundice, microcephaly
and respiratory distress:
A. Herpes Simplex virus
B. Varicella-Zoster virus
C. Cytomegalovirus
D. Epstein-Barr virus
95. Which of the following statements regarding Creutzfeldt-Jakob disease is true?
A. Creutzfeldt-Jakob disease is primarily a disease of young adults.
B. Creutzfeldt-Jakob disease is called the animal form of spongiform encephalopathy.
C. Creutzfeldt-Jakob disease is readily inactivated by proteases and nucleases.
D. Creutzfeldt-Jakob disease is characterizedby dementia with subsequent coma and
death.

96. This dimorphic fungi grows well in soils with a high nitrogen content and can be isolated from
bird roosts and bat caves:
A. Coccidioides immitis
B. Histoplasma capsulatum
C. Blastomyces dermatitidis
D. Paracoccidioides brasiliensis
97. Botulin toxin prevents the release of what chemical that initiates the signal for muscle
contraction?
A. serotonin
B. acetylcholine
C. dopamine
D. tyramine
98. Host that harbors the parasite only for a short transition period and does not reach sexual
maturity is called:
A. reservoir host
B. definitive host
C. amplifying host
D. intermediate host

Continued on Page 15
PHYSICIAN Licensure Examination Page 15
Saturday, September 9,2017 - O2:OO p.m. - O4:O0 p.m.
M ICROBIOLOGY AN D PARASITOLOGY SET A

99. Avian influenza strains are divided into two types based on their pathogenicity, namely
low pathogenic avian influenza (LPAI ) and high pathogenic avian influenza (HPAI). LPAI
viruses cause only mild disease in poultry while HPAI virus can kill up to 90-1O0% of the
flock and cause epidemics that may spread rapidly, devastate the poultry industry and result
in severe trade restrictions. Question: What emerging HPAI subtype that is causing global
concern as a pandemic threat and is often referred to simply as "bird flu" or'avian influenza"?
A. HlNl virus
B. H5N1 virus
C. HlN2 virus
D. H3N2 virus
100. Japanese Encephalitis virus is biologically related to the virus causing:
A. Marburg virus
B. California Encephalitis virus
C. St. Louis Encephalitis virus
D. Western Equine Encephalitis virus
*** END ***

WARNING: Failure to submityour TestQuestions(Complete)set will c€ruse the cancellation


of your Test-Result for this subject.
MICROBIOLOGY & PARASITOLOGY 2017
ANSWER KEY

1. D 26. C 51. A 76. B


2.8 27. C 52. C 77. D
3.C 28. D 53. D 78_ D
4.4 29_ B 54. B 79. C
5.B 30. A 55. D 80. B
6.C 31. C 56. B 81. B
7.D 32. A 57. A 82. D
8.A 33. A 58. A 83. A
9.E} 34. B 59. D 84. A
10. A 35. C 60. B 85. C
11. D 36. C 61. A 86. B
12. D 37. C 62. C 87. B
13. B 38. D 63. C 88. D
14. C 39. C 64. B 89. B
15. B 40. B 65. C 90. D
16. C 41. D 66. A 91. B
17. C 42. D 67. C 92. C
18. A 43. A 68. C 93. C
19. D 44. A 69. C 94. C
20. B 45. B 70. B 95. D
21. A 46. C 71. B 96. B
22. C 47. D 72. C 97. B
23. A 48. B 73. D 98. D
24. D 49. A 74. C 99. B
25. C 50. c 75. B 100. c
Seat No.
Republic of the PhiliPPines
PROFESSIONAL REGULATION COMMISSION
Manila
BOARD OF MEDICINE

PHYStCIAN Licensure Examination


Sunday, September 10, 2017
PHYSIOLoGY sET B

INSTRUCTION: Select the correct answer for each of the following questions.
t tart Sdy o I for each item by marking the box corresponding to the letter of your choice
on the aniwei sheet provided. STRICTLY NO ERASURES ALLOWED. Use pencil no. 2 only.
MULTIPLE CHOICE:

1. You are alerted that a 3o-year-old, qncanssious male is being-atbrought by ambulance to the
Emergency Department where yoffii auty. While home, the patient suddenly
developed heegleghe, dizziness, weapess, nausea, vomi,ting and-diarrhga. En route to the
Emergency -oGl6rtmeiltr'e lof-consciGfr6ss and experienced- @ry and -feroal
incontinence. When the patient arrives at the Emergency Department, you note that he has
egeils, generali€g.Pgrafygis,f@lEns, and i.s ull9gPgnsive. to deep
pain.
@iogt
firneat ;nO gag reflexes areGnt. He haEaro-ltiFe salivation, diaphoresis, and excess
lacrimation. Vital signs include the following: blood pressure 14O/9A mm Hg, temperature
gg.2 oF (37.2 "C), pulse 58 beats /minute and regular. Rales are noted during chest
auscultations. Heart examination is unremarkable except for an 54 gallop. Abdominal
examination reveals no detectable masses, organomegaly, or hyperactive bowel sounds.
Mucoid secretions are suctioned from the trachea at the time of intubation, and mechanical
respiratory support is instituted. You initiate treatment with Narcan for possible opiate
ingestion, with no effect. From the patient's brother, you learn that the patient returned from a
S-day vacation in the province. This morning he changed into work clothes and began mixing
pesticides for subsequent tree spraying in their family orchard. About noon, the patient became
nauseated and started sweating profusely. The brother cannot recall any unusual events
before his brother's illness and assures you that they had performed the tree-spraying
operations many times in the past with no ill effects. After treatment and antidotal therapy, the
patient improves remarkably - he has spontaneous respirations and regains consciousness
within 4 hours. Mechanical ventilation is discontinued, and patient is well enough to be
discharged 3 days after admission. What is the mechanism of action of the.absvq case?
A. They are involved in postsynaptic hyperpolarization (increasing K+ efflux) or reducin-g post-
synaptic Ca++ influx inhibiting neuronal activity
g. tiUinOstothepresynapticmembraneof the neuromuscular junction and is internalized
and transported retroaxonally to the spinal cord.
C. They work by inhibiting acetylcholinesterase (AChE) resulting in an overabundance of
acetylcholine (ACh) at synapses and the myoneuraljunction.
D. lt reduced activity of dopamine-secreting cells caused by cell death in the pars compacta
region of the substantia nigra.

2. Ourbodies are wonderfully designed to rise to daunting challenges - escaping from dangerous
situations; performing feats of strength tl'rat seem impossible such as carrying an injured
person from a burning building; having all senses on red alert when peak performance is
required, such as in an athletic competition or dealing with complex, high-stakes situations.
The body, however, is not particularly good at distinguishing between true, life-threatening
emergencies and the anxieties, worriei, stresses and challenges of modern life. Whether the
emergency is real or perceived, the body responds as if it were a life or death situation.
Question: What physiologic response when a person risk his own life in the event of fire by
smashing windows and drag another person out of a burning building?
A. Day-To-Day Response
B. Fight-Or-Flight Response
C. Systemic-lnflammatory-Response
D. Acute-Stress-Response

Continued on Page 2
PHYSICIAN Licensure Examination Page 2
Sunday, September 10,2017 - 08:00 a.m. -- 10:00 a.m.
PHYSIOLOGY SET B

J. This is a case of a24-year-old Chinese looking young man who was brought by his uncle to
the rehabilitation center because of drug abuse. He looks like a man of 4O years- Firstly, his
friends introduced him to drugs as a means of enjoyment. Gradually he became AddicteO- Ue
started with marijuana. He sometimes changes his drugs to meet his satisfaction level. He
changes drugs one after another from nrarijuana to methamphetamine. Now, he is fully
addicted to methamphetamine for 4 years and has to take it four times in a day. Without having
it he can't do anything. He spends Php3,OOO - PhpS,OO0 everyday for drug purposes. For the
excess money, sometimes he takes loan from friends or steals his own household materials.
He collects drugs from local spots or a particular person. On physical examination, he was
egpls looking and iq!-ta!pd. Speech vvss elqwed. Physique was lean and emaciated. Mild
anemia and jaundice wE present. No abnormality was detected in heart, lungs and kidney.
He feels egg@ria, snxiefy, increased libidg" impaired concentration, ee€f,SFll-loss, Ioss of
self- confidenc€, sociability, irriJahiljty and ag€ression. MEtmilfi'phetamine is a white, odorless
crystalline powder that dissolves easily in liquids. lt is a stimulant to the nervous system which
cause the user to have an increase in alertness, concentration and energy. After ingestion of
Meth, users experience an intense sense of euphoria which is caused by the increased level
of dopamine, together with other highly desirable effects including increased productivity, a
heightened sense of attention/awareness, hypersexuality, increased energy and decreased
anxiety. lt increase the production of dopamine in the brain while suppressing the levels of
adrenaline. The result is a highly euphoric feeling while the drug is active, but this is faced
with high tension and irritability when the effeCts wear off due to this chernical imbalance.
Question: Methamphetamine is a psychostimulant drugs which trigger the cationic lipophilic
molecule that has dramatic effect on:
A. sympathetic pathway
B. parasympathetic pathway
C. iympitnbtic and parasympathetic pathways
D. sympathetic, parasympathetic and somatic pathways
4. A middle-aged man with a medical history of hypertension, diabetes, dyslipidemia, smoking
and family history of premature CAD presents with retrosternal crushing chest pain (10/10 in
intensity), radiating down the left arm and left side of the neck. He feels nauseated and light-
headed and is short of breath. Examination reveals a hypotensive, diaphoretic man in
considerable discomfort with diffuse bila,tdral rales on chest auscultation. ECG was done.
Question:WhatistheinitialECGchangesfoundin$EUl?
A. ST segment depression
B. ST segment elevation
C. Prolonged (deep, broad) Q wave
D. lnverted T wave

A69-year-oldmanpresentswithal-yearhistoryof mildslowness and loss of dexterity. His


handwriting has become smaller, and his wife feels his face is less expressive and his voice
softer. Over the last few months he has developed a subtle tremor in the right hand, noted
while watching television. His symptoms developed insidiously but have mildly progressed. He
has no other medical history, but he has noted some mild depression and constipation over
the last 2 years. His examination demonstrates hypophonia, masked facies, decreased blink
rate, micrographia, and mild right-sided bradykinesia and rigidity. An intermittent right upper
extremity resting tremor is noted while he is walking. The rest of his examination and brain
MRI are normal. Question: lt can be verified that the above case satisfies this definition:
A. A neuromuscular disease characterized by considerable weakness and abnormal fatigue
of the voluntary muscles cause by insufficient secretion of acetylcholine, excessive
secretion of cholinesterase and unresponsive of the muscle fibers to acetylcholine.
B. A chronic, progressive, non-contagious, degenerative disease of the CNS characterized
by demyelinization of the neurons.
C. A progressive degenerative disease involving the motor system related to an exeess of
glutamate, a chemical responsible for relaying messages between the motor neurons.
D. A degenerative disease caused by the depletion of dopamine, which interferes with the
inhibition of excitatory impulses, resulting in a dysfunction of the extrapyramidal system.

Continued on Page 3
PHYSICtAN Licensure Examination rPage
Sunday, September 10,2017 - O8:OO a.m. - 1O:OO a.m.
PHYSIOLOGY SET B

A S4-year-old widower with a history of chronic obstructive pulmonary disease was rushed to
the emergency department with increasing shortness of breath, pyrexia, and a productive
cough with yellow-green sputum. He has difficulty in communicating because of his inability to
complete a sentence. One of his sons says he has been unwell for three days. Upon
examination, crackles and wheezes can be heard in the lower lobes; he has a tachycardia and
a bounding pulse. Measurement of arterial blood gas shows pH = 7.3, PaCO2 = 68 mm Hg,
HCO3 = 28 mmol/L, and PaO2 = 6O mm Hg. How would you interpret this?
A. Respiratory Acidosis, Uncompensated
B. Respiratory Acidosis, Partially Compensated O^t
C. Metabolic Alkalosis, Uncompensated
D. Metabolic Acidosis, Partially Compensated
7. A QZ:Vear-old man was referred to the hypertension clinic because of refractory hypertension
associated with gout. The onset of hypertension occurred in his 20s. Since his early 30s, he
also had recurrent po5lrgra, and at that tine was taken off thiazide, but nonetheless continued
to experience recuirfri Sorty episodes -"rbry several months. Apprbximately 1 year prior to
his hypertension evaluation, the patierit,was prescribed allopurinol and last experienced
podagra 6 months previously. Recent blood pressures (BPs) has been 150/80 mm Hg and
175185 mm Hg and was 160/85 mm Hg when seen in the hypertension clinic. Creatinine was
1.1 mg/dl (normal O.7 - 1.3 mg/dl) and uric acid was 4.6 mg/dL (normal 3.4-7.2 mg/dl).
Medications included atenolol 50 mg twice a day, allopurinol 200 mg twice a day, amlodipine
10 mg once a day, clonidine 0.1 mg twice a day, losartan 5O mg once a day, and terazosin 6
mg at bedtime. The patient was hesitant to retry a diuretic, but was persuaded to take
hydrochlorothiazide 12.5 mg daily, subsequently advanced to 25 mg daily. Follow-up BP were
125 to 130 mm Hg/ 65 to 70 mm Hg without recurrence of gout 6 months later. Question: What
is the diuretic mechanism of action of hydrochlorothiazide?
A. These diuretics act on the distal convoluted tubule and inhibit the sodium - chloride
symporter leading to a retention of water in the urine, as water normally follows
penetrating solutes.
B. ihese are Eiuretics which do not promote the secretion of potassium into the urine thus
potassium is retained and not lost as much as with other diuretics., , !

C. This diurretics reduces medullary osmolality and thus impairs the concentration of urine in
the loop of Henle which usually uses the high osmotic and solute gradient to transport
solutes and water.
D. These are diuretics that inhibit the enzyme carbonic anhydrase which is found in
the proximal convoluted tubule, this results in several effects including bicarbonate
accumulation in the urine and decreased sodium absorption.
A 63 - year - old woman underwent a nephrectomy on the right side for renal cancer.
Postoperatively she developed abdominal and lower back pain, which was treated with an
injection of analgesics in an epidural catheter. The following morning it was discovered that
the patient had cold legs with pallor and no palpahle femoral pulse. Rigor mortis and livor
mortis were diagnosed in both tegs, even though the patient was still alive and awake. Doppler
ultrasound examination revealed the absdnce of blood flow in the lower part of the abdominal
aorta and distally. A cross disciplinriry conference including specialists in urology,
orthopedics, vascular surgery, anesthesiology, internal medicine, and intensive care
concluded that no lifesaving treatment was possible, and the patient died the following day.
A forensic autopsy revealed severe atherosclerosis with thrombosis and dissection of the
abdominal aorta. Question: What is the chief cause of rigot'mortis?
A. decline in ATP production
B. decrease in muscle temperature
C. decrease in muscle calcium
D. death of neuromuscular junption
9. A 21-year-old white woman presents to a family practice clinic seeking a levonorgestrel
intrauterine device (lUD) for contraception. She has only used condoms in the past, but she
recently became engaged and wants to avoid pregnancy for several more years while she
completes her doctoral degree. Her medical history is noncontributory; she has no known
allergies, is a non-smoker, uses no illicit drugs, and has minimal alcohol intake. She takes a
Continued on Page 4
PHYSICIAN Licensure Examination Page 4
Sunday, September 10,2017 - O8:OO a.m. 1O:0O a.m. '-age"4
-
PHYSIOLOGY sET B

daily multivitamin. Further history reveals tl'iat since puberty she has experienced increased
growth of facial hair. lt has caused considerable embarrassment for her, and she is reluctant
to discuss this issue with her physician. ,She also has mild acne, but has not tried any
prescription therapies. She has heard from friends that contraceptive pills cause increased
growth of facial hair, and for this reason she has never used any birth control medication. She
has regular monthly menses. Q,uestion: The hir.sutism seen in the patient is caused by high
levels of:
A. testosterone
B. estradiol
C. aldosterone
D. cortisol
10. A 58-year-old woman was admitted to the hospital because of increasing edema. She had
been well until 4 months previously, when there was the onset of progressive edema of the
feet and hands, puffiness under the eyes and swelling of the abdomen. She noticed frothing
of the urine, but no dysuria, oliguria or discoloration of the urine. 1 month before admission
she entered another hospital. lne Otood pressure was 125 systolic, 8O diastolic. Ttte urine
gave a ++++ test for protein; the sediment contained 2O to 30 white cells and many hyaline
and granular cast. Question: What is the most important means by which substances are
transferred between plasma and interstitial fluid?
A. diffusion
B. osmosis
C. active transPort
D, exchange transPort
11. Altitude sickness can affect people who climb or travel (ascend) to more than 25OO meters
(8,OOO feet) altitude, particularly if they ascend too quickly. For most people, it causes mild
symptoms that improve with rest and time spent getting used to altitude. However, in some
people, it can lead to serious symptoms which can become life-threatening. This is
particularly the case if the signs are not r:ecognized and the person does not move down
(descend) to a lower altitude. The most important treatment if you develop symptoms of
altitude sickness is to stop your ascent and fest. lf your symptoms are severe, do not improve;
or they are getting worse, you need to descend to a lower altitude. There are various
preventatirJmeaJures, the most important being slow ascent so that your body can adjust
to conditions at the right pace (acclimatize). Question: The primary cause for the shift of
oxyhemoglobin dissoliation curve to the right in high altitude pulmonary edemd:
A. effects of low temperature
B. effects of decreasing level of carlcon dioxide
C. effects of methemoglobinemia
D. effects of 2, 3-biphosphoglycerate level
12. A21-year-oldwhite woman presents with an 8-year history of pancreatitis. She had 2 mild
and self-limited episodes of wellcharacterized acute pancreatitis at age 17. She admitted
to drinking alcohol most weekends, sometimes "bingeing," since age 16. Her clinicians
diagnosed alcoholic panbreatitis and advised abstinence. The woman was asymptomatic for
2 years. At age 20 and a senior in college, she had had 3 more discrete episodes of clinical
acute pancreatitis. After the second episode, an empiric cholecystectomy was performed for
possible microlithiasis, but her episodes persisted. Her alcohol ingestion was now moderate:
3 to 4 beers at least 3 days/week. The patient was again advised to discontinue alcohol, and
she complied. She was referred to a tertiary care center where an endoscopic retrograde
cholangiopancreatography with pancreatic sphincterotomies was performed. A diagnosis
of chronic pancreatTtis'(Cp) secondary to sphincter of Oddi dysfunction wasmade.The
procedure was complicated by her most severe episode of acute pancreatitis, necessitating
a 1O-day hospitalization. Despite complete alcohol abstinence, the woman had.recurrent
attacks of abute pancreatitis requiring hospitalization every other month (on average) for the
next 4 years. Often the painful episodes lacked biochemical evidence of acute pancreatitis
(normal amylase and lipase). For 1 year prior to referral to the medical center, she has had
persistent abdominal pain in between attacks and has required hydrocodone 3 days/week.
The pain is epigastric with infrascapuiar radiation exacerbated by meals. There is no
Continued on Page 5
PHYSICIAN Licensure Examination Page
Sunday, September 10,2017 - O8:OO a.m. - 1O:OO a.m.
PHYSIOLOGY sET B

evidence of glucose intolerance or history of chronic diarrhea. She has lost 1O pounds
unintentionally. Routine physical exam, blood test, and abdominal computed tomography
results are normal. An endoscopic ultrasound demonstrates 6 criteria for CP: hyperechoic
strands and foci, parenchymal lobulation, small shadowing calcifications in the pancreatic
head, an echogenic main duct wall, and an irregular main duct contour. A diagnosis of
idiopathic chronic pancreatitis (lCP) is established. Question: Patient with chronic pancreatitis
is expected to develoP:
A. fat malabsorption
B. pernicious anemia
C. impaired insulin production
D. decrease acid secretion
13. Becoming a military pilot is a formidable physical and psychological challenge. Those hoping
to fly fighier jets musi prove their ability to withstand a high level of G-forces, or the equivalent
of the torce of gravity multiplied. That's why fighter pilots-in-training have to spend some time
in a centrifuge to prove they can handle the pressure. Essentially, they are placed in a
chamber that is on the end of a tong arm lhht spins around the room the faster it spins,
the more G-forces they feel on their body. lt feels like weight is pushing- down on every part
of their body. lf you've ever been on a roller coaster, you know what a quick spike in G-force
feels like. gut tn-e sensation of intense prOssure is greater and longer in duration for someone
strapped into the cockpit of a fighter jet, a vehicle which can "pull" up to nine times the force
of gravity. You Tube is full of videos of what pilots call "g-monsters," or individuals adept at
nanOting the incredible pressure created by the centrifuge, which is meant to simulate a
fighter let's G-force. Question: When a pilot performs a -Gz maneuver immediately followed
by a +Gz maneuver, the G level to cause'unconsciousness during the +Gz maneuver is:
A. increased due to cardiovascular compensation mechanisms
B. decreased due to cardiovascular compensation mechanism
C. unaffected due to cardiovascular comp€nsation mechanism
D. none of these
14. A 4O-year-old man presented to the emergency department with abdominal pain. He
descri6ed the pain as the most excruciating he had ever had. lt developed suddenly, without
warning, t hour before presentation. The pain was located in the right lower quadrant and
radiated to his right flank and groin. lt was slightly alleviated when he was lying supine.
Question: The abdominal pain felt by the patient is most likely conducted by the nerve fibers:
A. A alpha nerve fibers
B. A delta nerve fibers
C. B fibers
D. C fibers
15. A 3Q-year-old male patient is brought to the ER with the following symptoms attributed to a
drug overdose: increased HR and BP, mydriasis, behavioral excitation, aggressiveness,
paranoia and hallucination.Of the following drugs, which one is most likely to be responsible
for these symptoms?
A. amphetamines
B. ethanol
C. fentanyl
D. marijuana
16. A49-year-oldmalecyclist washitbyasports utility vehicle. On arrival in the emergency
department, he was intubated because of respiratory distress and multiple facial fractures.
He had symmetric bilateral breath sounds, but subcutaneous emphysema on the left side. A
chest tube was inserted, which drained 300 mL of blood. His blood pressure was 6O/40 mmHg
andpulse 130/min. Hisabdomenwas distended and the pelvis was stable as were both
femurs. His Glasgow Coma Scale (GCS) wis 15. Chest X-ray showed a chest tube on the
left side in good position, lung contusion and several rib fractures. The laboratory values at
the time of presentation is: Hb 2.7 mmol/L, BE -11.5 mEq/L, APTI >120 sec, PT 34.6 s, and

Continued on Page 6
PHYSICIAN Licensure Examination Page:6
Sunday, September 10,2017 - 08:OO a.m. - 1O:OO a.m.
PHYSIOLOGY sET B

INR 2.33. Massive transfusion protocol was initiated and the patient was immediately
transported to the operating room. Question: The following are consequences of the
decrease in blood volume seen in trauma patients, except
A. increased renal perfusion pressure
B. increased renin secretion
C. increased circulating levels of Angiotensin ll
D. increase circulating levels of Aldosterone
17. A 66-year-old male complaining of chest pain is brought to the emergency room by
ambulance. The pain began six hours ago arld has become more severe over the past hour
prompting him to call emergency services. He describes the pain as retro-stemal, pressure-
like, and non-radiating. He endorses some mild dyspnear and nausea accompanying the pain.
He has also noted intermittent palpitations since last evening. He has a past medical history
of hypertension (not currently under treatment) and tobacco use of one pack per day of
cigarettes for the past 47 years. Vital signs on arrival to t he emergency room show a blood
pressureof 160/100mmHg,aheartrateofg2beatsperminute,andarespiratoryrate of 18
per minute. His physicalexam is unremarkable. He reports no allergies and no medication
use. Question: What factor causing fatal arrhythmia in patient with acute Ml?
A. hypokalemia
B. hyperkalemia
C. hyponatremia
D. hypernatremia
18. A 4O-year-old woman who had been diagnosed as having medication-induced asthma was
admitted with stridor. She had a history of surgery for sinusitis and nasal polyps, and sev€re
asthmatic attack requiring mechanical ventilation after administration of NSAID drugs. On
the second day of hospitalization, she took several Aspirin pills for a stiff shoulder.
Stridor developed five hours later, progressing to a severe asthmatic attack with loss of
consciousness, requiring mechanical ventilation. The serum level of Aspirin was very high,
therefore, bronchial challenge test was performed to confirm the possible relationship
between the ASA agent and asthmatic attack. Results show that Aspirin induced the
asthmatic attack in this patient. Question: Aspirin can precipitate an asthmatic attack
in sensitive individual because it:
A. causes accumulation of arachidonic acid
B. inhibits blood clotting
C. blocks leukotrienes receptors
D. inhibits mast cell degranulation
19. A S5-year-old man with recently diagnosed type2 diabetes mellitus comes to the physician
for a follow-up examination. Physical examination shows no abnormalities. Laboratory
showan increased hemoglobin AIC despite patient compliance with diet and exercise
recommendations. Treatmentwith sulfonylurea is started. Which of the following is most likely
to occur in this patient?
A. decreased entry of glucose into thb muscle cells
B. increased speed of carbolrydrates absorption in the intestine
C. decreased secretion of glucose from the liver
D. increased secretion of insulin from the pancreas
20. Which of the following statements is INCORRECT?
A. Myasthenia gravis is autoimmune disease cause by antibody directed against post-
synaptic acetylcholine receptors
B. Lambert-Eaton syndrome is caused by antibody directed against pre-synaptic calcium
channel in neuromuscular junction
C. Guillen-Barre syndrome causing deseending paralysis while botulism toxin causing
ascending paralysis
D. Duchenne Muscular dystrophy don't produce dystrophin but there is enlargement of the
calf muscle

Continued on Page 7
Page 7
Sunday, September 10,2017 - O8:OO a.m. - 1O:OO a-m.
PHYSIOLOGY SET B

21. What drug is useful in the prevention of Acute Mountain Sickness (AMS)?
A. Lasix
B. Nifedipine
C. Oxygen
D. Acetazolamide
22. Dehydration increases the plasma concentration of all the following hormones, except
A. vasopressin C. norePinePhrine
B. angiotensin ll D. atrial natriuretic peptide
23. What is the result if there is destruction of the optic chiasm?
A. bitemporal hemianoPsia
B. total blifidness
C. homonymous hemianoPsia
D. quadrantanoPsia
24. The caudate, putamen and anterior horn cells are involved in:
A. language comprehension
B. emotional reactions
C. motor function
D. sensory discrimination
25. Which of the following statements is TRUE?
A. The ANS is under voluntary control
B. The sympathetic division of the ANS is known as the "rest and digest division"
C. The parasympathetic division of the ANS is also known as the craniosacral division
D. The somaiic nervous system is involuntary
26. Which of the following would have the greatest effect on peripheral resistance?
A. doubling the length of a blood vessel
B. doubling the diameter of a blood vessel
C. doubling blood viscositY
D. doubling the white blood cell count
27. Closure of the semilunar valves marks the beginning of:
A. ejection Phase
B. isovolumetric relaxation phase
C. ventricular filling Phase
D. isovolumetric contraction phase
28. The over-all regulator of cellular activity is the:
A. mitochondria
B. nucleus
C. endoplasmic reticulum
D. cell membrane
,

29. The plateau seen in ventricular action potential is believed to be due to:
A. Na+ current C. K+ current
B. Ca++ current D. Cl- currenl:
30. When a pheochromocytoma (tumor of the adrenal medulla) suddenly discharges a large
amount of epinephrine into the circulation, the patient's heart rate would be expected to:
A. increase because the increase in blood pressure stimulates the carotid and aortic
baroreceptors . .' I

B. increase because epinephrine has a direct chronotropic effect on the heart


C. increase because of increase tonic paraSympathetic discharge to the heart
D. decrease because the increase in blood pressure stimulates the carotid and aortic
BarorecePtors
Continued on Page 8
PHYSICIAN Licensure Examination Page 8
Sunday, September 10,2017 - O8:OO a.m. -- 1O:OO a'm.
PHYSIOLOGY SET B

31. lmmediately following resection of the spirral cord in man, there is:
A. retention of urine and feces
B- retention of urine with increase muscle tone
C. a period of spinal shock for not more tlran 48 hours
D. loss of reflexes during spinal shock
32. The release of acetylcholine from the synaptic vesicles is facilitated by the influx of:
A. sodium C. lysozyme
B. glycine D. calcium
33. The major reaction in cross-matching is:
A. dono/s cell + patient's serum
B. donor's cell + patient's cell
C. recipient's cell+ donor's plasma
D. none of these
34. Which one of the following do morphine and D-tubocurarine have in common?
A. increase bladder tone
B. ANS ganglionic blockade
C. malignant hyperthermia
D. histamine release
35. The following are true of most WBC,, except
A. can multiply in circulation because they are nucleated
B. have phagocytic property
C. contain myeloperoxidase
D. capable of amoeboid movement
36. ln which situation would stroke volume be the GREATEST?
A, when venous return is increased
B. when venous return is decreased
C. when the force of contraction is decreased
D. when the sympathetic stimulation to the heart is low
37. Why does lactation not occur during pregnancy?
A. prolactin levels surge only after delivery
B. oxytocin from sucking of infant on the nipple stimulates milk let down
C. estrogen and progesterone block prolactin's effects on the breast
D. prolactin levels do not rise until after few hours after delivery
38. Excitation of beta-2 receptors brings about:
A. vasoconstriction
B. pupillary dilatation
C. bronchodilatation
D. bladder sphincter contraction
39. The main cause of symptoms experienced by divers suffering from bends is:
A. oxygen toxicity
B. metabolic alkalosis
C. increase in size of nitrogen bubbles
D. severe respiratory acidosis
40. What transport mechanisms allow the absorption of nutrients across the small intestinal
epithelial brush borders by secondary active transport with sodium?
A. fructose C. fatty acids
B. glucose D- vitamin K

Contintred on Page 9
I,

PHYSICIAN Licensure Examination Page 9


Sunday, September 10,2017 - O8:OO a.m. - 1O:OO a.m.
PHYSIOLOGY SET B

41. No wave summation is observed in cardiac muscle responae because of the:


A. prolonged refractory period
B. low resting potential I

C. Iesser excitability of cardiac muscle I

D. absence of nerves
42. A45-year-old man comes to the physician for a follow-up examination 1 week after he passed
a renal calculus. X-ray crystallographic analysis of the calculus showed calcium as the
primary cation. Physical examination today shows no abnormalities. A24-hour collection of
of urineshowsincreasedcalciumexcretion. Whichofthe following is the most appropriate
pharmacotherapy?
A. carbonic anhydrase inhibitor
B. Na+-€l- symport
C. osmotic diuretic
D. renal epithelial sodium channel inhibitor
43. The velocity of conduction of action potentials along a nerve will be increased by:
A. stimulating the Na+-K+ PumP
B. inhibiting the Na+-Cl- symport
C. decreasing the diameter of the nerve
D. myelinating the nerve
44. Erythroblastosis fetalis is most instances is likely to occur when:
A. Rh+ mother carries an RH+ fetus
B. Rh+ mother carries an RH- fetus
C. Rh- mother carries an RH- fetus
D. Rh- mother carries an RH+ fetus I

45. ln a person standing upright, which region of the lungs has the highest ventilatibn rate and
which region has the highest circulatory perfusion rate?
A. highest ventilation: Apex ; highest perfusion: Apex
B. highest ventilation: Apex ; highest perfusion: Base
C. highest ventilation: Base ; highest perfusion: Apex
D. highest ventilation: Base; highest perfusion: Base
46. The blood vessels that play the most important role in the regulation of blood flow to a tissue
and blood pressure are the:
A. Arteries C. Capillaries
B. Arterioles D- Venules
47. Which of the following transport processes will NOT require the direct input of energy in the
form of ATP?
A. movement of the sodium out of the c4ll
B. movement of glucose into muscle cells
C. movement of calcium into the sarcoplasmic reticulum
D. movement of hydrogen ion into the lumen of the distal nephron
48. After ingestion of one (1) liter of water:
A. free water clearance increases
B. urine osmolality increases
C. ICF volume maintained
D. solute excretion decreases
49. A new drug is developed that blocks the transporter for H+ secretion in gastric parietal cells.
Which of the following transport processbs is being inhibited?
A. simple diffusion
B. facilitated diffusion
C. primary active transport
D. counter transport
Continued on Page 10
PHYSICIAN Licensure Examination Page 1O
Sunday, September'1O,2017 - 08:00 a.m. - 1O:OO a.m.
PHYSIOLOGY SET B

50. Muscle stretch leads to a direct increase irr firing rate of which type of nerve?
A. o - motoneurons
B. V - motoneurons
C. group 1a fibers
D. group 1b fibers

51. The following are stimulators of erythropoietin (EPO) production, except


A. high altitude
B. renalfailure
C. decreased 2,3 DPG
D. androgens
52. At which site is systolic blood pressure the highest?
A. renal artery
B. renal vein
C. pulmonary artery
D. aorta
53. A Z4-year-old woman presents to the emergency department with severe diarrhea. When
she ii supine (lying down), her blood pressure is 90/60 mm Hg (decreased) and her heart
rate is 1OO beats/min (increased). When she is moved to a standing position, her heart rate
further increases lo 12O beats,/min. Which of the following accounts for the further increase
in heart rate upon standing?
A. decreased total peripheral resistance
B. increased vasoconstriction
C. increased contractility
D. decreased venous return
54. The counter - current exchange mechanism depends most directly on which set of blood
vessels in the kidney?
A. renal arteries C. vasa recta
B. renal veins D. afferent arterioles
55. Cyanosis (a bluish color of the skin and mucous membranes) indicate's a higher than normal
blood concentration of:
A. carbon dioxide
B. carbon monoxide
C. diphosphoglycerate (DPG)
D. reduced hemoglobin

56. Which of the following associations is CORRECT?


A. Glomerulus - filtration
B. PCT - reabsorption
C. Collecting Duct - renin release
D. Loop of Henle - urine concentration
57. Which of the following is a consequence when the anterior lobe of the pituitary gland is
resected?
A. inability to secrete epinephrine during stress
B. inability to secrete insulin during oral glucose challenge test
C. inability to increase ADH in response to water deprivation test
D. inability to stimulate production of thyroid hormones
58, Hematocrit refers to:
A. a percent of the blood that is cell
B. the amount of iron in the blood
C. the criticalsedimentation rate
D. the amount of hemoglobin in red cells

Continued on Page 11
PHYSICIAN Licensure Examination Page 111
1
Sunday, September 10,2017 - O8:OO a.m. - 1O:OO a.m.
PHYSIOLOGY SET B

59. A 39-year-old man with untreated diabetes mellitus type 1 is brought to the emergency room.
An injection of insulin would be expected'to cause an increase in his:
A. breathing rate
B. blood glucose concentration
C. blood K+ concentration
D. blood pH
60. Which of the following conditions decreases the likelihood of edema formation?
A. arteriolar constriction
B. standing
C. nephrotic syndrome
D. inflammation
61. lf the Expiratory Reserve Volume (ERV) is 1,000 ml and the Functional Residual Capacity
(FRC) is 3,5OO ml, what is the resultant ResidualVolume (RV)?
A. 4,500 ml C. 2,500 ml
B. 3,500 ml D. 1,500 ml
62. Which of the following substances increases the sensitivity of pain receptors?
A. bradykinin C. serotonin
B. histamine D. prostaglandin
63. When this hormone binds to its receptors, located in the cell membrane, it cause an increase
in cAMP levels. This second messenger in turn causes Sertoli cells to grow and secrete
spermatogenic factors. What most likely hormone was described?
A. Luteinizing hormone
B. Follicle-stimulating hormone
C. Adrenocorticotropic hormone
D. Parathyroid hormone
64. When parietal cells are stimulated, they secrete:
A. HCI and intrinsic factor
B. HCI and pepsinogen
C. HCI and HCO" -
D. HCO" - and pepsinogen
65. Due to polio, a patient suffers total paralysis of his intercostal muscles, for this patient, which
of the following values would still be expected to be essential normal?
A. inspiratory reserve volume (lRV)
B. expiratory reserve volume (ERV)
C. total lung capacity (TLC)
D. none of the above
66. According to Frank-Starling Law of the Heart, the cardiac output is directly related to:
A. Mass of the ventricular myocardium :

B. Pulse pressure
C. End diastolic volume
O. End systolic volume
67. Exercise prior to decompression:
A. increase incidence and severity of decompression sickness (DCS) during subsequent
high altitude exposure
B. can reduce inoidence of decompression sickness (DCS) during subsequent high altitude
exposure if performed during prebreathe.
C. increases the likelihood of hypoxia during subsequent high altitude exposure due to
residual effects of increased metabolic rate.
D. has no effect on incidence and severity of decompression sickness (DCS) during
subsequent high altitude exposure.

Continued on Page 12
PHYSICIAN Licensure Examination
Sunday, September 10,2C17 - O8:OO a.m. - 1O:OO a.m.
PHYSIOLOGY SET B

68. Which of the following is absorbed by facilitated diffusion?


A. glucose in duodenalcells
B. fructose in duodenal cells
C. vitamin B1 in duodenal cells
D. cholesterol in duodenalcells
69. Reason why tetanic contraction is'not observed in cardiac muscle
A. longerdepolarization
B. presence of plateau phase in the action potential
C. longer refractory period
D. less calcium stores
70. Its transport is responsible for the rapid rr=polarization of the membrane:
A. Na+ C. CI-
B. K+ D. all of these
71. Bile salts as well as vitamin E12 are normally absorbed in the:
A. jejunum C. duodenum
B. colon D. all of these
72. ln beta-cell failure of the lslet of Langerhans, there will be deficiency in the secretion of:
A. glucagon C. insulin
B. somatostatin D. all of these
73. Which of the following does not dilate arterioles in the skin?
A. increased body temperature
B. epinephrine
C. bradykinin
D. substance P
74. Which of the following statements is true regarding internal respiration?
A. the rough endoplasmic reticulum primarily uses 02
B. 02 tension in the cytoplasm is higher than arterial blood
C. respiratory quotient is higher if carbohydrate is main source of energy
D. a high cellular 02 tension produces lactic acid production
75. Which of the following factors increase pre-load?
A. increased blood volume
B. horizontal posture or negative intrapleural pressure
C. increased sympathetic tone to veins
D. all of the above
76. An intestinal hormone that stimulates the gallbladder to release bile is:
A. secretin
B. gastrin
C. cholecystokinin
D. enterokinase
77. When a person hyperventilates at altitude:
A, the blood pH increases
B. the respiratory quotient is driven below 1
C. the kidney will retain bicarbonate
D. carbonic acid levels fall faster than carbon dioxide levels
78. The highest proportion of stored oxygen in the body is derived from:
A. oxygen dissolved in body fluids
B. orygen combined with hemoglobin in the blood
C. oxygen in the air inside the lungs
D. oxygen combined with myoglobin in muscles
Continued on Page 13
PHYSICIAN Licensure Examination Page13
Sunday, September 10,2017 - O8:00 a.m. - 1O:OO a.m.
PHYSIOLOGY sET B

7g. As carbon dioxide diffuses from the tissues to the plasma and subsequently to the RBC, the
combination of CO2 with hemoglobin facilitates the offloading of oxygen. This is known as
the
A. Chloride shift
B. Bohr effect
C. Haldane effect
D. Gibbs.Donnan equilibrium
80. Within physiologic limits, the heart pumps all the blood that comes to it without allowing
excessive damming in the veins. This intrinsic ability of the heart is called:
A. Stokes-Adams phenomenon
B. Bohr effect
C. Frank-Starling law
D. Hering-Breuer reflex
81. A55-year-oldmancomestohisprimarycare clinician complaining of erectile dysfunction.
He is given a prescription forViagra, and on follow-up, reports that his ability to sustain an
erection has been improved markedly by this treatment. The action of which of the following
vasoactive mediators would primarily be increased in this patient?
A. histamine
B. prostacyclin
C. nitric oxide
D. atrial natriuretic peptide
82. During which phase of the cardiac cycle does the mitral valve open?
A. isovolumetric ventricular contraction
B. rapid ventricular ejection
C. reduced ventricular ejection
D. isovolumetric ventricular relaxation
83. The most accurate statement concerning the physiologic role of oxytocin during normal labor
and delivery is:
A. inhibits cervical dilatation
B. stimulates uterine smooth muscle contraction
C. secreted by the adenohypophysis
D. decreases uterine irritability
84. One of the following has opiate-like effect:
A. GABA C. endorphins
B. acetylcholine D. s-HT
85. Compared with systemic arterial blood, pulmonary arterial blood has a higher:
A. oxygen content
B.pH
C. bicarbonate ion concentration
D. Hb concentration
86. Curare blocks the transmission of impulses across the neuromuscular junction by:
A. competing with acetylcholine for the receptor sites at the muscle cell membrane
B. preventing release of the neurotransmitter
C. causing hydrolysis of neurotransmitter
D. making the muscle cell membrane permeable to K+
87. When an individual is naked in a room in which the air temperature is 21"C (69.8"F) and the
humidity 8O%, the greatest amount of heat is lost from the body by
A. respiration
B. urination
C. vaporization of sweat
D. radiation and conduction
Continued on Page 14
PHYSICIAN Licensure Examination Page 14
Sunday, September 10,2017 - OB:OO a.m..- 1O:OO
PHYSIOLOGY SET B

88. Which of the following neurons release norepinephrine?


A. sympathetic preganglionic neurons
B. parasympathetic preganglionic neurons
C. sympathetic postganglionic neurons
D. preganglionic postganglionic
89. For the past several months, a 67-year-old woman experienced difficulty initiating and/or
maintaining sleep several times a week. A friend suggested that she take melatonin to
regulate her sleep-wake cycle. Melatonin secretion would probably not be increased by:
A. stimulation of the superior cervical ganglia
B. intravenous infusion of tryptophan
C. intravenous infusion of epinephrine
D. stimulation of optic nerve
90. The tendency for blood flow to be turbulent is increased
A. increased hematocrit
B. partial occlusion of blood vessel
C. increased viscosity
D. decreased velocity of blood flow
91. Which of the following solutions will most likely yield the highest amount of urine volume?
A. isotonic C. hypertonic
B. hypotonic D. osmotic diuretic
92. Which of the following is the result of atropine poisoning?
A. salivation C. increasing intestinal motility
B. hypotension D. dilatation of the pupils
93. A 28-year-old man was seen by a neurologist because he had experienced prolonged
episodesof tingling and numbness in his right arm. He underwent a neurologic exam to
evaluate his sensory nervous system. Which of the following receptors is correctly paired with
the type of stimulus to which it is most apt to respond?
A. Pacinian corpuscles and motion
B. Meissner corpuscle and deep pressure
C. Merkelcells and warmth
D. Ruffini corpuscles and sustained pressure
94. A newborn baby is brought to the pediatrician suffering from severe diarrhea that worsen with
meals. The symptoms diminish when nutrients are delivered intravenously. The child most
likely has a mutation in which of the following intestinal transporters?
A. N+, K+ATPase C. H+, K+ATPase
B. SGLT-1 D. NKCC1
95. When a person rises quickly from a prone position:
A. the carotid baroreceptors are unaffected
B. venous return is increased
C. vasoconstriction of peripheral blood vessels soon follows
D. heart rate decreases
96. A 57-year-old professor at a medical school experienced numerous episodes of a sudden
loss of muscle tone and an irresistible urge to sleep in the middle of the afternoon. The
diagnosis was narcolepsy, which
A. is characterized by a sudden onset of NREM sleep
B. has a familial incidence associated with a class Il antigen of the major histocompatibility
complex
C, maybe due to the presence of an excessive number of orexin-producing neurons in the
hypothalamus
D. is often effectively treated with dopamine receptor agonists

Continued on Page 15
PHYSICIAN Licensure Examination Page 15
Sunday, September 'i'O,2017 - O8:OO a.m. - 10:00 a.m.
PHYSIOLOGY SET B

97, Hearing aids are most usefulfor individuals suffering from:


A. central deafness
B. conduction deafness
C. sensorineural defects
D. damaged Organ of Corti
98. Ketoacidosis in untreated diabetes mellitus is due to:
A. excessive fluid loss
B. hypoventilation
C. excessive eating and obesity
D. excessive fat catabolism
99. Many of the regulatory mechanisms of the body operate on the principle of
A. negative feedback
B. positive feedback
C. reciprocal innervation
D. active transport
100. Of the total body water (H2O), the largest fraction is found in the:
A. plasma
B. interstitial fluid
C. intracellular fluid
D. transcellular fluid

*r* END *r*

WARNING: Failure to submit your Test Questions (Complete) set will cause the cancellation
of your Test-Result for this subject.
PHYSIOLOGY 2017
ANSWER KEY

1. C 26. B 51. B 76. C


2.8 27. B 52. A 77. A
3.C 28. B s3. D 78. B
4.8 29. B 54. C 79. B
5.D 30. B 55. D 80. c
6.E} 31. C 56. C 81. C
7. A 32. D 57. D 82. D
8.4 33. A 58. A 83. B
9.4 34. D 59. D 84. C
10. A 35. A 60_ A 85. C
11. D 36. A 61. C 86. A
12. A 37. C 62. D 87. D
13. B 38. C 63. B 88. C
14. D 39. C 64. A 89. D
15. A 40. c 65. D 90. B
16. A 41. A 66. C 91. D
17. B 42. B 67. B 92. C
18. A 43. D 68. B 93. D
19. D 44. D 69. C 94. B
20. c 45. D 70. B 95. C
21. D 46. B 71. A 96. B
22. D 47. B 72. C 97. B
23. A 48. A 73. D 98. D
24. C 49. C 74. C 99. A
25. C 50. c 75. D 100. c
Seat No.
Republic of the Philippines
PROFESSIONAL REGULATION COMMISSION
Manila
BOARD OF MEDICINE

PHYSICIAN Licensure Examination


Sunday, September 10, 2017
LEGAL MEDICINE. ETHICS AND MEDICAL JUR]SPRUDENCE SET B

INSTRUCTION: Select the correct answer for each of the following questions.
ft4*t gnlL."" *I for each item by marking the box corresponding to the letter of your choice
on the inswer sheet provided. STRICTLY NO ERASURES ALLOWED. Use pencil no. 2 only.
MULTIPLE CHOICE:

1. A doctor who is found guilty of unprofessional conduct by the Board of Medicine shall suffer
the following consequences, except
A. censure or reprimand
B. revocation of his certificate of registration
C. corresponding fine for his demeanor
D. write a letter of explanation
2. The correct authority to received reports of child abuse
A. barangay captain
B. medical director
C. DSWD representative
D. women & children protection desk
3. A financially trapped surgeon known for being habitual traffic offender performed his last
cosmetic operation case at a local hospital in Davao City. On his way home, 2 unidentified
motorcycle riding men accidentally shot him contributing to his early demise. Which of the
following choices are considered legally appropriate?
A. The claim for professional fee cannot be made from the patient or her descendants since
the surgeon's civil and political rights were disenfranchised due to his early death.
B. As a legal entity, the hospital can follow up and manage the procedures left by the surgeon.
C. Asset forfeiture including surgeon's instruments and library by a law enforcement officer
or sheriff may now be legally justified.
D. The surgeon's traffic citation ticket are eligible for the penalty fee waiver.
4. The type of witness who is a publicity seeker
A. reluctant witness
B. destructor witness
C. hostile witness
D. false witness
5. Which of the following principles apply to the action of the physician to refrain from discussing
the disease condition of the patient with those who are not involve with his care?
A. Beneficense
B. Non-Maleficense
C. Justice
D. Autonomy
6. Which of the following is the least concern of the PMA's Ethics Committee?
A. Doctors caught violating the state's driving laws
B. Doctors acting as product promo merchandizer
C. Tactless commentary against physician colleague
D. Social viral video showing a doctor berating a patient
7. Defined as spontaneous return of circulation after failed attempts at resuscitation
A. suspended animation C. near death experience
B. comatose D. Lazarus phenomenon

Continued on Page 2
PHYSICIAN Licensure Examination Page:2
Page
Sunday, September 10,2017 - 1 1:00 a.m. - 01:00 p.m'
LEGAL MEDICINE. ETHICS AND MEDICAL JURISPRUDENCE SET B

8. Who can be held liable for any inaccuracies done in the medical record of the patient?
A. the doctor C. both the doctor and the hospital
B. the hospital D. the patient
g. A child deserted by his parents or guardians for six continuous months is classified as
A. Neglected C. Abandoned
B. Dependent D. Abuse
10. Which of the following statements is incorrect?
A. Virginity is synonymous to chastity.
B. Seducement is synonymous to disenchantment.
C. Lasciviousness is synonymous to voluptuousness.
D. Carnal knowledge is synonymous to fornication.
11 . A 2 year-old child is brought to a newly open state of the art medical center with dry lips,
wrinkled skin, rapid breathing and blotchy hands & feet. You are quite certain the child is
severely dehydrated and need medical attention. When you discuss the need for scalp vein
catheteiization and rehydration treatment, the parents refuse permission saying "We'd prefer
to take him home and have Mang Kanor ciur Spiritual Leader pray over him". How should the
physician
-The handle this?
A. physician should do what the paients request to take the child home.
B. The physician should call the parent's religious leader and have him come to the
hospital before providing standard medical care.
C. The physician has no duty to provide treatment to the child when the parents refuse
treatment.
D. When effort to obtain parental permission to treat patient fail, the physician is justified
in seeking legal help or maybe legally authorized to proceed with the procedure and
treatment of the child.

12. Which of the following will warrant the need for detention of a patient in the hospital?
A. lf mentally ill
B. The illness presents danger to the public
C. When underage
D. The patient has no accompanying relative
13. ln carrying out capital punishment by death, lethal injection uses the following drugs, except
A. Sodium pentothal C. Hyoscine hydrobromide
B. Pancuronium bromide D. Potassium chloride
14. Situations when confidentiality can be broken
A. lnforming patient's relative that the patient is suicidal.
B. lnforming employee's illness which may endanger either his performance on a job cir his
own health as he performs the job.
C. The patient waives to the confidential,nature of his illness and allowing a third party to be
present.
D. All are correct
15. An iconic plastic surgical clinic advertises breast augmentation, tummy tuck and liposuction
services with assurance from prospective clients high success rate as shown on their
facebook page. What do you think is the kind of medical fee for this kind of promotion?
A. contingent fee C. straight fee
B. retainer fee D. dichotornous fee
'16. ln treatment of a patient, a physician is obligated to
A. care for him to the best of his ability
B. care for him in accordance with an obje'ctive standard of excellence
C. act with the same degree of care as other physicians in the area within the framework of
the science as then known
D. act with a degree of care commensurate to the patient's ability to pay

Continued on Paqe 3
PHYSICIAN Licensure Examination
Sunday, September 10, 2017 - 1 1:00 a.m. - 01:0O p,m.
LEGAL MEDICINE. ETHICS AND MEDICAL JURISPRUDENCE SET B

17. Criminal liability does not arise when a non-therapeutic abortion


A. is advised but not acted upon by the pregnant,woman
B. is advised but the woman is not pregnant
C. is advised and a drug is taken which id ineffective
D. is performed and it is clear that if a child were born it would have died at birth
18. Fingerprints left on various surface at the crime scene which are not clearly visible
A. plane impression
B. visible fingerprints
C. rolled impression
D. latent fingerprints
19. There are three administrative bodies tasked to regulate the licensure of the medical
profession and maintain the high quality and standards of the medical practice in the
Philippines. Which one of the foltowing is not included in the regulation task?
A. Commission on Higher Education
B. Philippine Medical Association
C. Professional Regulation Commission
D. Board of Medicine
20. Which of the following statements regarding consent for treatment procedure to a patient is
incorrect?
A. lnformed consent is very necessary and needs explanation of the condition, procedures
and possible risk to the patient.
B. Consent must be for hospital admission, for surgery & anesthesia and for each & every
invasive procedures.
c. Emotional immaturity and high level of stress are still acceptable criteria, in signing
consent since these are exclusion to.the notion of inadequate reasoning faculties.
D. lnformed consent is pattern according to the guidelines set by medical ethics.

21. The three responsibilities of the hospital in emergency cases are the following, except
A. admission C. transfer of patient
B. management D. discharge
22. A physician criticized in public the treatment rendered by a colleague. The offending
physician may be charge with
A. libel
B. slander
C. unprofessional conduct
D. immoral conduct
23. Eleemosynary hospital is classified under
A. government hospital
B. private charitable hospital
C. public hospital
D. private pay hospital
24. A barrio doctor was conducting house to house community health teaching when he
accidentally caught an addict smoking "shabu" (methamphetamine) in front of him. What
would be the appropriate legal term for the above scenario which the physician just solely
witness?
A. Corpus Delicti C. En Flagrante Delicto
B. Res lpsa Loquitor D. Non Compos Mentis
25. The cellular or molecular stage of death is known as
A. primary flaccidity
B. rigor mortis
C. secondary flaccidity
D. putrefication

Continued on Page 4
PHYSICIAN Licensure Examination
Sunday, September rc,2017 - 1 1:00 a.m. - O1:0O p.m.
LEGAL MEDICINE. ETHICS AND MEDICAI- JURISPRUDENCE SET B

26. The child born within 180 days following celebration of marriage is prima facie presumed to
be legitimate and such child is conclusively presumed legitimate if
A. Sexual intercourse can be proven before the celebration of the marriage.
B. lf the man consented, being present, to the putting of his surname to the record of birth of
the child.
C. lf the child resemble the alleged father
D. lf both husband and wife have accessed to each other for a long period of time.
27.To determine choline in human semen, the technique that is used in the laboratory
A. Barberio's test C. Florence test
B. Precipitin test D. Teichman test

28. A process in court Pillar of our criminal justice system where the accused is brought before
the court and aPPraised of the formal charges and informed of his constitutional rights.
What is this process?
A. Plea bargaining C. Arraignment
B. Custodial investigation D. Pretiminary investigation
29. The following types of wounds are caused by a blow with a hard blunt solid instrument, except
A. incised wound C. fracture
B. abrasion D- lacerated wound
30. A 70 year old grandmother shows signs of abuse that appears to be inflicted by her husband.
As h6 is her frimary caregiver, she feels dependent on him and pleads with you not to say
-TtriJisto him about it. How would you handle
anything this Situation?
R- a case of elder abuse and the cloctor is required to always report'incidents of
abuse to the right Philippine authorities.
B. The doctor is not permitted under R.A. 9262 (Anti-Violence Against Women & Children
Act) to report abuse.
C. The law supporting reporting elder abuse allow the doctor to break confidentiality and
report suspected abuse.
D. The patient should not be reported. lnstead, she should obtain support and access to
another services in order to maintain her primary caregiver.
31. When somebody is hit by a lightning during a storm, arborizing red skin marks may be
produce. This is called
A. Lichtenberg skin C. Goose flesh skin
B. Crocodile skin D. Parchment skin
32. A person who gives his/her opinion or conclusion on a given scientific evidence is considered
A. interrogator C. prosecutor
B. expert witness D. ordinary witness

33. Chief Surgeon Magaling accidentally cut the patient's laryngeal nerve during a botched up
RND procedure and curiously witnessed by one of his surgical staff. Which doctrine is
applicable to this case?
A. Doctrine of Avoidable Consequence
B. See No Evil, Hear No Evil, Speal< No Evil Doctrine
C. Doctrine of Res lpsa Loquitor
D. None of these
34. Strictly speaking caliber means
A. diameter of the cartridge shell C. power of the gun
B. diameter of the barrel of the gun D. power of the bullet
35. A patient with PTB was erroneously diagnosed as lung CA by a licensed MD. This was the
result of the non performance of the properly diagnostic work-ups. The doctor is guilty of
A. criminal negligence C. dereliction of duty
B. malpractice D. illegal practice of medicine

Continued on Paoe 5
PHYSICIAN Licensure Examination age
Sunday, September 10,2017 - 11:OO a.m- - O1:00 p.m.
LEGAL MEDICINE. ETHICS AND MEDICAL JURISPRUDENCE SET B

36. The act of lasciviousness encompasses the following examples, except


A. The act of making a woman take off her clothes and dance before a crowd.
B. The act of stealthily approaching a woman and then embracing her.
C. The act of approaching a woman and getting hold of her breast.
D. The act of ailowing any form of liberties to a woman but not to the extent of rupturing her
hymen. i

37. Upon passing the physician licensure examination, which of the following is a priority action
of a newly successful examinees?
A. Take an oath of profession before the Board of Medicine
B. Apply membership to the Philippine MedicalAssociation
C. Prepare credentials for job opportunities
D. Apply registration with the Professional Regulation Commission
38. A resident surgeon attended his first lnternational Convention and brought with him several
items from his leisure trips abroad. Which of these items are considered taxable airport duties
that does not extend beyond the surgeon's incidental rights?
A. General Surgery Books, lnternational Edition #5
B. Surgical lnstrument Products #10
C. Boxes of Chromic Catgut & Steel Sutures #2O
D. All are Subject to Duties and Taxes
39. When an addict is suddenly deprived of opiates, the following symptoms occurs, except
A. loss of weight and appetite
B. frequent yawn, sweat and lacrimation
C. restlessness and anxietY
D. pupils are constricted, gooseflesh and shivering
40. A master is liable for the wrongful acts of his servant and the principal for those of agents
A. Doctrine of Res lpsa Loquitor
B. Doctrine of Assumption of Risk
C. Doctrine of Respondent Superior
D. Doctrine of Foreseeability
41. The physician wrote a medication order for a patient. The nurse thought the dosage was
incorrect. She questioned the physician who said it was alright. The nurse gave the
medicine and the patient died from an overdose. Who is liable?
A. the physician
B. the nurse ,

C. both the physician and the nurse


D. the hospital
42. Cafe coronary is a sudden heart attack in the middle of dinner due to asphyxia by choking.
What manner of death is this?
A. Accidental death C. Undetermined death
B. Naturaldeath D. Pending death
43. Dr. Tokhang was coming home from United Arab Emirates where he worked for 2 years
when found to have several ampules of Fentanyl, Morphine and Pethidine in his hand carried
luggage. He was right away detained for interrogation at the airport. To protect himself fronr
future legal issues, Dr. Tokhang must see to it that he followed legal prophylaxis. Which of
these remedies should he follow for his own judicial protection?
A. The traveler should have in his possession triplicate prescription form and narcotic
license issued by the Philippine Dangerous Drug Board.
B. The traveler should have in his possession the approved documents from the
Registration and Drug Department, Ministry of Health of his last country destination.
C. The traveler should invoke his incidental or inherent rights since he's a fully registered
physician of the Philippines.
D. The traveler should catry a letter or copy of his prescription (written in English) from
his doctor to certify that the Medicine has been prescribed to him to treat his medical
condition'
continued on paoe 6
PHYSICIAN Licensure Examination " Page 6'
Sunday, September 10,2017 - 1 1:00 a.m. : 01:00 p.m.
LEGAL MEDICINE. ETHICS AND MEDICAL JURISPRUDENCE SET B

44. A common ethical concept of medicine which reflects of a professional value of avoiding
harm to the patient in the executory of task is
A. Beneficense C. Autonomy
B. Non Maleficense D. Veracity
45. The Board of Medicine has these functions, except
A. To study the conditions affecting the practice of medicine in all parts of the Philippines.
B. To issue subpoena duces tecum and subpoena ad testificandum to violators of R.A. 2382.
C. To approve registration without examination.
D. Tohandle examinations upto 5subjects per exarninerupon the approval oftheboard
chairman.

46. To ensure competent practice and quality care management, doctors must
A. attend seminars, symposia and forum
B. conduct or participate in research
C. go on graduate studies
D. practice medicine
47. Pre-employment doctors who deliberately altered OFWs medical certificates even if results
of repeated diagnostic work-ups done by thiid party accredited laboratory showed "FlT TO
WORIC, is guilty of
A. malpractice C. illegal practice of medicine
B. dereliction of duty D. no liability

48. Method use to determine the passing average of the entire batch of examinees by
transmuting each examinee's raw score based on the correct mean passing level
A. Angoffs method C. Ebel's melhod
B. Nedelsky method D. All of thesb
49. ln each of the following situations, which one is not considered an implied consent?
A. A surgeon is performing Billroth ll procedure on his patient. During the surgery, he sees
the patient's deceased appendix. The surgeon removed it without asking either the patient
or his surrogate.
B. Following a vehicular accident a tracheostomy is performed to produce an airway for a
seriously injured unconscious patient without waiting for his consent.
C. During an appendectomy, the surgeon discovers an abnormal ovary and decided to
remove it without the patient's consent.
D. A child in a life/death emergency situation cannot give consent for surgery. The doctor
may decide surgery is necessary to save the child's life and proceed without consent.

50. This refer to the deflection of the bullet from its normal path after striking a resistant surface
A. misfire C. ricochet
B. mushroom D. key hole shot
51.A65-year-oldman has a heartattack and is admittedtothemedicalfloorwithaverypoor
prognosis. He ask that you not share any of his medical information with his wife as he does
not think she will be able to take it. His wife catches you in the cafeteria and ask about her
husband's prognosis. What are you required to do legally?
A. The doctor should inform the wife about her husband's poor prognosis.
B. The doctor should not divulge the prognosis to the wife, but he should ask the nurse to let
the wife know about her husband:s iondition.
C. The wife is certainly affected by hqr husband's health and prognosis and every effort
should be made to encourage an open dialogue between them.
D. The doctor should not encourage the patient to talk to his wife about his condition.
52. Which of the following is the most widely accepted and practical way of determining the
presence of abused drug in the body?
A. blood C. body fluid
B. saliva D. urine

Continued on Page 7
PHYSICIAN Licensure Examination Page 7
Sunday, September 10,2017 - 1 1:0O a.m. - O1:OO p.m.
LEGAL MEDICINE. ETHICS AND MEDICAL .JURTSPRUDENCE SET B

53. Warrant of arrest and search warrant can be issued by a


A. judge C. prosecutor
B. police D. all of these
54. Medico legal or official autopsy is an examination performed on a dead body for the purpose
of?
A. Studying the natural course of disease process.
B. Providing correlation of clinical diagnosis and clinical symptoms.
C. Providing the effectiveness of the therapy.
D. Providin! interpretation and correlation of facts and circumstances related to death.
55. Addiction to alcoholic beverages is
A. criminal malpractice C. administrative malpractice
B. civil malpractice D. ethical malpractice
56. Before temporarily detaining or restraining a person of interest, you need to consider the
following circumstances, except for
A. citizen arrest
B. convicted felons held incommunicado
C. mentally ill patient endangering public safety
D. ongoing suspension of the writ of habeas corpus & declaration of martial law
57. A person with an intelligent quotient of 55 is considered
A. profound retardation C. mild retardation
B. moderate retardation D. moral defective
58. The first "police surgeon" and forensic pathologist
A. Antistius C. Paulus Zacchias
B. Justinian D. Pedro Solis
59. Which is NOT sufficient ground for penalizing a physician?
A. A physician demanding aggressive treatment that seemed inappropriate since the
patient is already dead.
B. Admitting patient with an emerging infectious disease in an over-crowded community
hospital.
C. A doctorwho makes contractuat pricing agreement that stipulates payment bythe hour
for a lengthy surgical Procedure.
D. Doctors who socially drink alcoholic beverages before or during hospital working hours.
60. Branch or division of law which defines crimes, treat of their nature and provide for their
punishment
A. Criminal law C. Legal medicine
B. Civil law D. Medical jurisprudence
61. The following are specific rules to be followed in the formulation of the questions in a polygraph
test except one
A. Questions must be clear and phrased in language the subject can easily understand
B. Questions must be answerable by yes or no
C. Questions must be short as possible
D. Questions must be in the form of accusations
62. lt is an expressed acknowledgement by the accused in a criminal case of the truth of his
guilt as the crime charged
A. interrogation C. admission
B. confession D. deception
63. The following are factors to consider in approximating theduration of death in a cadaver,
except
A. entomology C. washerwoman skin
B. blood vessel clot D. food in stomach
(aanfinrrad nn Paoc 8
PHYSICIAN Licensure Examination Page 8
Sunday, September 10,2017 - 11:OO ?.rr1. - 01:O0 p.m.
LEGAL MEDICINE, ETHICS AND MEDICAL JURISPRUDENCE SET B

64. The form of child abuse that can range from habitual humiliation of a child to withholding life-
sustaining nurturing refers to
A. mental abuse C. physical abuse
B. emotional abuse D. economic abuse

65. lf you happen to eavesdrops a private conversation between a doctor (ophthalmologist) and
a patient (general practitioner):
PATIENT: "Doc, magkano po sa pagtingin ninyo sa sore eyes ko?".
DOCTOR: "Php4,000 po!". l

PATIENT: "Ang mahal pala doc. Wala po'ba discount?".


DOCTOR: "Wala po. Ganyan po talaga ang rate sa mga private ophtha clinics".
PATIENT: "Libre na lang po doc. Physician din nanran po ako".
DOCTOR: "Ah ganun ba. Bakit ngayon fno lang po sinabi na doctor po kayo? Sige, bigyan
na lang kita ng discount. Kalahati na lang po ang consultation fee".
ln the above situational exchange of words, whiCh of these is a great concern to you as a
physician?
A. The specialist overcharged the patient as per Philhealth guidelines.
B. The specialist should not charge the patient (gratuitous service to colleague).
C. Corresponding payment must be made by the patient for the services rendered.
D. Physician's incidental rights to compensation was violated.
lllegitimate children includes the following, except
A. children born of incestuous marriage
C. children born of couples below 18 years of age
D. children born to prostituted mothers ,

67 Tache noir de la sclerotique. This sign found after death is in the


A. cornea C. retina
B. lens D. sclera
68. A specie of conveyance whereby a person is permitted, with the formalities prescribed by
law, to dispose of his property after his death with more or less freedom but limited to a certain
degree by law is called
A. testament c. codicil
B. wilt D. succession

69. ln fingerprint work, the term applied to a specialized study of the sweat gland opening of the
papillary ridges of the skin as a means of idehtification is
A. podoscopy C. chirosc6py
B. dactyloscopy D. poroscopy
70. Evidence which is the repetition of what others have said and is not from the personal
knowledge of the witness
A. opinion evidence C. competent evidence
B. hearsay evidence D. real evidence
71. Toxicologic examination on the fingernails of a deceased person showing "mees line" is a
determination of what kind of poisoning?
A. Potassium cyanide C. Carbon monoxide
B. Arsenic poisoning D. Lead poisoning
72. A 3 month old has been admitted to the hospital with a newly diagnosed ventricular septal
defect. She is in early congestive heart failure and Digoxin is indicated. After discussing the
proper dose with the attending physician, you write an order for the drug. Thirty minutes later
the baby vomits and dies. You discover that in writing the Digoxin order you misplaced the
decimal point and the child got 1O times too much Digoxin. What is your duty here?
A. lnform the parents about the mistake! and say you are sorry.
B. Do not inform the parents.
C. Do not say sorry.
D. Let the hospital Legal Department handle evefihing, because a malpractice lawsuit
may follow

Continued on Page 9
PHYSICIAN Licensure Examination Hage 9I
Page
Sunday, September 10,2017 - 1 1:OO a.m. - O1:00 p.m.
LEGAL MEDICINE. ETHICS AND MEDICAL JURISPRUDENCE SET B

73. Also known as physician assisted death, physician assisted suicide or mercy killing
A. voluntary euthanasia C. passive euthanasia
B. involuntary euthanasia D. active euthanasia
74. Which of the three branches of the government is the most powerful?
A. Executive branch C. Judicial branch
B. Legislative branch D. All of these
75. Which statement is incorrect?
A. Mugging is a form asphyxiation when you press the pillow over the head.
B. Gagging is a form of asphyxia when you apply material, usually handkerchief to prevent
air to have access through the mouth or nostrils.
C. Choking is a form of suffocation brought about by the impaction of foreign body in the
respiratory passage.
D. Smothering is a form of asphyxia caused by the closing of external respiratory orifices by
the use of the hands.
76. ln vehicle pedestrian collision, the primary impact is one of the following part of the body,
usually
A. the hips C. the legs
B. the buttocks D. the head
77. Medicaljurisprudence is the legal aspect of medical practice. Exclusion to its scope include:
A. Medical negligence C. Human transplantation
B. Drug dependence D. Medical ethics
78. What is the manner of death when the body ceases to function of its own accord or if there
are mitigating medical factors such as heart disease, terminal illness or the like which would
bring about death
A. Accidental C. Undetermined
B. Natural D. Pending
79. Conduct, whether of action omission, which may be declared and treated as lack of proper
care without any argument or proof as the particular surrounding circumstances, as in the
case of violation of a statute is known as
A. Misfeasance C. Malfeasance
B. Negligence per se D. Nonfeasance
80. Criminal liability as sentenced can be totally erased by the following, except
A. Death of convict before finaljudgemdnt
B. Service of the sentence
C. Amnesty
D. Conditional pardon
8'1 . lf a doctor is guilty of slander, the nature of liability against him would be
A. criminal C. administrative
B. civil D. ethical
82. The following are exception to the hearsay rule, except
A. Dying declaration C. Last will and testament
B. Res gestae D. No exception
83. The approximate time for the completion of one case for DNA testing
A. minimum of eight weeks C. minimum of four weeks
B. minimum of six weeks D. minimum of two weeks
84. The breaking down of the complex proteins into simpler components associated with the
evolution of foul smelling gases and accompanied by the change of color of the body
A. mummification C. putrefaction
B. saponification D. embalming
{lanfinrrad nn Dana 'lO
PHYSICIAN Licensure Examination Page 1O
Sunday, September 10,2017 - 1 1:00 a.m. - 01:00 p.m.
LEGAL MEDICINE. ETHICS AND MEDICAL JURISPRUDENCE SET B

85. The most important internalfindings in death by hanging is


A. Marbolization C. Tete de Negri
B, Tardieu Spots D. Tache Noire'
86. Gunshot wound to the chest is an example of
A. Non natural cause of death
B. Antecedent cause of death
C. Underlying caUse of death
D. lmmediate cause of death
87. The following constitute abandonment, except
A. Failure to follow-uP attention
B. When a colleague refuse to substitute for the physician in the same hospital
C. Refusal by the doctor to give his attention or at least first aid in an emergency
D. Failure to provide substitute when he is about to leave for 1 month
88. lf you are the medico-legal officer and you are supposed to testify in court and refuse to
answer the question which does not involved a criminal case (not covered by privilege
communication), you may be charged with
A. Direct contempt
B. lndirect contempt
C. Obstruction of justice
D. None at all
89. The result of <leception detection with the use of truth serum is not admissive as evidence
because
A. statement is not voluntary
B. medicine use is toxic to the least
C. it does not really compel someone to tell the truth
D. once a liar is always a liar
,

90. Consent of a patient to a specific operation


A. must be in writing
B. may arise by implication from necessity
C. is valid only if the patient is told what might result from an unsuccessful operation
D. must be before two witnesses
91. Cases of rape cannot be dismissed:
A. lf there are no witnesses
B. lf the complainant fails to prosecute
C. lf the offender marries the victim
D. lf the prosecutor fails to prove guilt beyond reasonable doubt
92. A form of sexual perversion (paraphilia) wherein a person focus of erotic interest to eating,
feeding and weight gain
A. Gluttonism C. Mukbang
B. Feederism D. Gobbler
93. Which is classified as mutilation
A. Loss of an eye due to stabbing
B. Lesch-Nyhan syndrome
C. Bilateral tubal ligation
D. lntentionally cutting off the ear
94. When a physician intends to advertise in,the newspaper regarding his practice, what item will
he not state?
A. limitation of practice
B. professional achievements
C. name
D. clinic hours
Continued on Paqe 11
PHYSICIAN Licensure Examination Page 11
Sunday, September 10,2017 - 11:OO a.m.'- O1:OO p.m.
LEGAL MEDICINE, ETHICS AND MEDICAL JURISPRUDENCE SET B

95.Agirl, 13yearsof age whileon her way to herlola was seen bythemale accused and
through cunning words, deceit, cajolery, he succeeded in persuading her to go with him and
enjoyed his carnal lust. What crime was committed?
A. corruption of minors C. consented abduction
B. forced abduction D. act of lasciViousness
96. The willful, deliberate and painless acceleration of death of a person
A. euthanasia C. musketry
B. negligent death D- cremation
97. A medicalwitness may not be compelled to answer questions profound in court in the following
A. lf in so answering, it will subject the physician to punishment for an offense.
B. When the medical witness is asked a question and the opposing counsel interposed an
objection which was sustained by the judge.
C. lf his answer will tend to degrade his character like in cases when such degradation of
character is the very fact at issue.
D. All of the above
98. A jetsetter government official graciously asked her kumareng doktora to issue her a
6-month medical sick leave certificate so' she could legitimately travel around the world
without bureaucratic hassle. How should the kumareng doktora handle this?
A. The kumareng doktora is required to provide medical certificate on request even if these
practice go against her personal values and beliefs.
B. The kumareng doktora can only issue a 1O-day sick leave certificate as required by law.
C. The kumareng doktora need to talk directly to her jetsetter friend emphasizing that
issuing false medical certificate is against the law.
D. All statements are incorrect

99. Some of the characteristics of drug habitation are the following, except
A. There is psychic desire to have the drug.
B. Continuous use will be harmful to the person.
C. There is tendency to increase the dose.
D. Withdrawal symptoms appear when deprived of the drug.
100. The basic weapon of doctors
A. stethoscope
B. neuro hammer
C. caduceus shield
D. knowledge, experience & skills
*** END ***

WARNING: Failure to submit your Test Questions (Complete) set will cause the cancellation
of your Test-Result for this subject.
LEGAL MEDICAL, MEDICAL JURISPRUDENCE & ETH!CS 2017
ANSWER KEY

1. C 26. B 51. C 76. C


2.C 27. C 52. D 77. B
3.D 28. C 53. A 78, B
4.8 29. A 54. D 79. B
5.4 30. c 55. C 80. A
6.4 31. A 56. B 81. C
7.D 32. B 57. C 82. C
8.B 33. A 58. A 83. C
9.C 34. B 59. A 84. C
10. B 35. B 60. A 85. B
11. D 36. D 61. D 86. C
12. B 37. A 62. C 87. B
13. C 38. D 63. C 88. D
14. D 39. D 64. B 89. A
15. A 40. c 65. B 90. B
16. C 41. C 66. D 91. A
17. A 42. A 67. D 92. B
18. D 43. D 68. B 93. D
19. B 44. C 69. D 94. B
20. c 45. D 70. B 95. C
21. B 46. D 71. B 96. A
22. C 47. A 72. A 97. D
23. B 48. D 73. B 98. C
24. C 49. C 74. D 99. C
25. B 50. c 75. A 100. D
Seat No.
Republic of the Philippines
PROFESSIONAL REGUI.ATION COMMISSION
Manila
BOARD OF MEDICINE

PHYSICIAN Licensure Examination


Sunday, September 10, 2017 02:OO p.m.

PATHOLOGY SET B

INSTRUCTION: Select the correct answer for each of the following questions.
Mark onlv one answer for each item by marking the box corresponding to the letter of your choice
on the answer sheet provided. STRICTLY NO ERASURES ALLOWED. Use pencil no. 2 only.
MULTIPLE CHOICE:

1. A 3S-year-old woman complains of nausea, fatigue, and loss of appetite of about one month
duration. Her liver enzymes are slightly elevated. A needle biopsy of her liver is taken and
you diagnose chronic active hepatitis. What is the inflammatory cell type in this liver section?
A. Monocytes
B. Cytotoxic T-lymphocytes
C. Polymorphonuclear leukocytes
D. Mast cell
2. A 28-year-old you tube travel vlogger developed episodes of hemolysis after taking anti-
malarial medication. Numerous Heinz bodies were found in his erythrocytes during these
period. Which type of anemia is closely related to this case?
A. Sickle cellanemia
B. Anemia due to glucose-6-phosphate dehydrogenase deficiency
C. Pernicious anemia
D. Aplastic anemia
3. A 26-year-old man developed recurrent infection with Neisseria gonorrhea. His 3S-year-old
brother also suffered gonococcal infections. A genetic work-up revealed a deficiency in the
C3b component of complement of both men. What functions of the inflammatory response
would be affected in these individual?
A. Chemotaxis
B. Anaphylaxis
C. Vascular permeability
D. Phagocytosis
A 60-year-old man comes to your clinic with a two-month history of coughing up blood. You
suspect tuberculosis and take a sample to identify the microorganism and to determine the
antibiotic sensitivity. You place your patient on standard antibiotic treatment for tuberculosis
while awaiting the laboratory results. However, the man dies seven days after the first clinic
visit. Histopathologic finding shows multinucleated giant cells with nuclei arranged like a
horseshoe (Langhans giant cell) and foreign body giant cell. How would you diagnose these
lesion?
A. Acute inflammation
B. Chronic inflammation
C. Granulomatous inflammation
D. Metaplasia
5. At the time carcinoma of the esophagus is discovered in most patients, the tumor would have
spread beyond its local confines. This early spread is most likely ascribed to:
A. Histological type of the tumor
B. Rich esophageal submucosal lymphatie^s
C. A history of chronic alcoholism
D. A history of chronic tobacco abuse

Continued on Page 2
PHYSICIAN Licensure Examination
I
.i

Sunday, September 1O,2017 - O2:OO p.m. - 04:00 p.m.


PATHOLOGY SET B

6. The cardiac muscle undergoes hypertrophy in response to chronic stressors, such as


untreated hypertension. The mechanism of myocytes hypertrophy involve induction of many
genes that produce cellular growth factors. How would you define hypertrophy?
A. An increase in the size of the cell
B. An increase in the number of cell
C. A shift from one undifferentiated cell type to another undifferentiated cell type
D. A shift from one undifferentiated cell type to another undifferentiated cell type
7. An autopsy is performed on a 6o-year-old man who died suddenly of myocardial infarction.
The man had smoke for 40 years. Sections from his bronchi reveals areas of squamous
differentiation. Based on this information, what tissue changes BEST fit the microscopic
evidence in the epithelium?
A. Hypertrophy
B. Hyperplasia
C. Metaplasia
D. Apoptosis
8. Which of the following is not a hormonal effect stimulated by the actions of androgens?
A. Temporal balding
B. Sebum production
C. Breast budding
D. Epiphysealclosure
9. All of the following will occur during viral infection EXCEPT
A. Macrophages may phagocytose virus
B. CD4+ T-cells respond to viral antigens and MHC Class I molecules to directly attack
virus-infected cells
C. T and B cell areas of the spleen and lymph nodes are involved in the flltration of the
blood and lymph respectively
D. B cells in the presence of helper T-cells and antigen presenting macrophages
differentiate into plasma cells

10. A liver biopsy performed on a patient with acute alcoholic hepatitis mav be expected to show
all of the following EXCEPT
A. Mallory-Denk bodies
B. Lymphocytie infiltrate
C. Macrovesicular fat
D. Ballooning degeneration i

1'1. Within about 30-40 minutes after a myocardial infarction, myocytes undergo irreversible cell
injury. What are the two organelles whose dysfunction is characteristic of irreversible cell
injury?
A. Mitochondria and plasma membrane
B. Endoplasmic reticulum and ribosomes
C. Lysosomes and golgi apparatus
D. Microtubule and centrosome
12. The pigment deposited in the liver and spleen in cases of malaria and schistosomiasis is
chemically closely related to:
A. Ferritin C. Hemosiderin
B. Hematin D. Bilirubin
13. Which one of the following cancer types is correctly matched with its causative etiology?
A. Burkitt's lymphoma Cytomegalovirus
B. Hepatocellular carcinoma Nitrates
C. Urinary bladder carcinoma Anilrne dyes
D. Cervical carcinoma Human T-Cell Leukemia virus

Continued on Page 3
PHYSICIAN Licensure Examination Page 3
Sunday, September 10,2017 - O2:OO p.m. -- O4:OO p.m.
PATHOLOGY SET B

14. The most distinctive feature of acute myeloblastic leukemia is the presence of:
A. Eosinophilic rods in myeloblast
B. Philadelphia chromosomes in neutrophils
C. Blast crisis
D. Virus particle in myeloblast
15. Thyroid hormone secretion is regulated by the action of TSH (Thyrotropin), which act9 bI
stimulating cell shape receptors. Which of the following is most clearly associated with TSH-
like effect?
A. Hashimoto's Thyroiditis C. Graves'Disease
B. Wolff-Chaikoff Effect D. Waterhouse-Friderichsen Syndrome
16. A 44-year-old malevisited an out-patient department with symptoms of nausea, vomiting,
and generalweakness that had developed over the previous 2 weeks . He had history of
medication, nonsteroidal anti-inflammatory drugs. On admission to general ward, his serum
creatinine level was markedly elevated. A renal biopsy was done and 30 mg of corticosteroid
per day was immediately initiated. Supsequently, his serum creatinine level and uremic
symptoms dramatically decreased. Based on the case, expected findings on glomerular
electron microscopic examination will demonstrate:
A. "Spike and dome" appearance with subepithelial deposits
B. "Foot processes" effacement
C. "Tram-track" appearance due to GBM splitting caused by mesangial ingrowth
D. Subepithelial immune complex "humps"
17. A 36-year-old, morbidly obese primigravida presented at the emergency room at 17 weeks
of gestational age complaining of abdominal pain. At clinical examination, the uterus
appeared to be of higher volume compared to the gestational age, the abdomen was
painful but treatable, and the obstetrical examination was normal. The patient was then
referred to the Obstetric Department for further evaluation. The sonographic assessment
revealed the presence of three subserous uterine myomas located on anterior wall, the right
wall, and the left wall of the uterus, respectively. All myomas were vacuolated inside as for
suspected necrosis. The scan also showed other multiple myomas less than 3 cm in
size. Vital signs were monitored (blood pressure 14Ol9O mmHg, maternal heart rate
124 bpm,S 02 94o/o,apyretic). Amniotic fluid was normal and fetal well-being was preserved.
Endometrial biopsy showed uterine fibroids with whorled (fascicular) pattern of smooth
muscle bundles separated by well vascularized connective tissue. The most likely cellular
adaption for this case is:
A. Hypertrophy
B. Atrophy
C. Metaplasia
D. Dysplasia
'18. A 10-year-old boy is seen at the clinic with a chief complaint of shortness of breath.
Exertional dyspnea has been present for the previous month and is associated with
intermittent dry cough. The patient has no associated fever, chills, or chest pain. Chart
review indicates no history of asthma or other pulmonary disease, although the patient has
been seen several times for "hay fever." The patient is accompanied by his mother, who
appears quite anxious. The mother emotionatly relates that her 65-year-old cousin has
recently been diagnosed with mesothelioma and is dying. Furthermore, he had been a
custodian at the patient's school for the previous 3 years, after retiring from his career as a
maintenance worker. His work at the school involved general cleanup and boiler room
maintenance. The mother is afraid thdt her son's dyspnea and cough are related to
asbestos exposure at the school and that he may be developing asbestosis since he often
helped his cousin after school. Question: What is the name of the silicate particulates
where the histopathological findings suggestive of inflammatory asbestosis?
A. Birbeck granules
B. Ferruginous bodies
C. Herring bodies
D. Lewy bodies
Continued on Page 4
PHYSICIAN Licensure Examination Page
Sunday, September 10,2017 - 02:00 p.m. - 04:00 p.m.
PATHqLOGY SET B
19. A49-year-old man presentsto the emergency department complaining that "my skin has
turned yellow". Physical examination reveals the man is significantly jaundiced. He has no
abdominal pain and has a negative Murphy's sign. The physician is concern that he can feel
the patient's gallbladder and orders a CT scan. What is the most likely cause of this patient's
jaundice?
A. Acute hepatitis C. Cholelithiasis
B. Choledocholithiasis D. Pancreatic cancer
20 A 4-year-old child was brought to the Philippine Childrerr's Hospital after his parents started
noticing a left sided renal mass, initially small but however become progressively large and
asymmetrical. Prior the child had hematuria with frank blood and clots intermittently. The child
would cry on passing urine. The most likdly diagnosis is:
A, Renal cell carcinoma
B. Polycystic kidney
C. Nephroblastoma
D. Transitional cell carcinoma
21. The presence of which of the following in the mucosa of an esophageal biopsy is most
diagnostic of reflux esophagitis?
A. PMN C. Basalcell hyperplasia
B. Lymphocytes D. Macrophages (histiocytes)
22. A 4-year-old child presents with a temperature of 40 oC, which she has had for the past
4 days. On physical examination, she is noted to have conjunctivitis', an erythematous
rash, cervical adenopathy, and swollen hands and feet. Laboratory findings include an
absolute neutrophilic leukocytosis, left shift, normal platelets, and an elevated erythrocytes
sedimentation rate (ESR). Which of the following is the most likely diagnosis?
A. Scarlet fever
B. Kawasaki syndrome
C. Acute rheumatic fever
D. Disseminated lupus erythematosus
23. Macrophages and neutrophils both share all of the following characteristics, EXCEPT
A. phagocytosis
B. chemotactic movement
C. receptor for Fc portion of lg G
D. antigen presentation to T cell
24. Adaplor protein that mediate the attachment of integral membrane protein to spectrin in
Hereditary Spherocytosis:
A. Ankyrin C. Actin
B. Fibrillin D. Leptin
25. A 7-year-old boy presents to the physician with acute-onset edema and facial swelling.
Dipstick urinalysis reveals 4+ proteinuria. Renal biopsy shows no appreciable changes under
light and fluorescence microscopy, but electron microscopy demonstrate glomerular
epithelial cell foot process effacement. A diagnosis of minimal change disease is made. How
does this disease affect the pressures governing the flow of fluid across the glomeruli?
A. Bowman's space hydrostatic pressure will be increase
B. Bowman's space oncotic pressure will be decrease
C. Glomerular capillary hydrostatic pressure will be increase
D. Glomerular capillary oncotic pressure will be decrease
26. Latent syphilis is associated with all of the following EXCEPT
A. The presence of clinical lesions
B. A negative dark field examination
C. Normal cerebrospinal fluid
D. Positive non-treponemal serologic test

Continued on Page 5
PHYSICIAN Licensure Examination
Sunday, September 10,2017 - 02:00 p.m. - 04:OO p.m'
PATHOLOGY SET B

27. Which of the following is least likely related to a malignancy?


A. Hepatoma C. Myoma
B. Sarcoma D. Seminoma
28. A72-year-old government official known to be a heavy smoker developed gangrene of the
leg, which had to be amputated. lntraluminalthrombi associated with microabscess in the
wall of medium sized arteries were found in the resected leg. What is the most probable
vasculitis disorder in this case?
A. Polyarteritis nodosa
B. Takayasu arteritis
C. Thromboangiitis obliterans
D. Henoch-Schonlein purpura

29. Combined hypertrophy and hyperplasia are most likely to occur in:
A. Post-menstrual endometrium
B. Skeletal muscle of an athlete who "pumps iron"
C. Left ventricular myocardium in a patient with aortic stenosis
D. Enlarged prostate gland
30. A 65-year-old man with a 40 pack-year history of smoking presents with a 7 Kg weight loss
over the last 3 months and recent onset of streak of blood in the sputum. Physical
examination reveals a thin, afebrile man with clubbing of the fingers, an increased
anteroposterior diameter, scattered coarse rhonchi and wheezes over both lung fields, and
distant heart sounds. A chest x-ray exhibit left hilar adenopathy, dilated tubular markings,
and flattened diaphragms. A sputum cytology using Papanicolaou's stain shows numerous
cells with deeply eosinophilic - staining cytoplasm and irregular, hyperchromic nuclei
intermixed with inflammatory cells. Which of the following is the most likely diagnosis?
A. Tuberculosis
B. Pulmonary embolism with infarction
C. Bronchiectasis
D. Squamous cell carcinoma of the lung
3'1. The primary antigen presenting cells in the skin are known as:
A. Keratinocytes C. Langerhan's cell
B. Mast cells D. Melanocytes
32. High-power photomicrograph demonstrate shadowy outlines of dead fat cells. There is
a bluish cast from the calcium deposits which are basophilic. Gross pathology shows
chalky, white areas from the combination of the newly-formed free fatty acids with calcium
(saponification). This pancreas shows sign of:
A. Coagulation necrosis
B. Liquefaction necrosis
C. Caseation necrosis
D. Enzymatic fat necrosis
33. A 7-year-old Nigerian boy was presented with several month history of jaw swelling which
has been treated with antibiotics. The tumor was ulcerated and draining. Examination of the
mouth of this patient with Burkitt lymphoma shows disruption of teeth and partial obstruction
of airway. What do you think is the clinicalvariant associated with the above case?
A. Endemic Variant of Burkitt lymphoma
B. Sporadic Type of Burkitt lymphoma
C. lmmunodeficiency-Associated Burkitt lymphoma
D. Non-African Type of Burkitt lymphoma
34. Herpes simplex virus type 1 most commonly causes cold sores. The site of reactivation for
this virus is the:
A. B lymphocyte C. Epidermal cell
B. Vagus nerye D. T:ioeminal nerve

Confinuad nn Paoc 6
PHYSICIAN Licensure Examination Page 6o
Sunday, September 10,2017 - O2:OO p.m. - O4:00 p.m.
PATHOLOGY SET B

35. When 3 o-globin become inactive because of deletional mutations, the affected individual
woutd have only one functional o-globin Srene. These people usually have moderate anemia
or marked microcytosis and hypochromia. This alpha thalassemia hereditary disorder is
called:
A. o-Thalassemia silent
B. o-Thalassemia trait
C. Hemoglobin H disease
D. Hydrops fetalis
36. A malnourished child was brought to a lying-in clinic because of breathing difficulties. He
was the seventh child born to a couple in Tondo, Manila. The father delivered the baby at
home, which the parents did not weigh at birth. lnitially, the mother breast-fed the baby until
nipple bleeding forced her to stop at 2 Yz months then shifted to formula milk. ln the course
of feeding, the patient's parents believed theirchild had a milk allergy and consequently
restricted his daily intake. Dietary history revealed a prolonged, inadvertent administration
of a restricted diei, deficient in protein and severat other nutrients. Consequently, the child
with his life-threatening condition died 5 hours later and ruled out as child neglect by the
DSWD representative] ln the autopsy, the pathologist found no evidence of dehydration on
the ocular chemistries and ruled that, "rnalnutrition" was the immediate and underlying
cause of death. Question: What is the wear and tear pigment that has been inculpated in
cases of malnutrition?
A. Lipofuscin
B. Ceroicl
C. Hemosiderin
D. Melanin
37. Exercise acts as a trigger for sudden cardiac death in people with cardiovascular disease.
The risk for sudden death in young athletes with cardiovascular disease is 2.5 times higher
than that in non-athletes. Greater than 9O% of sudden cardiac death occurs during
or immediately after a training session or competition. Sudden cardiac death (SCD) in
an athlete is a rare, but tragic event. Sport r.elated SCD was defined as non-traumatic
SCD during or within t hour after moderate- to high-intensity exercise in a competitive
athlete. The deceased was considered a competitive athlete if he or she did physically
demanding sports and took part in competitions. Question: What is the single most
common cardiovascular cause of sudden cardiac death (SCD) in young athletes?
A. Congestive cardiomyopathy
B. Hypertrophic cardiomyopathy
C. Obliterative cardiomyopathy
D. lschemic cardiomyopathy
38. An 11-year old malefrom Manila presented to the Emergency Department of Philippine
General Hospital with history of multiple episodes of generalized tonic-clonic seizure for the
last 10 days. ln past, he had multiple hospital admission for the same reason and was
on antiepileptic drug since the age of one year with poor control. He was born of non-
consanguineous marriage with uneventful birth history. There was no history of seizure in
family members; however his father had skin nodules over the face and neck along with a
hypopigmented macule over the trunk. On detailed examination, the child had multiple
hyper-pigmented papules over the nasolabial region (adenoma sebaceum). He also had
multiple (five) hypo - pigmented macules (ash leaf) over the lower limbs along with a
Shagreen patch over the lateral aspect of the left buttock. Detailed CNS examination
revealed increased tone of both upper and lower limbs with brisk tendon reflexes and
bilateral positive Babinski's sign but absent superficial reflexes. Other systemic examination
and fundus examination revealed no abnormality. This condition is called:
A. Von Recklinghausen's disease
B. Tuberous sclerosis
C. Marfan's syndrome
D. Von-Hippel-Lindau

Continued on Page 7
PHYSICIAN Licensure Examination Page 7
Sunday, September 10,2017 - 02:00 p.m. '- O4:00 p.m.
PATHOLOGY SET B

39. Edema due to the following pathogenic mechanism is correctly matched with each clinical
setting, EXCEPT
A. lntrease capillary hydrostatic pressure - Malignancy
B. Decrease plasma oncotic pressure Nephrogenic syndrome
C. lncrease vascular permeability Trauma and lnflammation
D. Lymphatic blockage Elephantiasis

40. A Z2-year-old man presenttothe physicianwith a 1-year history of chronic recurrent right
lower quadrant abdominal pain and diarrhea. The patient also has had low-grade fevers and
a 6.7 Kg weight loss during this period. Colonoscopy reveals multiple lesions of the terminal
ileum and colon. Biopsies of these lesions revealed inflammation from the mucosa to the
serosa with the presence of non-caseating granulomas. Which of the following is the most
likely diagnosis?
A. Celiac sprue
B. Crohn's disease
C. lrritable bowel syndrome
D. Ulcerative colitis
41. The most common renal lesion in lead poisoning is:
A. Proliferative GN
B. Acute tubular necrosis
C. Membranous GN
D. Tubulointerstitial nephritis
42. A predominant factor in the formation of ascites in cirrhosis is:
A. Peripheral vasoconstriction
B. High serum oncotic pressure
C. Peritoneal inflammation
D. Renal sodium avidity

43. The number of barr bodies in Turner syndrome:


A. None
B. One
C. Two
D. Three
44. Which of the following macrophages is incorrectly matched with the appropriate tissue?
A. Monocyte Bone marrow
B. Epitheliod cell - Connective tissue
C. Hofbauer cell - Placenta
D. Kupffer cell - Liver

45. A 4-year-old boy with a 3-week history of fevers and lack of energy is found to have severe
anemia, moderate thrombocytopenia, and a wbc of 12,OOO c,ells/mm3 with 3O7o blasts, 407o
lymphs, 2Ook polys, looh monocytes. Physical exam reveals no lymphadenopathy. CXR is
normal. A bone marrow biopsy would most likely reveal:
A. Acute lymphoblastic leukemia (ALL)
B. Acute myeloblastic leukemia (AML)
C. Chronic lymphocytic leukemia (CLL)
D. Chronic myeloid leukemia (CML)
46. The presence of antibodies to which of the following antigen is most highly specific for
systemic lupus erythematosus (SLE)?
A. Anti-SS-A antibody
B. Anti-Centromere antibody
C. Anti-dsDNA antibody
D. Anti-GBM antibody

Continued on Page I
PHYSICIAN Licensure Examination Page I
Sunday, September 10,2017 - O2:OO p.m. - O4:OO p.m.
PATHOLOGY SET B

47. Examination of peripheral blood smears may identify RBC abnormalities that may trigger a
physician to consider a particular diagnosis. Which association is INCORRECT?
A. Schistocytes ) DIC
B. Rouleaux ) Cold agglutinin disease
C. Howell-Jolly bodies ) Splenectomy
D. Basophilic stippling ) Lead poisoning
48. The most characteristic feature of rapidly progressive g1!omerulonephritis:
A. Hyaline nodule
B. Crescent formation
C. Subepithelial dense deposits
D. Thickened capillary loops

49. The most common primary malignant tumor of the ovary:


A. Endometroid carcinoma
B. Mucinous cystadenocarcinoma
C. Serous cystadenocarcinoma ;

D. Yolk sac tumor


50. The most characteristic microscopic finding of Retinoblastoma is:
A. Homer Wright rosette
B. Flexner Wintersteiner rosette
C. Blastema cell
D. Rosenthalfibers
51 . What is the main cause of Acute Tubular Necrosis other than ischemic injury?
A. Medication C. Hypertension
B. Vasculitis D. Systemic lupus erythematosus
52. MacCallum plaques is a condition featuring the thickening of posterior part of:
A. Left ventricle
B. Right ventricle
C. Left atrium
D. Right atrium

53. Heterophile antibody positive infectious mononucleosis:


A. Varicella-Zoster virus C. Cytomegalovirus
B. Herpes simplex virus D. Epstein-Barr virus
54. Which of the following chromosomal translocations would you expect to find in patient with
Acute Promyelocytic Leukemia?
A. r @;22)
B. t (8;14)
C. t (15;17)
D. t (11;14)
55. Routine gynecological cytology revealed a ClN, Grade 3 (severe dysplasia, carcinoma-in-
situ) in a 27-year-old woman. Salient features of the biopsy shows carcinoma-in-situ extend
downwards into glandular structures. The lumen of the cervical canal is at the top. The
epithelium has migrated into an underlying branching gland of the cervix displacing the
columnar epithelium and practically filling the lumen of the glands. The basement membrane
is intact. Question: Significant risk factors for developing this lesion include all of the following
EXCEPT
A. History of prostitution
B. Lack of circumcision of male sex partners
C. Early age at first sexual intercourse
D,Historyofpenilecondylomainmalesexpartners

Continued on Page 9
PHYSICIAN Licensure Examination Page 9
Sunday, September 10,2017 - O2:OO p.m. - 04:00 p'm.
PATHOLOGY SET B

56. A 25-year-old woman presents to her physician after finding a large lumpin the right side of
her neck which has been present for the past month. She also complains of fevers, frequent
drenching night sweats, and a 10 pound weight loss over the past 4 months. On exam, she
is found to have an enlarged supraclavicular lymph node and enlarged nodes in the left
lower posterior cervical chain and axillary regions. There is no evidence of hepato or
splenomegaly. Lymph node biopsy reveals the presence of Reed-Sternberg cells with
lacunae surrounding the nuclei of these cells. Which of the following variants of Hodgkin
disease is most likely present?
A. Lymphocyte-Depleted
B. Lymphocyte Predominance
C. Mixed Cellularity
D. Nodular Sclerosis

57. A deeply infiltration ulcer which slowly destroys bones and soft tissue:
A. Curling ulcer C. Rodent ulcer
B. Decubitus ulcer D. Stercoral ulcer
58. Acidophilic (Councilman's) bodies can be expected to be found most frequently in:
A. Alcoholic hepatitis C. Micronodular cirrhosis
B. Simple cholestasis D. Acute viral hepatitis
59. The presence of elevated alpha feto-pi'rtein in the serum strongly suggest the following
conditions, EXCEPT
A. Neuraltube defect
B. Neuroendocrine tumor
C. Yolk sac tumor
D. Hepatocellular carcinoma
60. A 55-year old woman with hypercalcemia discovered as an incidentalfinding during a normal
routine physical examination (including pelvic and breast examination) most likely has which
of the following?
A. Sarcoidosis
B. A history of taking thiazide diuretics
C. Metastatic breast cancer
D. A benign parathyroid adenoma
61. Exposure to substance known to be associated with an increased incidence of lung cancer
includes the following EXCEPT
A. Asbestos
B. Sulfur dioxide
C. Uranium
D. Bys(Chloromethyl) ether
62. The principal cause of multi-organ dysfunction syndrome associated with septic shock is the
following?
A. Direct invasion multiple organs during gram-negative bacteremia
B. The release of cytotoxic exotoxins by gram-negative bacteremia
C. lnduction of pro-inflammatory cytokines (TNF, lL-1 and lL-6)
D. Drug induced toxicity during therapy for gram negative bacteremia
63. Which of the following presents in the first or second decade of life with multiple neuromatous
mucosal nodules and a distorted body habitus?
A. MEN type I (Wermer)
B. MEN type ll (Sipple's syndrome)
C. MEN type lll
D. Zollinger-Ellison syndrome

Continued on Page 10
PHYSICIAN Licensure Examination Page 1O
Sunday, September 1O,2017 - O2:OO p.m. '- 04:00
PATHOLOGY SET B

64. Due to a lowering of the surface tension by severe proteinuria, nephrotic patient's urine is
usually manifested as:
A. Ammoniacal urine C. Tea colored urine
B. Fruity urine D. Foamy urine
65. A 78 - year - old man with disseminatecl carcinoma treated with chemotherapy died of
pulmonlry complications. Gross examination of the lungs showed areas of consolidation.
The finaldiagnosis of Aspergillosis was made at autopsy. Question: Which of the following
lesions LEAST likely to be e-ncountered in the lungs infected by this fungus?
A, Proliferation with preexisting pulmonary cavities - "fungus ball"
B. Granulomatous inflammation
C. Necrotizing pneumonia
D. Focal hemorrhagic necrosis
66. A 1g-year-old man comes to the emergency department because of blood in his sputum.On
history, the patient mentions he has had weight loss and night sweats. On examination, the
patient nas i fever and bronchial breath sounds with crepitant rales. Laboratory tests show
iymphocytosis and an increased erythrocytes sedimentation rate. X-r9y film of the chest
dno'ws a calcified lung lesion and hilar lynrphadenopathy. Which of the following is the stain
used to identify the most likely infectious organism?
A. Congo Red C. Periodic Acid-Schiff
B. Giemsa's Stain D. Ziehl-Neelsen
67. The tissue that is most resistant to invasion by cancer cells
A. Bone
B. Heart
C. Cartilage
D. Spleen
68. A tall, lanky 13-year-old boy presents to the ophthalmologist with a sudden change in vision.
Examination shows lens subluxation. The patient has always been tall and thin for his age.
and a family history reveals that his uncle died from a ruptuied aortic aneurysm. On physical
examination, the patient is found to have long and slender fingers, ligament laxity, and
abnormal ratio of arm length to height, alrd a diastolic murmur over the aortic area. Which of
the following is most likely deficient in this'patient?
A. Dystrophin
B. Elastase
C. Fibrillin
D. Type I collagen
69. The most frequent finding in the adrenal gland of patients with Addison disease is:
A. Atrophy C. Metastatic carcinoma
B. lnfarction D. Tuberculosis
70. Which of the following innate immune cell types plays a direct and important role in controlling
the early stages of the systemic response to EBV infection?
A. Mast cells
B. Natural killer cells
C. Plasma cell
D. Regulatory T lymphocytes
71. A patient who has had an aortic valve replacement is on therapeutic doses of Warfarin
(Coumadin). She is most likely to have a normal level of which faclor?
A. Factor Vll C. Factor V
B. Protein C D. Factor X

Continued on Page 1'1


PHYSICIAN Licensure Examination Page 11
Sunday, September 10,2017 - O2:00 p.m, - 04:00 p.m.
PATHOLOGY SET B

72. Alpha-1 antitrypsin deficiency with enlargement of airspaces is associated with:


A. Panlobular emphysema
B. Anthracosis
C. Wegener's granulomatosis
D. Tuberculosis
73. True statements about malignant mesothelioma include:
A. Exposure to cigarette smoke and to asbestos combined increase the risk of developing
mesothelioma.
B. It may be confused histologically with carcinomas or adenocarcinoma
c. ln its early stages it may be cured by surgical resection
D. It is generally very responsive to chemotherapy

74. A2-year-old boy presents with recurrent and severe paroxysmal colicky pain accompanied
by straining effort, loud cries and vomiting. A stoolwith red blood mixed with mucus is passed.
An oblong mass is palpated in the midepigastrum. Which of the following is the most likely
diagnosis?
A. Meckel's diverticulum
B. Congenital pyloric stenosis
C. lntussusception
D. Meconium ileus
4
75. Lesions mistaken clinically as malignant are the following, EXCEPT
A. Acral lentiginous
C. Papilloma
D. Hemangioma
76. Hepatocarcinoma most commonly develops in a liver which is the site of:
A. Aflatoxin toxicity
B. Cirrhosis
C. Clonorchis sinensis infestation
D. Hemochromatosis
77. A hemartomatous polyp associated with a syndrome that increase risk of developing cancer
in the pancreas, breast or lung:
A. Hyperplastic polyp
B. Peutz-Jeghers polyp
C. Tubular adenoma
D. Juvenile polyp
78. Also known as stone-mason disease or grinder's dise:rse:
A. Silicosis C. Byssinosis
B. Siderosis D. Anthracosis
79. Which lesion is most likely to be associated with breast malignancy?
A. Phyllodes Tumors
B. Paget's Disease of the Breast
C. Mammary Duct Ectasia
D. lntraductal Papilloma
80. Which of the following primary malignancies does not commonly metastasize to bone?
A. Thyroid
B. Prostate
C. Lung
D. Colon

Continued on Page 12
PHYSICIAN Licensure Examination Page 12
Sunday, September 10,2017 - O2:OO p.m. - O4:OO p.m'
PATHOLOGY SET B

81. Carcinoma in situ (ClS) refers to neoplastic cells that have not yet invaded the basement
membrane. ln many cases, CIS can be sr.lrgically removed providing curative treatment. An
invasive carcinoma occurs when a CIS invades through the basement membrane. The
aberrant expression of what factor could allow the neoplastic cells to invade through the
basement membrane?
A. Keratin C. o-Fetoprotein
B. Collagenase D. Mucin
82. A 3$-year-old man comes to his physician because he began experiencing flu-like symptoms
and an expanding rash on his leg following a camping trip '1 week ago. The physician
diagnoses him with Lyme disease and prescribe Tetracycline. The man finds some
Tetracycline in his bathroom from about 6 years ago and decides to use this instead of buying
a new bottle. One week later, the man presents to the emergency department and is
diagnosed with which of the following renal conditions?
A. Acute tubular necrosis
B. Glomerulonephritis
C. Renal papillary necrosis
D. Renal tubular dysfunction
83. "Starry sky" pattern of lymph node is characteristic of:
A. Burkitt's lymphoma C. Histiocytic lymphoma
B. Hodgkin's lymphoma D. Sezary's syndrome
84. An 33-year-old man presents to the cliric with gross hematuria. About 5 days ago, he had
a fever, runny nose, and sore throat. He is currently asymptomatic. There is no family
history of hematuria or renal failure. Physical exam and vital signs are normal. Urinalysis
demonstrate dysmorphic RBC's, RBC's cast, and trace proteinuria. Serum chemistries are
normal. Serum complement and other serologies are normal. Hematuria persist 1 week later.
What is the most likely diagnosis:
B. Minimal change disease
C. Membranoproliferative glomerulonephritis
D. Post-streptococcal glomerulonephritis
85. Which of the following is associated with cor pulmonale:
A. Mitral stenosis C. COPD
B. Pericarditis D. Myocarditis
86. This lesion is often seen in chronic alcoholics often caused by severe retching:
A. Esophageal metaplasia (Barrett esophagus)
B. Esophageal squamous cell carcinoma
C. Esophageal stricture
D. Esophageal laceration (Mallory-Weis syndrome)

87. A 4O-year-old non-smoking woman comes to the physician complaining of a persistent cough
and loss of weight and appetite over the past 3 months. On radiography, a well-demarcated
subpleuralmass is found on the right lung. Which of the following types of carcinoma does
this patient most likely have?
A. Adenocarcinoma of the lung
B. Carcinoid of the lung
C. Small-cell carcinoma of the lung
D. Squamous cell carcinoma of the lung
88. The pericardium of patients with acute rheumatic carditis may be expected to reveal:
A. Serous inflammatiorl r

B. Granulomatous inflammation
C. Suppurative inflammation
D. Fibrinous inflammation ,

Continued on Page 13
PHYSICIAN Licensure Examination 13
Sunday, September 10,2017 - O2:OO p.m. - O4:OO p.m.
PATHOLOGY SET B

89. Which lobe of the prostate is cancer most commonly found?


A. Anterior lobe
B. Posterior lobe
C. Lateral lobe
D. Median lobe
90. The type of bone neoplasm which arises in diaphysis of long bones:
A. Osteosarcoma C. Ewing's sarcoma
B. Giant cell tumor D. Metastatic carcinoma
91. Sputum cytology is most likely to be positive for cancer cells in a patient with:
A. Left upper lobe atelectasis
B. Solitary pulmonary nodules
C. Adenocarcinoma
D. Lymphoma
92. Which is not a c€luse of eosinophilic leukocytosis?
A. Allergic disorder
B. Parasitism
C. Viral infection
D. Drug reaction
93. A 3S-year-old woman visits her physician atter feeling a hard lump in her neck. Her physician
notes that she has a single, hard, non-tender nodule in the left lobe of her thyroid that
moves when she swallows. There is no cervical lymphadenopathy. The patient denies any
changes in her health. The patient does not have tremor, restlessness, heat intolerance, or
an increased level of anxiety. Blood test show normal thyroid hormone and calcitonin levels.
A scintiscan shows a cold nodule in the left lobe of her thyroid. Tissue is obtained and a
histological section shows branching papillae with a fibrovascular stalk. These papillae are
lined by epithelial cells with empty-looking, ground glass nuclei often called "Orphan Annie
eye" nuclei. Concentrically calcified Psammoma bodies are also seen. Which of the following
types of tumors would most likely result in these findings?
A. Follicular carcinoma
B. Medullary carcinoma
C. Multinodufar goiter
D. Papillary carcinoma
94. A 3S-year-old man comes to the physician complaining of painful genital vesicles. On further
questioning, he admits to unprotected sex with multiple partners. To confirm the diagnosis,
the physician performs a Tzanck test. Which of the following is the pathognomonic finding on
this patient's Tzanck smear?
A. Call-Exner bodies
B. Lewy bodies
C. Mallory bodies
D. Multinucleated giant cell
95. Dysplastic nevus is to melanoma as actinic keratosis is to:
A. Basalcellcarcinoma
B. Kaposi's sarcoma
C. Squamous cell carcinoma
D. Lentigo maligna

96. The pteural effusion associated with the following is an exudate EXCEPT
A. Congestive heart failure
B. Pulmonary tuberculosis
C. Metastatic carcinoma
D. Mesothelioma

Continued on Page 14
PHYSICIAN Licensure Examination Page 14
Sunday, September 1O,2O17 - 02:00 p.m. - O4:00 p.m.
PATHOLOGY SET B
.

97. Which of the following types of hepatocellular injury is commonly seen after acetaminophen
overdose?
A. Acute hepatitis
B. Centrilobular necrosis i

C. Granuloma formation
D. Microvascular fatty change
99. A 3S-year-old man who works at a facility processing highly radioactive substances
accidentally receives a high, whole - body dose of ionizing radiation estimated to be
1,500 rads. He dies 1 week later. At autopsy, histologic examination of the skin shows
scattered, individualepidermal cells with shrunken, markedly eosinophilic cytoplasm and
pyknotic, fragmented nuclei. These morphologic changes most likely indicate which of the
following processes?
A. Apoptosis
B. Coagulation necrosis
C. Liquefaction necrosis
D. Tumor initiation i

1OO. "Blushing" is a localized hemodynamic dysfunction of:


A. Activehyperemia
B, Acute Passive hyperemia
C. Chronic Passive congestion
D. Hypovolemia

*i* END i**

WARNING: Failure to submityour TestQuJstions(Complete)set will cause the cancellation


of your Test-Result for this subject.
PATHOLOGY 2017
ANSWER KEY

1. B 26. A 51. A 76. B


2.8 27.C , 52. C 77_B
3.D 28. C 53. C 78. A
4.C 29. D 54. C 79. B
5.B 30. D 55. B 80. D
6.A 31. C 56. D 81. B
7.C 32. D 57. C 82. A
LC 33. A 58. D 83. A
9.8 34. D 59. B 84. A
10. B 35. C 60. D 85. C
11. A 36. A 61. B 86. D
12. B 37. B 62. C 87. A
13. C 38. B 63. C 88. D
14. A 39. A 64. D 89. B
15. C 40. B 65. B 90. c
16. A 41. D 66. D 91. A
17. A 42. D 67. C 92. C
18. B 43. A 68. C 93. D
19. D 44. B 69. A 94. D
20. c 45. A 70. B 95. C
21. A 46. C 71. C 96. A
22. B 47. B 72. A 97. B
23. D 48. B 73. B 98. A
24. A 49. C 74. C 99. A
25. D 50. B 75. A 100. A
Seat No, ;'
Republic of the Philippines :

PROFESSIONAL REGUIATION COMMlSSION


Manila
BOARD OF MEDICINE

PHYSICIAN Licensure Examination


Saturday, September 16, 2017
PHARMACOLOGY & THERAPEUTICS SET A

INSTRUCTION: Select the correct answer for each of the following questions.
Mark onlv one answer for each item by marking the box corresponding to the letter of your choice
on the answer sheet provided. STRICTLY NO ERASURES ALLOWED. Use pencil no. 2 only.
MULTIPLE CHOICE:
1. A 19-year-old man presentswith edematous, pruritic papules on his abdomen and chest.
The lesions appeared shortly after eating a shellfish meal at a restaurant. He also complains
of difficulty breathing and swallowing and has wheezing on examination. What is the most
appropriate treatment for this patient?
A.'Benadryl
B. Topical glucocorticoids
C. Systemic glucocorticoids
D. lntramuscular (lM) epinephrine

2. A 21-year-old sexually active college student presents to the clinic complaining of odorous
green vaginal discharge and itchiness. Multiple oval flagellated motile organisms are seen on
wet mount. She is prescribed an antibiotic treatment. While on therapy, she goes to a party,
where on consumption of one alcoholic beverage she experiences flushing, tachycardia,
headaches, and vomiting. The same effects could be observed when alcohol is mixed with
which drug?
A. Ampicillin C. lmipenem
B. Glipizide D. Tolbutamide

3. Rhabdomyolysis, a rare muscle condition that can have a serious impact on kidney function,
has been reported as an outcome of a hazing ritual in which a college student was beaten
hundreds of times with paddles. These severe form of myotoxicity can also occur in patients
who took two common drugs namely:
A. Hydrochorothiazide to reduce extra fluids and Losartan to lower blood pressure
B. Azithromycin for infection and Simvastatin to control cholesterol
C. Clozapine to balance mood behavior and Famoticline to decrease gastric acid secretion
D. Loperamide to treat diarrhea and Cetirizine to relieve allergy'
4. A great deal of research in the past decade had focused on developing specific inhibitors of
the cyclooxygenase pathway of arachidonic acid metabolism. Celebrex, for.example, is a
specific inhibitor of COX-2. What drugs inhibit COX-3?
A. lbuprofen C. lndomethacin
B. Acetaminophen D. Aspirin

5. Which of the following medications/drug is NOT associated with gynecomastia?


A. Spironolactone C. Dihydrotestosterone
B. Tetrahydro-cannibinol D. Ketoconazole
6. A S9-year-old woman had an ischemic stroke 5 years ago, and is on Clopidogrel (Plavix) to
decrease her risk of subsequent ischemic events. This drug functions as an anti-platelet
drug via which mechanism? lt:
A. lnhibits cyclooxygenase.
B. lnhibits the Gpllb/llla reaction
C. lnhibit the ADP reaction

Continued on Page 2
PHYSICIAN Licensure Examination Page 2
Saturday, September 16,2017 - 08:00 a.m. -: 1O:00 a.m.
PHARMACOLOGY & TH ERAPEUTICS SET A

7. A 38-year-old man comes to the physician because of a 1-month history of cough and 4.5 Kg
weight loss. Physical examination shows no abnormalities. A chest x-ray shows a right upper
lobe infiltrate. One of three sputum samples is positive for acid fact bacilli. Treatment with
isoniazid, rifampin, ethambutol, and pyrazinamide is started. Which of the following should
be added to the medication regimen to prevent neurologic toxicity in this patient?
A. Folic acid
B. Nicotinic acid
C. Vitamin Bo (pyridoxine)
D. Vitamin Brz (cyanocobalamin)

8. Acetaminophen is one of the most common agents deliberately ingested for fever and pain.
Which of the following is a disadvantage of acetaminophen?
A. The risk of gastrointestinal ulcers
B. The risk of bleeding from an antiplatelet effect
C. The lack anti-inflammatory activity
D. The risk of hypersensitivity reaction

9. Which of the following is used only as an acute flare therapy and not as a prophylactic agent
in gout? '
A. Probenecid
B. High-dose aspirin
C. Colchicine
D. Allopurinol

10. All of the following are true regarding Plavix (Clopidogrel), EXCEPT?
A. lt inhibits the binding of adenosine diphosphate to its platelet receptor
B. The risk of bleedihg is increased if it is used with aspirin
C. When discontinued, bleeding time normalizes in approximately 2-3 days
D. lt inhibits platelet aggregation within 2 hours of oral administration
'1 1. Federal Aviation Administration (FAA) is recommending that pilots not take this drug within
six hours of flying because the side effects could make it tough for pilots to distinguish
between the blues and greens found in the cockpit instrument and runway lights. What
prescription medication is this?
A. Digoxin
B. Sildenafil
C. Tetracycline
D. Ethambutol

12. Which of thefollowing should be given to promote wound healing in patients receiving
steroids?
A. Vitamin A
B. Vitamin 81
C. Vitamin El2
D. Vitamin C
13. Which of the following may help pr€ve or dtotoxicity in a patient receiving vancomycin?
A. N-acetylcysteine
B. Alpha-tocopherol
C. Ferrous sulfate
D. Thalidomide
14. Nonsteroidal anti-inflammatorydrugs are commonly used for post-operative pain relief. All
of the following are potentially complications of non-steroidal use, EXCEPT?
A. Colonic ileus
B. Gastric ulceration
C. Platelet inhibition
D. Renal damage

Continued on Page 3
PHYSICIAN Licensure Examination Page 3
Saturday, September 16,2017 - 08:00 a.m. - 1O:00 a.m.
PHARMACOLOGY & TH ERAPEUTICS SET A

15. A 4S-year-old male homeless chronic alcoholic is brought to the Emergency Room because
of abdominal pain. He undergoes Laparotomy and omental patch of a perforated gastric
ulcer. On post-operative day two in the lntensive Care Unit, he develops tachycardia,
hypertension and agitation. This could be treated with all of the foltowing drugs, EXCEPT?
A. Benzodiazepine
B. Fentanyl
C. Proprofol
D. Haldol
16. All of the following antibiotic classes are potentially nephrotoxic, EXCEPT?
A. Aminoglycosides
B. Metronidazole
C. Penicillin
D. Cephalosporins

17. The'little pink pill" (female viagra) is classified under:


A. Anti-depressant drugs
B. Anti-epileptic drugs
C. Hormone therapy drugs
D. Drugs for dyspareunia
'lB. Which best describes the action of Corticosteroids?
A. t Muscle protein degradation, I Muscle protein synthesis,1 Glucose utilization, Anabolic
B. ] Muscle protein degradation, j Muscl6 protein synthesis, j Glucose utilization, Catabolic
C. I Muscle protein degradation, t Muscle protein synthesis, J Glucose utilization, Anabolic
D. J Muscle protein degradation, J Muscle protein synthesis, J Glucose utilization, Catabolic

19. Glucose-6-phosphate dehydrogenase deficiency predisposes to hemolytic anemia when an


affected individual receives the following medications, except for
A. Primaquine C. Trimethoprim
B. Mafenide D. Dapsone
20. Which of the following groups of diuretics has been found to be useful in the treatment of
glaucoma?
A. Osmotic diuretic
B. Carbonic anhydrase inhibitors
C. Thiazide diuretic
D. Aldosterone antagonists

21. Which of the following drugs incorrectly matched with the enzyme it inhibits?
A. Acetazolamide - Carbonic anhydrase
B. Allopurinol - Xanthine oxidase
C. Pyridostigmine - Cholinesterase
D. Theophylline - Monoamine oxidase

22. Which of the following opiods, when given in high dosages, may cause hyperflexia,
myocolon us, seizu res and anticholinerg ic effects?
A. Morphine
B, Demerol
C. Codeine
D. Fentanyl

23. Which of the following diuretics is contraindicated in the presence of hyperkalemia?


A. Acetazolamide
B. Furosemide
C. Ethacrynic acid
D. Spironolactone

Continued on Page 4
PHYSICIAN Licensure Examination
Saturday, September 16,2017 - 08:0O a.m. - 10:OO a.m.
PHARMACOLOGY & TH ERAPEI.JTICS SET A

24. The predominant mechanism of plasmid-dependent resistance of bacteria to aminoglycosides


A. Production of high levels of acetylating and phosphorylating enzyme
B. Alteration in penicillin-binding sites
C. Alteration of intracellular target site (ribosome)
D. Lack of oxidative metabolism and active transport
25. Furosemide (Lasix) is commonly used to treat fluid overload. All of the following are
mechanisms of action of Furosemide, EXCEPT?
A. lnhibition of active sodium absorption in the ascending loop of Henle
B. lncreased blood flow to the kidney
C. Aug mented catecholamine susceptibility to alpha-ad renerg ic receptors
D. lncreased venous capacitance
26. A 5O year old man has been in the ICU for three weeks following repair of a ruptured
abdominal aortic aneurysm. He develops fever, hypotension, and a new heart murmur.
Blood cultures grow vancomycin - resistant enterococcus. The best antibiotic treatment for
this condition is?
A. Azithromycin C. Levofloxacin
B. Linezolid D. Ciprofloxacin
27. Cough suppressant as powerful as codeine with few side effects and no addicting power:
A. Dextromethorphan
B. Guaifenesin
C. Levodropropizine
D. Camphor
28. A 33-year-old woman with a history of Grave's disease is brought to the emergency
department unresponsive following a bout of confusion and agitation. On physical
examination hertemperature is 39.2oC, her blood pressure is 100/7Omm Hg, and her
pulse is 65/min. A systolic ejection murmrlr is heard at the apex, and the patient has 3+
pitting edema at the ankles. Following primary stabilization, which of the following would
constitute appropriate pharmacotherapy for this patient's underlying condition?
A. Aspirin
B. Dobutamine
C. Levothyroxine
D. Propylthiouracil
29. A 68-year-old female theater patron was interrupting the operatic performances with her
intermittent coughing. What anti-hypertensive medication is she taking?
A. Beta blockers
B. Ace lnhibitors
C. Calcium antagonists
D. Angiotensin ll antagonists
30. Floppy iris syndrome is a variant of the small pupil syndrome characterized by a flaccid iris
which billows in response to ordinary intraocular fluid currents. lt has been observed during
cataract surgery in some patients currently or previously treated with:
A. Alpha -1 blockers
B. Alpha -2 blockers
C. Beta-1blockers
D, Beta-2blockers
31. Which of the following cytochrome P45O isoen 4meis involved in the metabolism of largest
number of drugs in human beings and has been implicated in some dangerous drug
interaction?
A. (J^YP 1A2 c. cYP 281
B. CYP 2C9 D, CYP 344

Continued on Page 5
PHYSICIAN Licensure Examination Page 5
Saturday, September 16,2017 - OB:00 a.m. - 10:00 a.m.
PHARMACOLOGY & THERAPEUTICS SET A

32. Of the following, the one most likely to help maintain renal function in a patient with early
diabetic kidney disease vrould be:
A. Angiotensin receptor blocker
B. Beta blocker
C. Calcium channel blocker
D. Hydrochlorothiazide
33. A drug that combines with a receptor, hers an action and initiates a series of effects is
called:
A. A partial agonist
B. An agonist
C. A noncompetitive antagonist
D. Competitive antagonist
34. Which of the following hypnotic agent is able to interact with both BZt and BZz receptor
subtypes?
A. Flurazepam C. Phenobarbital
B. Zolpidem D. Zaleplon
35. Gastric acid secretion is under the control of the following agents, except
A. Histamine C. Serotonin
B. Acetylcholine D. Gastrin
36. The derivative of lysergic acid for migraine attack prevention is:
A. Metoclopramide C. Sumatriptan
B. Methysergide D. Ergotamine
37. Which of the following statements is FALSE?
A. Glucagon is synthesized in the A cells of the pancreatic islets of Langerhans
B. Glucagon is a peptide - identical in all mammals - consisting of a single chain of 29
amino acids
C. Glucagon is extensively degraded in the liver and kidney as well as in plasma, and at its
tissue receptor sites
D. Half-life of glucagon is between 6 and B hours, which is similar to that of insulin

38. Mifepristone (RU-486) is:


A. Anti-progestin
B. Anti-androgen
C. Anti-estrogen
D. Androgen

39. Which of the following coenzymes is of vitamin origin?


A. Carnitine
B. Coenzyme Qro
C. Pyridoxa l-5'-phosphate
D. Lipoic acid

40. All of the following are typical cholinergic effects, except


A. Decrease in heart rate
B. lncrease secretion of sweat
C. Contraction of bronchial muscle
D. lncreased in cardiac automaticity
41. Which of the following local anesthetic is a useful antiarrhythmic agent?
A. Cocaine C. Bupivacaine
B. Lidocaine D. Ropivacaine

Continued on Page 6
PHYSICIAN Licensure Examination !

Saturday, September 16,2017 - 08:00 a-m.,- 10:O0 a.m.


P HARMACOLOGY & THERAPEUTICS SET A

42. Flushing caused by this drug can be reduced by taking it after meals and/or pretreatment with
aspirin:
A. Lovastatin C. Gemfibrozil
B. Nicotinic acid D. Probucol
43. lnsulin is NOT reguired to facilitate glucose transport across membranes in:
A. Skeletal muscle C. Erythrocytes
B. Heart muscle D. Adipose tissue
44. Which of the following most directly describes the mechanism of action of digitalis?
A. lnhibit sodium-potassium ATPase
B. Decreases intracellular sodium concentration
C. lncreases the intracellular level of ATP
D. Stimulates the production of cAMP
45. After an elective TURP for BPH, a 69 year old man develops palpitations. He .has a history
of congestive heart failure. He is awake and conversing with his nurse. His blood pressure
is 1301/5. His ECG showsventriculartachycardia, ata rate o'f 12O beats per minute. The
best initial treatment of this arrhythmia would be?
A. Amiodarone 15O mgs lV over 10 minutes
B. lmmediate defibrillation with 360 J
C. Bretylium 5 mgs/kg lV
D. Diltiazem 15 mgs lV over 2 minutes

46. Which of the following opioids has the shortest duration of action?
A. Morphine C. Codeine
B. Meperidine D. Fentanyl
47. Which of the following is a G protein coupled receptor?
A. Muscarinic cholinergic receptor
B. Nicotinic cholinergic receptor
C. Glucocorticoid receptor
D. lnsulin receptor
48. Ketoconazole affects permeability of a microorganism's cell membrane by:
A. lntercalating into the membrane
B. Binding to membrane proteins
C. lnterfeiing with ergosterol synthesis
D. Disrupting cross-linkages in the cellwall
:

49. Flashback symptoms causing intense, incapacitating anxiety may occur in some patient who
have abused:
A. Secobarbital C. Diazepam
B. Ethanol D. Mescaline
50. All of the following could increase drug absorption EXCEPT
A. lncrease surface area dedicated to absorption
B. lncrease lipid solubility
C. lncrease p H when the drug is a weak acid
D. lncrease blood flow to the site of administration

51. Class of Tacrolimus (FK-506) is:


A. lmmunoglobulins
B. lmmunosuppressive agents
C. lnterferons
D. Monoclonal antibodies

Continued on Page 7
PHYSICIAN Licensure Examination Page 7
Saturday, September 16,2017 - 08:OO a.m. - 10:00 a-m.
PHARMACOLOGY & TH ERAPEUTICS SET A

52. There is a mechanism for antibiotic resistance to penicillin-derivative methicillin, NOT due to
a beta-lactamase, by all of the clinically important bacteria EXCEPT
A. Staphylococcus aureus
B. Escherichia coli
C. Streptococcus pneumoniae
D. Neisseria gonorrhea
53. Which of the following commonly used anesthetics inhibits methionine synthase activity and
causes megaloblastic anemia?
A. Desflurane
B. Halothane
C. Nitrous oxide
D. Soveflurane :

54. The primary reason for a physician to prescribe human insulin is that:
A. lt has a faster onset of action than other insulins
B. tt has a shorter duration of action than other insulins
C. lt can be given to patients who have an allergy to animal insulins
D. lt is more effective in preventing thd complications of diabetes than animal insulin
55. Signs and symptoms of toxicity from administration of an amide local anesthetic (e.9.
lidocaine) include all of the following, EXCEPT?
A. Seizures
B. Profound hypotension
C. Nausea and vomiting
D. Perioral numbness

56. Which of the following antibiotics is associated with bone marrow suppression?
A. Nitrofuran
B. Chloramphenicol
C. Kanamycin
D. Tetracycline

57. Blocks the final common pathway of platelet aggregation:


A. Ticlopidine
B. Clopidogrel
C. Aspirin
D. Abciximab
58. Sulfonamides increase the risk of n"onaial kernicterus because they:
B. Diminish the production of plasma albumin
C. Compete for bilirubin binding sites on plasma albumin
D. lnhibit the metabolism of bilirubin

59. lndication of Vitamin D3 is:


A. Hypercalcemia
B. Paget's disease
C. Hypophosphatemia
D. Osteomalacia
60. What drug is useful in the prevention of Acute Mountain Sickness (AMS)?
A. Lasix
B. Nifedipine
C. Oxygen
D. Acetazolamide
E. Dexamethasone

Continued on Page 8
PHYSICIAN Licensure Examination Page 8
Saturday, September 16, 2017 - 08:00 a.m. - 1O:00 a.m.
PHARMACOLOGY & THERAPEUTICS i SET A
i

61. With regard to the use of vasopressors in the management of hemorrhage shock, which of
the following statement is NOT correct?
A. Vasopressors usually result in an elevation of the blood pressure
B. Vasopressors achieve their goal of blood pressure support primarily through inotropic
effects
C. Dopamine given in low to moderate dosages may provide inotropic and chronotropic
support to the heart as well as enhance renal blood flow
D. ln general, the use of vasopressors in hemorrhagic shock is discouraged

62. lndication for calcium administration is:


A. Failure of formation of vitamin D in skin
B. Malabsorption of vitamin D from intestine
C. Hypercalcemia of malignancy
D. Vitamin D deficiency

63. Red man syndrome is associated with the use of which of the following antibiotics?
A. Vancomycin
B. Clindamycin
C. Ciprofloxacin

64. A 42 year old woman with metastatic breast cancer is lethargic and has mental
status changes. Blood work reveals: Seruin calcium = 4.2 mmol/L, ALP = 2OOO lU/L,
Creatinine = 130 umol/L, BUN = 15 mm/L. lmmediate treatment of her acute hypercalcemia
should include lV saline, calcitonin and?
A. Mithramycin
B. Bisphosphates I

C. Loop diuretics
D. Glucocorticoids

65. Which of the following drugs is most useful in the pre-operative management of patients with
pheochromocytoma?
A. Phenoxybenzamine (alpha blocker)
B. Metoprolol (beta blocker)
C. Furosemide (diuretic)
D. Enalapril (ACE inhibitor)
66. A 75 year old man develops pseudomembraneous colitis two weeks after undergoing
bilateral total knee replacement. He istreated with oral metronidazole for 15 days. One
week later he again develops diarrhea. This is best treated with?
A. Another 14 day course of oral metronidazole
B. Oralvancomycin for 14 days
C. lmodium 2 mg tab prn
D. Fecal enemas

67. ln acute myocardial infarction with premature ventricular contractions (PVCs) and potential
thrombus formation, which of the following represents an appropriate treatment sequence?
A. Lidocaine, Propranolol, Verapamil, Quinidine
B. Lidocaine, Nitroglycerine, Aspirin, Hepaiin
C. Amiodarone, Verapamil, Esmolol, Quinidine
D. Phenytoin, Digoxin, Propranolol, Captopril
68. lncorporation of uracil into macromolecules would be rapidly inhibited by the addition of which
of the following to a culture of sensitive cells?
A. Chlorampheniccil
B. Penicillin
C. Rifampin
D. Streptomycin

Continued on Page 9
PHYSICIAN Licensure Examination Page v
tsage 9
Saturday, September 16,2017 - 08:00 a.m. - 10:00 a"m.
PHARMACOLOGY & THERAPEUTICS SET A

69. Arecentlythyroidectomizedpatientbecamemarkedlytetanic. Which of the following would


rapidly reverse this symPtom?
A. Vitamin D
B. Calcitonin
C. Calcium gluconate
D. Parathyroid hormone
70. A yellow discoloration to newly erupted teeth in a 3-year-old child can be the result of prior
therapy with:
A. Tetracycline
B. Erythromycin
C. Ampicillin
D. Streptomycin
71. Most general inhalation anesthetic are effective through depression of the:
A. Cerebral cortex
B. Hypothalamus
C. Reticular formation
D. Pontine nuclei

72. This agent is very effective for aspirin-induced asthma and an alternative maintenance
medication for mild asthma and allergic rhinitis:
A. Theophylline C. Cromolyn sodium
B. Montelukast D. Ketotifen
73. Beta-receptor blocking drugs are used in the treatment of all the following diseases, except
A. Hypertension, ischemic heart disease, cardiac arrhythmias
B. Glaucoma
C. Pheochromocytoma
D. Hyperthyroidism
74. This drug is a Class lll antiarrhythmic drug:
A. Flecainide C. Lidocaine
B. Sotalol D. Verapamil
75. Ketorolac, a nonsteroidal anti-inflammatory drug, is often used in post - operative patients.
Which of the following statements concerning Ketorolac is NOT correct?
A. A 30 mg dose has demonstrated analgesic efficacy roughly equivalent to 10 mgs of
morphine
B. Ketorolac in large doses, may depress the respiratory drive
C. Gl and renal complications of Ketorolac use are very rare with short - term (3 days) peri-
operative use
D. Ketorolac may impair blood coagulation
76. This drug group useful in angina decreases myocardial oxygen requirement (by decreasing
the determinations of oxygen demand) and does not increase myocardial orygen delivery
(by reversing coronary arterial spasm):
A. Nitrates and nitrite drugs (Nitroglycerin, lsosorbide dinitrate)
B. Myotropic coronary dilators (Dipyridamole)
C. Potassium channel openers (Minoxidil)
D. Beta-ad renoceptor-blocking d ru gs (Atenolol, Metop rolo )I

77. A targeted cancertherapy is any drug that specifically acts on a well-defined target or
biological pathway. Which of the following drugs is NOT targeted therapy for cancer:
A. Herceptin
B. Cisplatin
C. Avastin
D. Gleevec

Continued on Page 10
PHYSICIAN Licensure Examination Page 10
Saturday, September 16,2017 - 0B:0O a.m. - 10:00 a.m.
PHARMACOLOGY & THERAPEUTICS SET A

78. Which of the following actions explains the antihypertensive effect of Guanethidine?
A. Blockade of intraneuronal storage of catecholamines
B. Substitution in storage granules of norepinephrine by false transmitter
C. lnhibition of postsynaptic adrenergic rebeptors
D. lnhibition of postsynaptic acetylcholine rticeptors
79. Which of the following diuretics causes the greatest degree of phosphaturia for any given
degree of natriuresis?
A. Ethacrynic acid
B. Acetazolamide
C. Furosemide
D. Hydrochlorothiazide
80. Peripheral neuropathy with prolonged use of antibiotics is commonly associated with?
A. Clindamycin
B. Metronidazole
C. Ciprofloxacin
D. Vancomycin

81. A chronic alcoholic patient may develop delirium tremors following surgery. Which of the
following drugs is UNLIKELY to be helpful in the management of this problem?
A. Dilantin
B. Benzodiazepine
C. Vitamin Br (thiamine)
D. Magnesium sulfate

82. Biliary tract infection, which are often caused by E. coli, are most effectively treated by an
antibiotic that is present in its active form in significant concentration in bile. Which of the
following antibiotics meets this criterion?
A. Penicillin C. Kanamycin
B. Ampicillin D. Gentamicin
oe Which one of the following is most effective in treating the ischemic pain of variant angina?
A. Sodium nitroprusside C. lsosorbide dinitrate
B. Atropine D. Nifedipine
84 Mechanism of the anti-emetic action of Metoclopramide:
A. Hr and Hz-receptor blocking effect
B. M-cholinoreceptor stimulating effect
C. Dz-dopamine and S-HTe-serotonin receptor blocking effect
D. M.cholinoblocking effect
85. Which of the following is TRUE of placebo?
A. Placebo is a dummy medication
B. Placebo is the inert material added to the drug for making tablets
C. Placebo do produce any effect
D. All patient respond to placebo
86. lmpressive evidence indicates that the andlgesic action of Aspirin:
A. ls enhanced as a buffered preparation
B. Result to a large extent by its ability to block histamine
C. Result to a large extent by its central and peripheral,actions
D. Results to a large extent by its ability to block bradykinin
87. Which antibiotic is an analogue of PABA (Para-Amino Benzoic Acid)?
A. Griseofulvin C. lsoniazid
B. Sulfonamide D. Trimethoprim

Continued on Page 11
PHYSICIAN Licensure Examination Page 11
Saturday, September 16,2017 - O8:OO a.m. - 1O:OO a-m.
PHARMACOLOGY & THERAPEUTICS SET A

88. All of the following antiviral drugs are the analogs of nucleosides, except
A. Saquinavir
B. Didanozin
C. Zidovudine
D. Acyclovir
89. Morphine is used to relieve pain in many clinical situations. ln which of the following
situations is morphine contraindicated secondary to its physiologic effects?
A. Angina
B. Acute cholecystitis
C. Acute pulmonary edema
D. Ureteralcolic
90. Which of the following effects would NOT be expected from ganglionic blocking drugs:
A. Vasodilatation
B. Salivation
C. Mydriasis
D. Decreased intestinal motilitY
91. Which of the following drugs is most likely to antagonize the bactericidal effect of Penicillin?
A. Bacitracin
B. Erythromycin
C. Neomycin
D. Streptomycin .i

92. Which one of the following patients is LEAST likely to require antimicrobialtreatment tailored
to the individual's condition?
A. Patient undergoing cancer chemotherapy
B. Patient with kidney disease
C. Patient with hypertension
D. Elderly patient
93. Which of the following drugs acts equally well on alpha and beta-adrenergic receptors?
A. Epinephrine
B. lsoproterenol
C. Norepinephrine
D. Propranolol
94, The drug is a potassium-sparing diuretic that block Na+ channels in the collecting tubules"
A. Acetazolamide C. Furosemide
B. Amiloride D. Hydrochloiothiazide
95. Mechanism of Trimethoprim action is:
A. lnhibition of cyclooxygenase
B. lnhibition of dihydropteroate reductase
C. lnhibition of dihydropteroate synthase
D. lnhibition of DNA gyrase
96. Which is the most appropriate treatment for Ascaris intestinal infestation?
A. Piperazine C. Praziquantel
B. Albendazole D. lvermectin
97. Mannitol, an osmotic diuretic, may be useful in all of the following procedures, EXCEPT
A. Treatment of elevated cerebrospina,l fluid pressure
B. Treatment of elevated intraocular pressure
C. Treatment of pulmonary edema with congestive heart failure
D. Diagnostic evaluation of acute oliguria

Continued on Page 12
PHYSICIAN Licensure Examination Page 12
Saturday, September 16,2017 - O8:OO a.m. - 10:OO a.m.
PHARMACOLOGY & THERAPEUTICS SET A

98. A 72-year-oldman exhibits progressive tremors, bradykinesia, and muscular rigidity. He is


slow to initiate movements and shows other motor abnormalities. Which drug is most useful
in the treatment of this disease?
A. Dopamine
B. Carbidopa
C. L-dopa
D. Haloperidol
99. Which of the following is the antidote for the toxin Copper?
A. Glucagon
B. Aminocaproic acid
C. Atropine
D. Penicillamine
'1
OO. lnstruction for the quantity and frequency of medication are written in a prescription after the
abbreviation:
A. Sig.
B. Rx
C. M.D.
D. p.o.

*** END ***

WARNING: Failure to submityour TestQuestions (Complete) set will cause the cancellation
of your Test-Result for this subject,
PHARMACOLOGY AND THERAPEUTICS 2017
ANSWER KEY

1. B 26. B 51. B 76. D


2.D 27. A 52. B 77. B
3.8 28. D 53. C 78. A
4,8 29. B 54. C 79. B
5.C 30. A 55. C 80. B
6.C 31. D 56. B 81. A
7.C 32. A 57. D 82. B
8.D 33. B 58. C 83. D
9.C 34. A 59. C 84. C
10. c 35. C 60. D 85. A
11. B 36. B 61. B 86. C
12. A 37. D 62. D 87. B
13. B 38. A 63. A 88. A
14. A 39. C 64. B 89. B
15. B 40. D 65. A 90. B
16. B 41. B 66. A 91. B
17. A 42. B 67. B 92. C
18. B 43. C 68. C 93. A
19. C 44. A 69. C 94. B
20. B 45A 70. A 95. B
21. D 46. D 71. C 96. B
22. B 47. A 72. B 97. C
23. D 48. C 73. C 98. C
24. A 49. D 74. B 99. D
25. C 50. c 75. B 100. A
Seat No.
Republic of.the Philippines
PROFESSIONAL REGULATION COMMISSION
Manila
BOARD OF MEDICINE

PHYSICIAN Licensure Examination


Saturday, September'16, 2017
SURGERY & OPHTHALMOLOGY. OTOLABYNGOLOGY & RHINOLOGY SET A

INSTRUCTION: Select the correct answer for each of the following questions.
Mark onlv one answer for each item by markirrg the box corresponding to the letter of your choice
on the answer sheet provided. STRICTLY NO ERASURES ALLOWED. Use pencil no. 2 only.
MULTIPLE CHOICE:
1. You are seeing a 41 ylo engineer for his periodic short physical examination. He tells you
that his father (70 y/o) recently died of cancer of the prostate and his older brother
is undergoing an evaluation for prostate disease. He denies any voiding difficulties. You
should (do):
A. A digital rectal exam (DRE) alone
B. A Prostatic Specific Antigen (PSA) test alone
C. Counselthe patient regarding options a and b and offer both to him
D. Send a consultation to your local urologist

2. A patient complain of dysphagia to solids and liquids. Barium swallow reveals a mildly
dilated esophagus which tapers down at the gastroesophageal junction. Manometry
reveals aperistalsis of the distal esophagus and non-relaxing lower esophageal sphincter.
TreatmentoptionsmayincludeeVeMhingexcept
A. Botulinum toxin injection into the lower esophageal sphincter
B. Prokinetics such as cisapride
C. Pneumatic balloon dilation across the lower esophageal sphincter
D. Calcium channel blockers such as diltiazem
3. The primary mechanism of reflux in patients with gastroesophageal reflux disease is:
A. Belching
B. Straining
C. Transient lower esophageal relaxations
D. Absent lower esophageal tone
4. First line treatment of cirrhotic ascites may include all of the following, except
A. Paracentesis C. Spironolactone
B. Dietary salt restrictions D. Transjugular intrahepatic portosystemic shunt
5. Striated muscle is present in the:
A. Lower Esophageal Sphincter C. ExternalAnal Sphincter
B. lnternalAnal Sphincter D. Pyloric Sphincter
b. Higher than normal levels of gastric acid secretion are associated with:
A. Esophageal ulcer C. Duodenal ulcer
B. Gastric ulcer D. Gastroesophageal reflux disease
7- A 65-year-old woman presents to the hospital complaining of severeepigastric pain,
radiating through to the back. She has diffuse abdominal tenderness and absent bowel
sounds. Her WBC is elevated, as are hei amylase and lipase. The most useful test to
assess the severity of her disease is:
A. Abdominal CT scan with lV contrast
B, ERCP
C. Liver and gallbladder ultrasound
D. 72-hour fecal fat collection

Continued on Page 2
PHYSICIAN Licensure Examination
Saturday, September 16,2017 - 11:OO a.ril. - O1:OO p-m.
SURGERY & OPHTHALMOLOGY. OTOI-ARYNGOLOGY & RHINOLOGY SET A

8. "Singer's node" refers to:


A. Enlarged cervical lymph node in a singer with laryngitis
B. Laryngeal papilloma
C. Polypoid laryngeal carcinoma
D. Laryngeal polyp
9. ln Pancreas Divisum:
A. The pancreas drains primarily through the (ventral) duct of Wirsung
B. The head and tail of the pancreas are bisected
C. The pancreas drains primarily through the (dorsal) duct of Santorini
D. Most patients with this anomaly will dev'elop pancreatitis
10. Surgery is contraindicated in which of the following lung cancers?
A. Small cellcarcinoma
B. Squamous cell carcinoma
C. Adenocarcinoma
D. Bronchiolar carcinoma
11. Melanoma in females is most frequently located on the:
A. Face C. Chest
B. Arms D. Lower legs
12. Pyloric stenosis is characterized by which of the following statements?
A. There is gastric outlet obstruction caused by hypertrophy of the muscular layer of the
pylorus
B. The hallmark symptom is bilious projectile vomiting
C. The string, olive and railroad signs are all seen
D. lt is more common in black males
13. Patient that are considered candidates for Goronary Artery Bypass Grafts (CABG) include all
of the following, except
A. Presence of left main coronary artery disease
B. Patients with single-vessel disease with proximal, discrete high-grade stenosis of the
major coronary artery, most commoilly the anterior descending
C. Abnormal left ventricular function
D. Life threatening arrhythmias caused by a previous myocardial infarction
14. Signs of colorectal cancer include the following, except
A. bloating C. dyspepsia
B. anemia D. occult bleeding
15. Recent reports indicate promising results with the use of this shunt in the treatment of Budd-
Chiarisyndrome:
A. Distal SplenoRenal Shunt (DSRS)
B. Transjugular lntrahepatic Portosystemic Shunt (TIPS)
C. SideTo-Side Portacavalshunt '
D. End-To.Side Portacaval Shunt
16. Contraindication for colonoscopy include all of the following, except
A. Fulminant colitis C. Acute diverticulitis
B. Severe toxic megacolon D. Pregnancy
17. The most common peripheral arterial aneurysm and the second most common aneurysm
after abdominal aortic aneurysm:
A. Femoral artery aneurysm
B. Thoracic aortic aneurysm
C. Popliteal artery aneurysm
D. lliac artery aneurysm

Continued on Page 3
PHYSICIAN Licensure Examination
Saturday, September 16,2017 - 11:OO a.m. - O1:OO p.m.
SURGERY & OPHTHALMOLOGY. OTOTARYNGOLOGY & RHINOLOGY SET A

18. Which of the following is an absorbable suture?


A. Polyglycolic acid suture
B. Silk suture
C. Polypropylene suture
D. Stainless steel suture
19. A29-year-old man presents with 4 months of diarrhea, malaise and intermittent tenesmus.
Physical exam is unrevealing. The patient has a mild normochromic, normocytic anernia.
LFi's are normal. Serum albumin is mildly decrease at 3.4 gm/dL. Stool sudan stain is
negative. D-xylose test reveals normal urinary excretion of D-xylose at 5 hours. Stool fecal
WBC's are positive. Stool heme-occult test is positive. The next most appropriate step in
this patient's work-up would be:
A. ERCP
B. Flexible sigmoidoscopy
C. Small bowel biopsy
D. 72-hour fecal fat collection
20 A4-year-oldgirl hassuddenonsetofabrJominal painandvomiting.She has a !'nass in the
right lower quadrant and hyperactive bowel sounds. A segment of resected bowel shows
prolapsing cecum/ascending colon. There is a necrotic mucosal lesion at the lead point of the
prolapse. Which of the following is the most likely diagnosis?
A. Appendicitis
B. lntussusception
C. Meckeldiverticulum
D. Strangulated hernia
21. Regarding peptic ulcer disease, which of the following is a false statement?
A. Atrophic gastritis caused by H. pylori eventually degenerates into carcinoma
B. Blood group O is associated with a higher incidence of duodenal ulcers
C. Spicy foods don't increase your risk for ulcers but they can irritate your stomach further
O. 1OO% of patient with gastric ulcers are H. pylori positive
22. The removal of the gallbladder, common bile duct, part of the duodenum, distal stomach and
the head of the pancreas is known as:
A. Whipple procedure
B. Billroth I procedure
C. Puestow procedure
D. Frey procedure I

23. The best urgent indication for a patient with initial hematocrit of less than 25o/o
A. Cryoprecipitate
B. Whole blood
C. Packed Red Blood Cells
D. Fresh Frozen Plasma
24. Which of the following nasogastric tubes with inflatable balloons used to decompress bleeding
esophageal varices?
A. Miller-Abbot tubes
B. Sengstaken-Blakemore tube
C. Salem Sump tube
D. Levin Rubber tube
25. Which of the following is not a radiological sign associated with acute pancreatitis?
A. Grey-Turner's sign
B. Frosberg inverted 3 sign
C. Sentinel loop sign
D. Colon cut-off sign

Continued on Page 4
PHYSICIAN Licensure Examination
Saturday, September 16, 2017 - 11:OO a.m. - O1:OO p.m.
SURGERY & OPHTHALMOLOGY. OTOLARYNGOLOGY & RHINOLOGY SET A

26. The presence of bloody nipple discharge in a 2S-year-old woman with the absence of a
palpable mass is most likely:
A. Fibrocystic Breast Disease
B. Paget's Disease of the Breast
C. Mammary Duct Ectasia
D. lntraductal Papilloma
27. Struvite renal calculi are most commonly oaused by which urea-splitting microorganism?
A. Escherichia coli C. Bacteroides fragilis
B. Proteus mirabilis D. Enterobacteraerogenes
28. Fine needle aspiration (FNA) of a solitary thyioid nodule will most accurately identify which
of the following?
A. Papillary carcinoma C. Medullary carcinoma
B. Follicularcarcinoma D. Hurthle cell carcinoma
29. A unilocular cystic lesion that is located in the middle neck, near the hyoid, is most likely to
be:
A. Brachial cleft cyst C. Cystic hygroma
B. Thyroglossal duct cyst D. Ectopic salivary gland tissue
30, Acute appendicitis is characterized by all of these except
A. The diagnosis may easily be confused with that of mesenteric lymphadenitis
B. The disease may be caused by the o-bstruction of appendiceal lumen by human pinworm
C. The presence of either neutrophils or eosinophils in the muscularis propria is diagnostic
D. The treatment of choice is surgery
31. All of the following mechanisms have been proposed for the pathogenesis of gastric stress
ulcers due to extensive burns (Curling's ulcer) exe,ept
A. Hypersecretion of gastric acid l

B. Mucosal hypoxia related to neurogenic vasoconstriction


C. Systemic (metabolic) acidosis
D. Lowering of intracellular pH levels
32. A 12-year-old boy is admitted to the hospital with a chief complaint of "severe pain in the left
leg". History reveals that one week prior to admission the patient had been treated for scarlet
fever. He now presents with the following clinical features: restlessness and headache,
vomiting, general appearance of severo illness, rapid pulse rate, temperature of 40 "C and
severe pain the left leg. The white blood cell count 21,000 per cu. mm. The left knee joint is
extremely tender to touch and manipulation. Point tenderness is localized to the distal end
of the femur. The neighboring .ioint can be manipulated without increasing the pain. No
swelling isfound in anyother joints. X-rays reveal no abnormalities including films of the left
knee joint. Which of the following would be of least value in the management of this patient?
A. Early treatment with salicylates
B. Aspiration of knee joint
C. Hyperbaric oxygen therapy
D. Blood culture
33. A67-year-oldfemalecomplainsofcrampy epigastric pain, occurring in spasms. She has
regurgitated greenish mucoid material several times over the past sixteen hours. She has
passed one normal dark brown stool about 2 hours ago. On physical examination the
abdomen is noted to be somewhat tender and distended, and there are occasional visible
peristaltic waves. On x-ray you would probably find:
A. A positive pyloric "string-sign
B. A "water-bottle' stomach shadow
C. Dilated sigmoid and transverse colon
D. Dilated loops of small bowel with fluid levels

Continued on Page 5
PHYSICIAN Licensure Examination Page 5
Saturday, September '16,2017 - 11:00 a.m. - O1:O0 p.m.
SURGERY & OPHTHALMOLOGY. OTOLARYNGOLOGY & RHINOLOGY SET A

34. A young man is seen in the emergency room after a head-on deceleration collision in
which he passed through the steering wheel and struck his face against the windshield. The
mandible appears intact, but there are fractures bilaterally and in the central compartment of
the face whereby the lower face is "floatin!" in dysfunction from the cranium. The injury is
known as:
A. LeFort I C. LeFort lll
B. LeFort ll D. Temporomandibular disorder (TMD)
35. Apatientpresentstotheemergencyroom'witha temperature of 39"C, aheartrateof 115,
and a respiratory rate of 25. There are no localizing symptoms and the work-up does not
reveal any specific source for the fever. Which of the following best describe this patient's
condition? I

A. lnfection C. Sepsis
B. SIRS D. Septib shock
36. A 43-year-old black woman complains of right upper quadrant pain, as well as nausea
and vomiting appearing about one hour following dinner. Physical examination reveals slight
icterus as well as local tenderness in the right hypochondrium. The most likely diagnosis is:
A. Acute pancreatitis C. Acute choledocholithiasis
B. Leaking gastric ulcer D. Bilaterral pyelonephritis
37. A 48-year-old man presents with the history of sudden onset of pain in the epigastrium
which radiates through the back and the right shoulder. The pain is described as
"agonizing" and lasted for two hours. Following this attack the patient experienced residual
"soreness". The initial attack was associated with nausea and vomiting. Physical
examination reveals marked tenderness and some muscle rigidity in the right upper
quadrant of the abdomen. Pulse rate is rapid, temperature 38.3 oC and white cell count is
15,000cu.mm.Assumingthattheultrasoundoftheabdomenrevealsnocalculi:
A. Acute cholecystitis is ruled out
B. Acute pancreatitis is now established
C. Acute cholecystitis is stilla possible diagnosis
D. Renal calculi are definitely not present
38. A 25-year-old male presents with a history of malaise,nausea and vomiting, and generalized
severe abdominal pain. Several hours later the pain localized around the umbilicus
subsequently shifting to the right lower quadrant. The nausea and vomiting gradually
subsided. On examination the patient had tenderness over the right lower quadrant with
muscular rigidity in this area. Rectal examination localized the pain to the right lower
quadrant. Temperature was 38.5 oC, pulse 14Olmin., and the leukocyte count was 12,OOO cu.
mm. The impression was acute appendicitis and was confirmed at surgery. lf the surgeon
finds a normal appendix at the time of surgery, the next logical step is:
A. Close the abdomen and observe for 24 hours
B. Remove the appendix to eliminate the future possibility of appendicitis
C. Extend the incision and look for an inflamed gallbladder
D.Askaseniorconsuitantforasecondorfurtheropinion
39. A 3-year-old boy is found to have a heart murmur on a routine physical examination. The
child has been in good health and has developed normally. The murmur is present ovei the
' second left intercostal space; to-and-fro with a systolic accentuation; .describ-ed best ias a
"machinery type murmur". Blood pres3ures are within normal limits. Heart size is normal on
x-ray examination and the pulmonary markings are slightly accentuated. However, on
fluoroscopy a typical "hilar dance" is seen. Which of the following would be the best
indication for immediate surgery in this case?
A. Age of patient
B. Recovery from subacute bacterial endocarditis
C. Subacute bacterial endarteritis not responding to antibiotics
D. Associated anomalies of the heart

Continr.ied on Page 6
PHYSICIAN Licensure Examination Pagie 6
Saturday, September 16,2017 - 11:OO
SET A

40. Regarding the scenarios listed below, which one of the operations should proceed as
scnbdutea?
A. A 67 year old man scheduled for left total hip arthroplasty who has a pulse of 8O and
blood pressure 18O1110, is asymptomatic, and takes beta-blockers
B. A 65 year old hypertensive man scheduled for bilateral total knee arthroptasty who has a
pulse of 90 and blood pressure 13OnO and who takes angiotensin-converting enzymes
(ACE) inhibitors
C. An 8O year old man scheduled for cataract surgery who has a pulse of 6O, blood pressure
180/1OO and is completely asymptomatic i

D. An 80 year old man scheduled for bitateral laparoscopic inguinal hernia repair who hab a
pulse of 42 and a blood pressure of 12A160 and who has a pacemaker and takes bpta-
blockers
.

41. A 65-year-old woman presents to the Emergency Room with a right hip fracture. Sh'e is
scheduled for open reduction and internal fixation the following day. The orthopedic resident
notices that the patient is tachycardic and hyper-reflexic. Blood test confirm that the patient
has hyperthyroidism. The appropriate peri-operative management of this condition is?
A. Close observation in the ICU post-operatively
B. Begin propylthiouracil and proceed with surgery
C. Begin beta blocker with propranolol and proceed with surgery
D. Arrange urgent radio-iodine ablation and proceed with surgery
42. lncreased intracranial pressure is correctty described as all of the following, except
A. Resulting from intracranial tumors
B. Relieve by solute diuresis
C D. Treatable with ventriculostomy ,i

43. A 28-year-old man sustains a gunshot wound just above the right clavicle. On arrival his
systolic blood pressure is 60 mmHg and he is agitated. Which of the following is indicated
as the next step to his management?

A
44. The examiner is attempting to elicit Chvostek's sign to his patient. Which of the following
operations was the patient most likely to have undergone?
A. Nephrectomy C. Carotid endarterectomy
B. Thyroidectomy D. Esophageal dilation
45. Which of the following is a false statement?
A. Both the rule of 9's and the Lund-Broward chart may be used to estimate the body surface
area in burn
B. It is important to include 1o burns when calculating fluid requirements for a major burn
C. Partial thickness burns present with weeping blisters
D. Tangential eschar debridement is associated with more intraoperative bleeding than full
thickness debridement
46. Appropriate duration of antibiotic therapy for most patients with bacterial peritonitis from
perforated appendicitis is:
A. 3-5 days C. 14-21 days
B. 7-1O days D. > 21 days

47. A patient with a BMI of 38 is considered:i


A. Overweight C. Severiely obese
B. Obese D. Superobese

Continued on Page 7
PHYSICIAN Licensure Examination PaEe 7
Saturday, September 16, 2017 - 11:OO a.m. - O1:00 p.m.
SURGERY & OPHTHALMOLOGY. OTOLARYNGOLOGY & RHINOLOGY SET A

48. A 78-year-old male has had long-standing epidermoid carcinoma of the penis. He souq!!
mediial attention only after pronounced hematuria developed. Which of the following is NOT
true of patient's condition?
A. Penile cancer develops in squamous epithelium of the foreskin or glan penis
B. The condition would not have developed had the patient been circumcised as an adult
C. Elderly subjects are more likely to develop this disorder
D. The lesion is slow-growing
49. Should the patient's cervical sympathetic plexus become involved by tumor spread, all of
the following may result, excePt
A. Ptosis
B. Horner syndrome
C. Mydriasis
D. Anhidrosis
i

SO. A 27-year-old man presents to the emergbncy room after receiving blows to i!9 head. He
open his eyes with painful stimuli, is confused, and localizes to pain. What is his Glasgow
Coma Score?
A. 13 c.9
B. 11 o.7
51. A gangrenous foot occurred in a patient with endocarditis who developed widespread
embolito numerous organs. Which of the following would be LEAST expected to have
occurred in this patient?
A. Pain C. Pink extremity
B. Paresthesia D. Loss of pulse
52. Non-neoplastic syndromes which may be associated with bronchogenic carcinoma would
LEAST likely include which of the following?
A. Myasthenic syndrome
B. Hypocalcemia i

C. Dermatomyositis
D. Peripheral neuropathy
53. All of the following are known complications of heparin administration, except
A. Skin necrosis
B. Cholestatic hepatic injury
C. Arterialthrombosis
D. Osteoporosis
54. A 34-year-old female with a history of ulcerative colitis developed a high fever, tachycardia,
and prostration. She appeared toxic,and considerable abdominal distention and tenderness
were noted. Which of the following might be expected in this patient?
A. Appendicitis C. lncreased hematocrit
B. Constipation D. Toxic megacolon
55. All of the following statements correctly clescribe wound contraction, except that
A. Onset is concurrent with the appearanbe of granulation tissue
B. Myofibroblast is believed to be the responsible agent
C. lt is inhibited by partial or full-thickness skin grafts'
D. lt is inhibited by epidermalgrafts
56. Which of the following is not an indication for diagnostic peritoneal lavage?
A. Unconscious patient
B. Pregnancy
C. Equivocal abdominalfinding
D. Thoracic spine injury

Continued on Page 8
PHYSICIAN Licensure Examination
Saturday, September 16, 2017 - 1 1:OO a.m. - O1:OO p.m'
SURGERY & OPHTHALMOLOGY. OTOLARVNGOLOGY & RHINOLOGY SET A
I

57. Apatientwasadmittedforfurtherdiagnosticwork-up. lntheward,hefell downfromhisbed


with his right chest hitting the floor. He c.:/nplained of right sided chest pain and difficulty of
breathing. Chest x-ray reveals a 10olo Pneumothorax of the right lung. One should order now
a:
A. Bronchoscopy to identify the site of the injury
B. Give high concentration of orygen and observe
D. Tube thoracostomy :

58. ln a woman under 4O years of age, which of the following breast abnormalities would have
the highest predictive value of malignancy?
A.PainfulmovablemaSSC.Bloodynippledischarge
B, Painless movable mass D. Breast skin edema with dimpling i

59.WhichofthefollowingSuggeStunresectabilityofaleftupperlobelungcancer?
A. Hemoptysis
B. Cough specimen with positive sputum cytology
C
D. Clubbing and blueness of fingers
60. A 29-year-old man presents to his primary care physician with a painless testicular mass.
Laboiatory studies show an elevated serum human chorionic gonadotrophin level. Which of
the following is the most likely site of nodal metastasis in this tumor?
A. Deep inguinal lymph nodes
B. External iliac lymph nodes
C. Gluteal lymph nodes
D. Para-aortic lymph nodes ,

61. ln penetrating neck injuries, Zone ll is refirred to an area between:


A. Clavicle and cricoid cartilage i

B. Cricoid cartilage and angle of mandible


C. Above the angle of the mandible
D. Below the clavicle
62. What is the cause of the most severe testicular pain?
A. Testicular malignancy
B. Torsion of the testicle
C. Testicular inflammation (Orchitis)
D. Undescended testicle (Cryptorchidism)
63. All of the following are beneflcial for decreasing the incidence of deep-vein thrombosis in
patient with hip fracture except
A. Warfarin C. Aspirin
B. Low-dose heparin D. Dextran
64. The three most important nerves which should be preserved during axillary dissection are:
A. Thoracodorsal nerve, Medial and Lateral pectoral nerves
B. lntercostobrachiat nerve, Long thoracic nerve, Medial pectoral nerve
C. Thoracodorsal nerve, lntercostobrachial nerve, Lateral pectoral nerves
D.Thoracodorsalnerve,Longthoracicrterve,]ntercostobrachialnerve
65. ln management of hemorrhagic shock, the best clinical sign of successful fluid resuscitbtion
is? |

A. An increase in blood pressure


B. An increase in urine output
C. An increase in arterial oxygenation
D. A decrease in tachycardia

Contiirued on Page 9
PHYSICIAN Licensure Examination
Saturday, September 16,2017 - 1 1:00 a.m. - O1:OO p.m.
SURGERY & OPHTHALMOLOGY. OTOTARYNGOLOGY & RHINOLOGY ser e
66. The type of hernia repair which only consist of tightening an enlarged deep ring is called:
A. Marcy's repair
B. High ligation of sac
C. McVay-Cooper repair
D. Bassini-Shouldice repair
67. A 64-year-old previously healthy man is admitted to hospital because of closed head injury
and a ruptured spleen following a motor vehicle accident. He is given 213 - 113 glucose/saline
solution at 125 ml/hr. The patient is somnolent but easily aroused until the 5th day when he
is noted to be in a deep coma. Shortly afterwards, he has a grand mal seizure. The following
lab data are obtained: Serum electrolytes: Na+ = 122, K+ = 1.9, Cl = 96, HCO" = 19. Serum
osmolarity = 260. Urine electrolytes: Na+ = 61, K+ = 18. What is the most likely cause of the
patient's seizure?
A. Hypokalemia c. lntracfanial bleeding
B. Hyponatremia D. Hypomagnesia

68. Which of the following is the vessel that is most commonly injured during a cholecystectomy?
A. Right hepatic artery
B. Portalvein
C. lnferior vena cava
D. Right gastroepiploic artery
69. Which of the following is the best initialtreatment for a patient with achalasia and a small
hiatal hernia?
A. Balloon dilation C. Myotomy of the LES
B. Botulinum toxin D. Long esophageal myotomy

70. A 1-year-old boy was brought to the ER with bilious vomiting, bloody stools, and abdominal
distention. The new intern order an abdominal radiograph and abdominal ultrasound.
The abdominal radiograph exhibits the non-specific "double-bubble" sign and the abdominal
ultrasounds shows the superior mesenteric artery and vein twisting in a clockwise fashion.
Which of the following conditions is the most likely cause of this patient's symptoms?
A. Colorectal carcinoma
B. Duodenal atresia
C. lntussusception
D. Volvulus
71. ln the acute-phase response to injury or infection, levels of which of the following serum
proteins is decreased?
A. C-reactive protein
B. Fibrinogen
C. Ceruloplasmin
D. Albumin
72. The most common bone malignancy in children is:
A. Periosteal sarcoma
B. Ewing's sarcoma
C. Osteosarcoma
D. Rhabdomyosarcoma
73. A 42-year-old man presents with headache, palpitations and diaphoresis. His urinary
metanephrines are elevated. lmaging studies reveals a 3 cm mass in his right adrenal gland.
The treatment of choice is:
A. Adrenalectomy
B. Radiotherapy
C. Tumor embolization
D. Chemoradiation

Continued on Page 1O
PHYSICIAN Licensure Examination Page 1:O

Saturday, September 16, 2017 - 1 1:00 a.m. - O1:00 p.m.


SURGERY & OPHTHALMOLOGY. OTOLARYNGOLOGY & RHINOLOGY SET A

74. Most bacterial abscesses require treatment with drainage and antibiotics however, all of the
following abscesses can be treated with antibiotics alone, except?
A. Lung abscess
B. Amebic liver abscess
C. Tubo-ovarian abscess
D. Empyema of the gallbladder
75. Which of the following is the nrost appropriate initial treatment of a sucking chest wound?
A. Occlusive dressing taped on 3 out of 4 sides
B. Chest tube placed through the wound, cover wound (and chest tube) with occlusive
dressing
C. Chest tube placed in a clear area, closure of the wound
D. Closure of the wound, intubation of the patient, sedation
76. Patients with bladder stones are at increased risk for which of the following bladder cancers?
A. Adenocarcinoma
B. Transitional cell carcinoma
C. Squamous cell carcinoma
D. Choriocarcinoma
77 , The best choice for initial fluid resuscitation in trauma patients with hypovolemic shock is:
A. Blood C. Albumin
B. Plasma D. Crystalloids .

78. A32-year-old woman complains of difficulty swallowing, foul breath and regurgitation of
undigested food. Her most likely diagnosis is:
A. Esophageal cancer
B. Achalasia
C. Boerhaave's syndrome
D. Zenker's diverticulum

79. Which of the following is the surgical procedure for the management of malrotation?
A. Sistrunk procedure 'l

B. Ladd's procedure l

C. Lilly's procedure
D. Kasai procedure
80. Which of the following is NOT a cause of intermittent claudication?
A. Popliteal cyst
B. Takavasu's disease
C. Popliteal entrapment
D. Active radiation surgery
81. A S2-year-old woman with chronic malnutrition is due to undergo major surgery. lt
is decided to start total parenteral nutrition (TPN) as part ol' the initial therapy. While
introducing a central venous catheter into the right subclavian vein, the patient develops
sudden dyspnea. Which of the following is the most likely diagnosis?
A. Air embolism
B. Fat embolism
C. Acute anxiety
D. Pneumothorax
82. Which of the following can cause spuriously elevated pulse oximeter readings?
A. Methemoglobinemia
B. Finger nail polish
C. Hyperbilirubinemia
D. lV methylene blue

Continued on Page 11
PHYSICIAN Licensure Examination
,:
Saturday, September 16, 2017 - 1 1:0O a.m. - 01:OO p.m.
SURGERY & OPHTHALMOLOGY. OTOLARYNGOLOGY & RHINOLOGY SET A

83. The primary treatment for dyspnea ("air hunger'') in a dying patient:
A. Supplemental oxygen by a non-rebreather mask
B. Cooling the room
C. Opioids
D. Anxiolytic agents
84. A 60-year-old woman presents to the physician because of red-brown streaks of blood
in her stool. She reports no pain with bowel movements. lnitial laboratory tests show:
Hematocrit 33o/o, Hemoglobin: 11 g/dL, Mean corpuscular volume: 73 pms, WBC count:
8,000/mma, Platelet count: 200,000/mm". Which of the following is the most important next
step in management?
A. Abdominal plain film
B. Colonoscopy
C. CT scan of the abdomen
D. Esophagoduodenoscopy
85. Which of the following statements concerning the resuscitation of trauma patients is TRUE?
A. Treatment shoulJ be terrninated if the patient requires continuing resuscitation after
receiving 5O units of packed RBCs
B. Hypotensive patients with blunt abdominal trauma should not be resuscitated until they
are able to receive definitive care
C. Hypothermia affects platelet function
D. Hypertonic saline improves survival
86. A S3-year-old woman underwent brain tumor resection four days ago develops left lower
extremity pain and swelling. An ultrasound examination demonstrates clot in the femoral
popliteal veins on the left. The most appropriate treatment for this is?
A. Begin ambulation and discontinue bed rest
B. Begin intravenous heparin administratibn
C Use intermittent leg compression and graduated-compression stockings
D. Place an IVC filter
82. The most important basis for microstaging of melanoma:
A. Presence of ulceration
B. Determination of cortical growth
C. Depth of skin invasion of tumor
D. Presence of lymphatic involvement
88. Patients undergoing elective splenectomy should receive vaccination against Streptococcus
pneumoniae, H. influenzae type B and Meningococcus:
A. 2-4 weeks before surgery
B. The day of surgery
C. 1 week after surgery
D. 1 month after surgery
89. A 2S-year-old asthmatic is scheduled for,elective right inguinal hernia repair. ln the holding
area he has severe wheezing bilaterally. The optimal management for this patient is?
A. Carry out the procedure under local anesthesia with lV sedation to avoid airway
manipulation
B. Administer ventolin and atrovent nebulizer treatqnent in the holding area and proceed
with surgery when the patient stops wheezing
C.Administercorticosteroidsandcarryouttheprocedureunderspinatanesthesia
D. Postpone surgery until the patient's asthma is under control

Continued on Page 12
PHYSICIAN Licensure Examination Page 12
Saturday, September 16, 2017 - 11:00 a.m. - O1:OO p.m.
SURGERY & OPHTHALMOLOGY. OTOLAR\'NGOLOGY & RHINOLOGY SET A

90. Which type of melanoma has the best overall prognosis?


A. Superficial spreading
B. Nodular
C. Lentigo maligna
D. Acral lentiginous
91. The Glasgow coma scale is best used for':
A. Patient who are brought to ER after VA
B. Drunk patient brought to ER after witnessed stabbing incident in the street
C. Awake patient seen in the ER on 4Oth hour after isolated blunt injury
D. lntubated patient brought to ER after CSW of the neck
92. Which of the following statements concerning chemical injuries is TRUE?
A.lmmediatewound-careinvotvesapplicationofaneutralizingagent
C. Alkali burns produce deeper injuries than do acid burns
D. Chemical injuries frequently cause severe hyperkalemia secondary to metabolic acidosis

93. What is the most common structural disturbance causing airway obstruction?
A. Nasal bone fracture
B. Nasal polyps
C. Septal deviation
D. Hypertrophied turbinates
94. One should not catheterize a trauma patient in the presence of:
A. Blood on the meatus
B. Hematuria
C. Pelvic fracture
D. Crushing renal injury
95. Orchidopexy (for an undescended testes):
A. lmproved fertility
B. Decreases cancer risk :

C. lncreases the risk of trauma to the te'stes


D. lncreases the risk of torsion of the testes
96. A 56-year-old man is diagnosed with transitionalcell carcinoma and treated with excision and
therapy, Which of the following risk factors most likely led to this patient's cancer?
A. Alcohol use
B. Exposure to aniline dyes
C. Exposure to asbestos
D. Previous pyelonephritis
97. ln which of the following is the potential loss of blood from a closed fracture greatest?
A. Humerus
B. Spine
C. Pelvis
D. Femur

98. Frequentfindingsinapatientwithatraumaticbasalskullfractureinclude all of the following,


except
A. Bruising behind the ear
B. Facial nerye palsy
C. Otorrhea
D. Severe epistaxis

,^^^ai^.,^J D^a^ .{t


^^
PHYSICIAN Licensure Examination Page 13
Saturday, September 16,2017 - 11:00 a.m. - 01:00 p.m.
SURGERY & OPHTHALMOLOGY. OTOLARYNGOLOGY & RHINOLOGY SET A

99. The Child-Pugh scoring system for liver function is frequently used to assess the risk of
surgery in patients known to have liver disease. Which of the following factors is NOT part
of the Child-Pugh score?
A. Overall nutritional status
B. Presence of ascites
C. Presence of encephaloPathY
D. INR

100. What is the most common cancer in the world?


A. Breast
B. Gastric
C. Lung
D. Liver

*** END ***

WARNING: Failure to submityour TestQuestions(Complete)set will cause the cancellation


of your Test-Result for this subject.
SURGERY, OPHTHALMOLOGY, OTOLARYNGOLOGY & RHINOLOGY
ANSWER KEY

1, C 26. D 51. C 76. C


2.D 27. B 52. B 77. D
3.C 28. A 53. B 78. D
4.D 29. B 54. D 79. B
5.C 30. c 55. D 80. D
6.C 31. A 56. B 81. D
7. A 32. A 57. D 82. A
8.D 33. D 58. D 83. C
9.C 34. C 59. C 84. B
'10. A 35. B 60. D 85. C
11. D 36. C 61. B 86. D
12. C 37. C 62. B 87. C
13 B 38. B 63. B 88. A
14. C 39. C 64. D 89. D
15. B 40. c 65. B 90. c
16. D 41. C 66. A 91. A
17. C 42. C 67. B 92. C
'18.A 43. A 68. A 93. C
19. B 44. B 69. C 94. A
20. B 45. B 70. D 95. A
21. D 46. A 71. D 96. B
22. A 47. C 72. C 97. C
23. C 48. B 73. A 98. D
24. B 49. C 74. D 99. A
25. A 50. B 75_ A 100. c
Seat No.
Republic of the Philippines
PROFESSIONAL REGULATION COMMISSION
Manila
BOARD OF MEDICINE

PHYSICIAN Licensure Examination


Saturday, September 16, 2017
MEDICINE SET A

INSTRUCTION: Select the correct answer for each of the following questions.
Mark onlv one answer for each item by marking the box corresponding to the letter of your choice
on the answer sheet provided. STRICTLY NO ERASURES ALLOWED. Use pencil no. 2 only.
MULTIPLE CHOlCE:
1. A24-year-old ranger presents to the emergency room with a board-like abdomen and rigidity.
Whichofthefollowingismostconsistentwiththispatient'spresentation?
A. Black widow spider bite
B. Rattlesnake bite
C. Scorpion's bite i

D. Centipede's bite
2. The incidence of Molluscum contagiosum as a sexually transmitted disease is increasing in
young adults and results in the formation of small wart like lesions in the genital region.
Which of the following viruses might also be suspected in such a case of sexually acquired
lesions?
A. Cytomegalovirus
B. Varicella-zoster virus
C. Human papiltomavirus
D. Human immunodeficiency virus
3. The most important feature of nephrotic syndrome:
A. Polyuria C. Edema
B. Proteinuria D. Hypolipidemia
4. A S4-year-old woman with newly diagnosed hypertension has started enalapril at an
appropriate dose. What is the most likely clinical adverse reaction of the patient to this drug?
A. Acute urticaria
B. Chronic cough
C. lnterstitial nephritis
D. Photoallergic reaction
5. A2O-year-oldmanpresentsto his physician with a 2-day history of fever, vomiting, and
diarrhea. His laboratory studies are unremarkable except for a serum albumin level of
7.5 g/dL. Which of the following conditions would most likely cause this patient's laboratory
abnormality?
A. Acute infection C. Nephrotic syndrome
B. Dehydration D. Poor nutritional status
6. A 64-year-old man with no prior medical history has had increasing back pain and right hip
pain for the past decade. The pain is worse at the end of the day. On physical examination, he
has bony enlargement of the distal interphalangeal joints. Common imaging findings also
include narrowing of the joint space, sclerosis and the presence of osteophytes. Which of the
following diseases is most likely the cause of this patient's symptoms?
A. Gout
B. Osteomyelitis
C. Osteoarthritis
D. Rheumatoid arthritis

Continued on Page 2
PHYSICIAN Licensure Examination
Saturday, September 16,2017 - O2:OO p.m. - O4:OO p.m.
MEDICINE SET A

7. A definitive diagnosis of ascariasis can be made by observing which of the following?


A. An eosinophilia in a differentialwhite blood cell count
B. Motile larvae in a stool samPle
C. Larvae in x-ray of lungs
D. An adult worm passed during a bowel movement
8. A 47-year-old man presents to the emergency room one and half hours after the onset of
severe substernal chest pain radiating to his left arm. The pain is accompanied by diaphoresis
and shortness of breath. His blood pressure is 165/95 mm Hg, pulse is 85/min, and respiratory
rate is 18/min. Which of the foltowing tests, is the most important tool in the initial evaluation
of patients in whom acute myocardial infarction (Ml) is suspected?
A. ECG C. Lactate dehydrogenase
B. Creatinine kinase D. Aspart'ate aminotransferase
g. A 6O-year-old patient from an undeserved indigent family sees a physician and complains of
years of polydipsia, polyuria, polyphagia, and worsening fatigue and weight loss. Urinalysis
ieveals severe proteinuria, and a renal biopsy is ultimately performed, with a classical
Kimmelstiel-Wilson lesion. Which of the following histological finding is apparent in the renal
tissue?
A. Wire loop abnormality
B. Nodular glomerulosclerosis
C. Crescent formation
D. Bence Jones protein
10. A 56-year-old woman present to your office with pain and stiffness in her hands. She states
that the stiffness seems to last the entire morning. Which of the following joint findings is most
suggestive of an inflammatory arthritis, rather than osteoarthritis, as the cause of her joint
pain?
A. Painful range of motion C. Swelling and warmth
B. Crepitus D. Bony articular enlargement
11. Which of the following has a greater chance of surviving traumatic cardiopulmonary arrest?
A. Patient who suffer blunt force trauma to the torso
B. Patient with non-dilated pupils
C. Patient with dilated unresponsive pupils
D. Patients with penetrating chest trauma who are hypothermic
12. A3O-year-old man has had difficulty swallowing for both solids and liquids over the past
6 months. What is the most likely diagnoSis?
A. Achalasia
B. Barrette's esophagus
C. Benign esophageal stricture
D. Esophageal carcinoma
13, A 70-year-old woman presents with a 1-year history of rrrild slowness and loss of dexterity.
Her handwriting has become smaller, and her husband feels her face is less expressive
and her voice softer. Over the last few months she has developed a subtle tremor in the
right hand, noted while watching television. Her symptoms developed insidiously but
have mildly progressed. She has no other medical history, but she has noted some
mild depression and constipation over the last 2 years. Her examination demonstrates
hypophonia, decreased blink rate, micrographia, and mild right-sided bradykinesia and
rigidity. An intermiftent right upper extremity resting tremor is noted while she is walking.
The rest of her examination and a brain MRI are normal. Which diagnostic facie is
indicative of the above case?
A. Snarling facie
B. Cushingoid facie
C. Mask like facie
D. Torpid facie

Continued on Page 3
PHYSICIAN Licensure Examination
Saturday, September 16,2017 - O2:OO p.m.i- O4:OO p.m.
l

MEDICINE !
SET A
14. A 48-year-old man is admitted to the hospital for treatment of congestive heart failure. 'He
has known historyof diabetes, Alzheimer's disease, and high-altitude pulmonary edema, in
addition to a severe allergy to all sulfa derivatives. Which of the following drugs is appropriate
for diuresis in this patient?
A. Acetazolamide C. Hydrochlorothiazide
B. Furosemide D. Spironolactone
15. A previously healthy 43-year-old man was admitted to the emergency department following
recurrent syncopal episodes with fever, weakness, shortness of breath, and cough
symptoms. His son reported that the patient had two episodes of loss of consciousness. The
first episode lasted about 1-2 min and the patient regained consciousness without confusion
soon afterward. The second episode occuried 5 min after the first episode while the patient
was sitting on the bed. The son denied that he had any accompanying seizure activity. His
past medical history was normal and he denied taking any drugs or having chest pain.
During the initial physical examination, he was alert, blood pressure was 105170 mmHg,
heart rate was BO beats/min, and body temperature was 39.0 "C. Cardiac auscultdtion
showed normal heart sounds with no murmur or friction rub. Chest auscultation revealed
bilateral rhonchi and inspiratory crackles in the right lower lung. The neurological
examination was normal. lnitial laboratory data showed leukocytosis (14,400/mm3),
elevated C-reactive protein (189 mg/l), and normal blood chemistry, including potassium
(4.3 mEq/L), calcium, magnesium, and serial'cardiac markers (troponin l, and CK-MB). The
chest X-ray showed hazy infiltrates in the right lower lung. A cranial computed tomography
was unremarkable. A 12-lead ECG was recorded while the patient was febrile (temperature
39.O "C), showed down sloping ST-segment elevation in leads V1-V2, consistent with a
type-l (coved) Brugada ECG pattern. He was transferred to the cardiac intensive care unit
for continuous ECG monitoring and was treated with intravenous paracetamol. A second
ECG was taken several hours after the aileviation of fever, converted to a type-2 (saddle-
back) Brugada ECG pattern. An echocardiogram showed normalsystolic function and the
absence of segmental wall motion abnorrnalities. Her family history was significant for the
sudden unexplained infant death (9 months of age) of her son. Ambulatory ECG monitoring
showed monomorfic short-run ventricular tachycardia. Pneumonia was healed with an
antibiotic (Ceftriaxone) and an antipyretic (Paracetamol) for 1O days. The patient was
diagnosed with BrS and as the patient had syncope and a positive family history of sudden
cardiac death, an implantable cardioverter defibrillator (lCD) implantation was suggested.
Because of these factors, together with his age and his request that an ICD not be
implanted, a subsequent electrophysiological study (EPS) was performed for risk
stratification and no ventricular arrhythmias were induced. Quinidine was administered and
the patient was recommended to receive urgent antipyretic treatment for fever that might
induce Brugada - like ECG changes. Electrocardiogram screenings were negative for
consenting family members. One year after his discharge, the patient remained alive
and free of cardiac events and syncope. Which of the following studies about Brugada
syndrome is/are correct?
A. lt is a genetic disease and follows autosomal dominant inheritance
B. Human SCNSA gene mutations alter the cardiac Na+ channel kinetics
C. lt commonly affects young men of South East Asian descent
D. Allare correct
15. An agitated and confus e 44-year-old mhn is brought to the emergency room after collapsing
at his office. The patient has a history of diabetes mellitus and was recently diagnosed with
hypertension. He is conscious when the medical team arrives and complains of fatigue and
dizziness but denies any chest pain, palpitdtions, shortness of breath, sweating, fever, or
chills. His diabetes has been poorly controlled with glyburide, and his most recent HbAIC
was 8.57o. He is unable to recall the namg of his antihypertensive drug. Which of the following
agents is most likely responsible for this patient's condition?
A. Enalapril C. HydLochlorothiazide
B. Hydralazine D. Propranolol

Continued on Page 4
PHYSICIAN Licensure Examination Page 4
Saturday, September 16, 2017 - O2:OO P.m. - O4:OO p.m.
MEDICINE SET A

16. Scientist say they have proven a link between the Zika virus and the development of
this neurological illness. Which of the following debilitating disease finding is the most likely?
A. Guillain-Barre' syndrome-
B. Lou Gehrig's disease
C. Duchenne Muscular dystrophy
D. Lambert-Eaton syndrome
17. Afair-skin 56-year-old farmer presents to the clinic with a scaling, indurated, utcerated nodule
just below his lower lip. Biopsy reveals, sheets of neoplastic epidermal cells and keratin
pearls. Which of the following is the most likely diagnosis?
A. Acral lentiginous
B. Actinic keratosis
C. Squamous cetl carcinoma
D. Basat cell carcinoma
18. Gut bacteria are responsible for all of the following, EXCEPT
A. Hydrolysis of urea and other proteins with the production of ammonia
B. Production of Vitamin K
C.Productionofluminalmutagenswhichmaypromoteneoplasiainthecolon

19. A 42-year-oldmancomestothephysician for a follow-up examination 1 week after he


passed a renal calculus. X-ray crystallographic analysis of the calculus showed calcium
as the primary cation. Physical examination today shows no abnormalities. A 24-hour
collection of urine shows increased calcium excretion. Which of the following is the most
appropriate pharmacotherapy?
A. Carbonic anhydrase inhibitor
B. Na+-Cl- symport inhibitor
C. Na+-K+-2Cl- symport inhibitor
D. Renal epithelialsodium channel inhibitor
20. Which of the following is NOT expected irr a patient with chr:onic renat failure?
A. Low levels of parathyroid hormones
B. Relatively low levels of erythropoietin
C. Lack of active Vitamin D
D. Elevated serum phosphorus

21. A 7O-year-old male iswith hypertension, chronic obstructive pulmonary disease (COPD),
benign prostatic hyperplasia, and peripheral vascular disease is treated with numerous
medications for recalcitrant high blood pressure (1701105 mm Hg). His medication include
prazosin, high-dose atorvastatin, lisinopril, and hydrochlorothiazide (HCTZ). Recently he
has been hospitalized to optimized management of his blood pressure. Hydralazine was
added to his medication regimen. Which of the following signs or symptoms would be most
concerning after administration of hydralazine?
A. Angina
B. Bradycardia
C. Dyspnea
D. Dysuria

22. A 14-year-old girl presentsto herprimaryphysicianwith an intensely pruritic rash on both


hands and on her right cheek. She notes that the rash appeared the day after camping trip
with her school. The rash consist of vesicles on erythematous plaques arranged linearly with
slight crusting. She denies fever or sore throat and otherwise feels well. She does not have
history of eczema or sick contacts. Which of the following is the most likely diagnosis?
A. Atopic dermatitis
B. Contact dermatitis
C. Erythema infectiosum
D. Seborrheic dermatitis

Continued on Page 5
PHYSICIAN Licensure Examination
Saturday, September 16,2017 - O2:OO p.m. - O4:OO p.m.
MEprcrNE sEI4
23. A patient presents to the emergency depiartment with severe headache, palpitation, and
elevated blood pressure. He is found to have elevated urinary vanillylmandelic acid levels.
He is diagnosed with pheochromocytoma with predominantly elevated norepinephrine levels.
Which of the following agents will antagonize both vascular and cardiac actions of
norepinephrine?
A. Atenolol C. Esmolol
B. Labetalol D. lsoproterenol

24 A 67-year-old woman with a long history of poorly controlled diabetes mellitus and chronic
renal failure is admitted to the hospital for treatment of cellulitis. Two days into her hospital
stay she complains of chest pain that is relieved when she leans forward. An ECG shows
diffuse ST segment elevations with PR depressions; her echocardiogram is normal. Which
of the following is the most appropriate treatment at this time?
A. Cardiac catheterization C. Nonsteroidal anti-inflammatory drugs
B. Dialysis D. Switch her to another antibiotic regimen
25. A 4O-year-old housewife has been experiencing left-sided electric shock-like facial pain for
several months which occurs in cycles and with periods of remissions. The pain was primarily
located in the left cheek and left jaw region and aggravated by chewing, laughing and other
muscle facial movements. She has had evaluations by several dentists and subsequently
had two root canals performed at an attempt to relieve the pain. But this procedure did not
provided relief. She proceeded then to a neurological evaluation which subsequently resulted
to a definitive diagfrosis. What do you think is the most likely signature presentation of this
patient?
A. Lumpyjaw C. Dropped jaw
B. Locked jaw D. Slapped jaw
26. A 23-year-old medicalstudent has been experiencing frequent headaches forwhich he has
been taking increasingly large doses of aspirin for 3 months. One night he takes a particularly
large dose, becoming confuse, and falls into seizures. He presents to the emergency room
with a serum salicylate levelof 130 mg/dL. Which of the following is the most appropriate
treatment?
A. Bicarbonate C. N-acetylcysteine
B. Vitamin K D. Protamine SO4
27. Leuprolide is an analog of gonadotropin - releasing hormone. lt results in decreased
synthesis of sex steroids and is an effective treatment for:
A. Prostate cancer C. Lung cancer
B. Cervical cancer D. Bladder cancer
i

28. A3O-year-old man presents to the resuscitation bay with gunshot wounds in the anteriorLnd
posterior left chest. Although in distress and dyspneic, the patient is cooperative. He has a
patent airway and is moving allextiemities. His pulse is 120/min, blood pressure 120190 mm
Hg, and respiratory rate is 30/min. He has bounding distal pulses, and no other injuries are
identified on secondary examination. X-ray of the chest reveals fluid in the pleural space,
and a left chest tube thoracostomy yields 600 mL of bright red fluid. Over the next hour
an additional 750 mL of blood is collected. What is the most appropriate next step in
management?
A. lnsert another chest tube
B. Thoracentesis
C. Left thoracotomy
D. Remove the chest tube and suture the incision closed

29. Overnight secretion of acid is primarily driven by:


A. Action of histamine alone
B. Action of acetylcholine and histamine ir
C. Action of gastrin alone ,l
D. Action of gastrin and histamine

Continued on Page 6
PHYSICIAN Licensure Examination Page 6
Saturday, September 16,2017 - O2:OO p.m. - O4:O0 p.m.
MEDICINE SET A
l

30. Examples of primary prevention for cardiovascular disease include all of the following
EXCEPT:
A. Control of high blood pressure
B. Control of congestive heart failure
C. Control of high blood cholesterol
D. Cessation of smoking

31. Which of the following is an appropriate definition of shock?


A. Low blood pressure
B. Low cardiac output
C. Low circulating volumes
D. lnadequate tissue perfusion
32. Proliferative retinopathy in D.M. is often treated using:
A. Tonometry
B. Fluorescein angiogram
C. Antibiotics
D. Laser surgery
33. This syndrome characterized by acute renal failure, macroangiopathic hemolytic anemia and
thrombocytopenia is a consequence of infection caused by VTEC:
A. Acute Glomerulonephritis
B. Henoch-schonlein purpura
C. Hemolytic uremic syndrome
D. ldiopathic thrombocytopenic purpura

34. Which one of the following complications is the most common cause of morbidity and
mortality in Type 2 diabetes?
A. Diabetic retinopathy
B. Coronary artery disease
C. Diabetic nephropathy
D. Diabetic neuropathy
35. Hemorrhagic lesions seen on the palms of the hands in patient with S. aureus endocarditis
are often called:
A. Osler nodes C. Roth spots
B. Janeway lesions D. Hammond's sign
36. A 4O-year-old woman reports a recent history of skin rashes that are exacerbated by the sun
(particularly in the cheeks and nose), and swelling of joints. What disease presents this way?
A. Hashimoto's thyroiditis
B. Systemic lupus erythematosus
C. Rheumatoid arthritis
D. Contact dermatitis
37. Typical features of pheochromocytoma does not include which of the following:
A. Severe and symptomatic paroxysmal hypertension always generateJ suspicion of
pheochromocytoma :

B. There are four (4) main syndromes asr;ociated with familial pheochromocytoma namely
von Hippel-Lindau syndrome, MEN,type 2A and 28, Neurofibromatosis type 1, and
Paragang lioma syndrome
C. lt is a rare usually cancerous neuroendocrine tumor of the inner medulla of the adrenal
glands
O. bften referred to as the "ten percent tumo/' because they do many things about ten
percent of the time.

Continued on Page 7
PHYSICIAN Licensu re Examination Page 7
Saturday, September 16,2017 - O2:OO p.m. - 04:00 p.m.
MEDICINE SET A

38. Hirschsprung's disease (congenital megacolon) is characterized by all of the following except
A. Erratic proliferation of non-myelinated nerve fibers in the nondilated segment of the colon
B. Absence of ganglion cells in the dilated,portion of the colon
C. Histochemical demonstration of acetylcholinesterase in the non dilated segnnent of the
colon
D. lt is clinically manifested by failure of the infant to pass meconium
39. A 22years old poorfarmerfrom a village is admitted with 6 months history of burning
sensation & weakness in lower limbs. He is complaining of nausea,/vomiting, constipation,
progressive dysphagia, abdominal pain and generalized body aches for the last three
weeks. Past history is not significant. Family history is only positive for hypertension.
Traveling / medication / allergic history is not significant. Vital signs: Pulse: 12O weak,
BP:110/80, Temp:98. O/E. Anemia in palms and conjunctiva, Respiratory system is clear.
CVS: Loud S1+S2+No murmurs. Gl: Diffuse irbdominal tenderness, mo-re at hypochondriac
region and left iliac fossa. Neurological + Musculoskeletal: Hyperreflexia, Hypertonia,
Hyperasthesia, Horizontal nystagmus, Dysdiodokinesia. Romberg's test is positive. Limb
power was 5/5 but movements were grossly reduced. Skin: Tight, Pigmented spots are
there. Eye=Decreased vision. Ears=Decreased hearing. CBC=Normocytic anemia. (Hb 8.1,
MCV 98). CPK= Normal. MRt=Normal. Electrolytes: Normal. Based on history and physical
examination, what is the most likely diagnosis?
A. Schistosomiasis C. Organophosphate poisoning
B. Lead Toxicity D. Vitamin B12 deficiency
4O. Which of the following infections is a significant cause of bronchiectasis?
A. Pneumococcus
B. Nontuberculous mycobacterial infection
C. Rhinovirus
D. Aspergillus
41. The most common type of atrial septal defect (ASD) is:
A. Sinus venosus defect
B. Ostium primum defect i

C. Ostium secundum defect


D. Atrioventricular defect
42. Which of the following is commonly seen in patients with chronic pericarditis?
A. Dyspnea at rest C. Chest pain
ts. Ascites D. Cough
43. A 64-year-old male with gastroenteritis developed a moderate deficit in extracellular fluid
volume. Which of the following would be LEAST expected in this individual?
A. lncreased heart sounds
B. Orthostatic hypotension :

C. Collapse neck veins


D. lncrease heart rate
44. Primarily responsible f6r edema:
A. Arterialpressure
B. Venous pressure
C. Lymphatic pressure
D. Gut luminal pressure
45. Which anterior mediastinaltumor is associated with myasthenia gravis?
A. Teratoma
B. Seminoma
C. Thymoma
D. Lymphoma

Continued on Page B
PHYSICIAN Licensure Examination
Saturday, September 16,2017 - O2:OO p.m. - 04:00 p.m.
MEDICINE SET A

46. Lung injury due to smoke inhalation is caused by?


A. Thermal injury to major airways
B. Adult respiratory distress syndrome (ARDS) induced by the burn wound
C. Smallairway obstruction with soot

47. Which of the following ECG changes is least likely to occur with hypokalemia?
A. ST-T segment depression
B. T-wave inversion
C. Second or third degree AV block
D. PVCs

48. Severely malnourished patients who are started on TPN may develop a "refeeding
syndrome" which may result in respiratory failure or even cardiac arrest. The electrolyte
abnormalities that contribute to the refeeding syndrome include all of the following,
EXCEPT?
A. Hypophosphatemia C. Hypokalemia
B. Hyponatremia D. Hypomagnesemia
49. Which of the following is the most common cause of death within five years in patients with
intermittent leg claudication?
A. Stroke
B. Myocardial infarction
C. Cancer
D. Lower extremity gangrene

50. Part of the cardiovascular response to injury or infection is arterial vasodilation and
increased protein permeability in the capillary membranes. Which of the following
inflammatory mediators causes arterial vasodilatation but NOT increased capillary
permeability?
A. Nitric oxide C. Histamine
B. Bradykinin D. Platelet-activating factor

51. A 17 year old man is brought to the emergency room following a motor vehicle crash. The
paramedics reportthat he lost hisvital signs 15 minutes prior to arrival to the emergency
room and that CPR was initiated. On arrival, he is being bag mask ventilated with bilateral
air entry. His pupils are not reactive and he is apneic with no palpable pulses. His ECG
shows a wide complex QRS at a rate of 36. The most appropriate management would be?
A. lntubate, resuscitative left thoracotomy
B. lntubate, lV access and massive transfusion
C. lntubate, lV access, massive transfusion and pericardiocentesis
D. Termination of resuscitation efforts

52. Which of the following is true regarding carbon monoxide (CO) poisoning?
A. CO shifts the oxygen-hemoglobin disassociation curve to the right
B. CO irretrievably binds to the heme-molecules of hemoglobin
C. Pulse oximetry is an accurate measurement of oxygen saturation after CO poisoning
D. CO has 200 times more affinity for hemoglobin than oxygen

53. All of the following are compensatory mechanisms that occur during the pathogenesis of
congestive heart failure EXCEPT:
A. An increase in ventricular end-diastolic volume
B. An increase in the concentration of plasma catecholamines
C. An increase in vagal tone
D. lncreased activity of the renin-angiotensin-aldosterone system

Continued on Page 9
PHYSICIAN Licensure Examination
Saturday, September 16,2017 - 02:O0 p.m. - 04:O0 p.m.
MEDICINE SET A
54. All of the following neoplasms have been associated with AIDS, EXCEPT?

A: Iii;lii::'ff#I'homa
C. Gastrointestinal stromal tumor (GIST)
D. CNS lymphoma

55. A serious complication of temporal arteritis is:


A. Hypertension
B. Bell's palsy
C. Blindness
D. Dysarthria

56. A 60 year old man undergoes parathyroidectomy for primary hyperparathyroidism. Post-
operatively, the patient develops perioral tingling, numbness, and carpopedal spasm.
A. Prolonged QT interval
B. Peaked T waves
C. Torsades de pointes
D. U waves
57. ln patients undergoing coronary artery bypass grafting, all of the following have been
identified as being associated with an increased operative risk EXCEPT?
A. Advanced age C- Male sex
B. Low ejection fraction D. Diabetes
58. New onset post-operative atrial fibrillation in non-cardiothoracic surgical patients?
A. Typically resolves with correction of the electrolytes and fluid balance
B. ls likely to be permanent if not cardioverted to normal sinus rhythm within 24 hours
C. Requires immediate cardioversion to sinus rhythm
D. ls best prevented with perioperative calcium channel blockers

59. A 64 year old man is scheduled undergo femoral-popliteal bypass grafting for peripheral
vascular disease. He had one episode of congestive heart failure two years previously and
is now well controlled with an angiotension - converting enzyme (ACE) inhibitor and a
diuretic. He gardens and is able to carry on normal activities without restrictions. His resting
ECG is normal. Additional pre-operative cardiac testing should be include:
A. Nuclear medicine cardiac study
B. Exercise stress testing
C. Coronary angiography
D. No further testing
60. All the following bacteria are associated with the production of exotoxin, EXCEPT?
A. Bacteroides fragilis
B. Clostridium tetani
C. Staphylococcus aureus
D. Streptococcus pyogenes

61. Weakness, fast pulse, and normal blood pressure suggest what condition?
A. Compensated hypovolemic shock
B. Decompensated neurogenic shock
C. Late burn shock
D. Late hemorrhagic shock

62. The most common organism associated with overwhelming post-splenectomy sepsis is?
A. Haemophilus influenzae
B. Streptococcus pneumonia :

C. Meningococus
D. Babesia microti

Continued'on Page 1O
PHYSICIAN Licensure Examination
Saturday, September 16,2017 - O2:OO p.m. - 04:00 p.m.
MEDICINE SET A

63. Which of the following statements is true regarding fluid resuscitation in burn victims?
A. Normal saline should be avoided
B. Hypertonic saline should be used in severe burns to reduce edema
C. Albumin use is associated with an increased mortality rate
D. Abdominal compartment syndrome frorrr over-resuscitation does not.occur

64. Cardiac orygen delivery can be increased by increasing all of the following EXCEPT?
A. Hemoglobin
B. Atmospheric pressure
C. Cardiac output
D. lnspired oxygen concentration ,

65. A 55 year old man presents with a 12 hour history of epigastric pain, nausea, and vomiting.
He has diffuse mild abdominal tenderness on palpation. Laboratory values are
significantforaserumamylaseof 800u/L, serum glucose of 11.3, WBC 12,OOO, serum
sodium 125 mg/L, and serum triglyceride level of 1800 mg/dl. The most like explanation for
his hyponatremia is:
A. Excessive fluid loss
B. Pseudohyponatremia
C. lnappropriate antidiuretic hormone response

66. A 7O-year-old man is brought to the emergency department by his wife because of fever
and shortness of breath for 2 days. He underwent an oral surgical procedure 6 weeks 5go.
His respirations are 22lmin, and blood pressure is 140/60 mm Hg. A soft diastolic,murmur is
heard. The diagnosis of bacterial endocarditis is made. Gentamicin therapy is initiated. This
patient is at increased risk for developing which of the following as a result of this therapy?
A. Cardiac ischemia C. Hyperglycemia
B. Hearing loss D. Lung infection

67. A 12-year-old girl is admitted to the hospital because of marked shortness of breath, an
erythematous rash, and painful, swollen hip and knee joints. She is agitated. A chest x-ray
shows an enlarged heart and changes consistent with pulmonary edema. lntractable
congestive heart failure develops, and she dies on the second hospital day. This child most
likely had a recent history of which of the following?
A. Jaundice C. Pharyngitis
B. Meningitis D. Skin infection

68. A 37-year-old woman comes to the physician because of a 2-month history of pain with
movement of her hands and feet. Physicarl examination shows warmth and swelling of the
metacarpophalangeal and metatarsophalangeal joints. Laboratory studies show increased
titers of antibodies to Fc component of lgG and a negative antinuclear antibody test result"
A drug is prescribed that binds to tumor necrosis factor-o (TNF-o) and blocks its interaction
with cell-surface TNF receptors. Her symptoms improve within 1 month. She is most likely
receiving treatment with which of the following drugs?
A. Adalimumab C. Gold
B. Anakinra D. Methotrexate

69. A S8-year-old man present with accelerating angina despite his current antianginal regimen
(calcium channel blockers and beta blockers). Coronary angiogram shows severe proximal
l-AD artery stenosis and severe proximal circumflex artery stenosis. Your recommendation
is:
A. Coronary bypass grafting
B. PTCA of both lesions
C. Discontinue present medications and begin nitrates
D. lncrease calcium channel blocker and beta blockers to control symptoms

Continued on Page 11
PHYSICIAN Licensure Examination Page 11
Saturday, September 16,2017 - O2:O0 p.m. - 04:00 p.m.
MEDTCINE gE+
,
70. A 28-year-old African American man present with fever, weight loss, diffuse myalgias and
arthralgias, and abdominal pain. On examination his blood pressure is 168/92 mm Hg,
pulse is 83/min. He has areas of ulceration and areas of mottled purple discoloration on his
lower extremities. His medical history is otherwise significant for hypertension and hepatitis
B infection. Laboratory test reveals an eleriated WBC count, erythrocytes sedimentation
rate, and C-reactive protein levels. What is the most likely diagnosis?
A. Tuberculosis infection
B. Churg-Strauss syndrome
C. Polyarteritis nodosa
i
I

D. Wegener's granulomatosis l

71. A58-year-old man isbroughttothe trauma bay unresponsive and intubated. He was an
unstrained drivdr in a high speed motor vehicle crash. Medical transporters report that the
car's windshield was broken. His pulse is 58/min and regular, blood pressure is 8O/50 mm
Hg, and respiratory rate is 1S/min. Chest auscultation reveals good air flow bilaterally and no
adventitial lung sounds. He opens his eyes to command and his pupils are equally round and
reactive to light. He is unable to move his extremities or withdraw to pain. His e{remities are
pink and warm. Which of the following is the most likely cause for this patient's hypotension?
A. Cardiogenic shock C. Neurogenic shock
B. Septic shock D. Hypovolemic shock
72. Recombinant lL--2 (aldesleukin) promotes the proliferation, differentiation, and recruitment
of lymphoid cells. lt is thought to promote immune-mediated antitumor effects and is used
to treat:
A. Renal cell carcinoma
B. Basal cell carcinoma
C. lnvasive ductal carcinoma
D. Squamous cell carcinoma
73. A 23-year-old man comes to the physician with a bacterial infection. On questioning the
patient reveals a history of recurrent bacterial, fungal, and viral infection. Blood is drawn and
sent for laboratory analysis, which reveals all levels of immune cells (T-lymphocytes, B-
lymphocytes) are low. Which of the following conditions is most likely to have caused the
patient's symptoms?
A. Ch6diak -Higashi disease
B. Severe combined immunodeficiency
C. Ataxia telangiectasia
D. Wiskott-Aldrich syndrome
74. Twenty-four hours after placement of a catheter, a hospitalized patient develops a fever and
chills. Within t hour her systolic blood pressure falls 3O points and she develops swelling in
her extremities. Despite valiant effort by the hospital staff, the patient dies. X-ray of the
patient's lungs taken only hours before she passed away shows pulmonary edema. Which of
the folftcwing mediators of this patient's disease process is most likely responsible for the
pathology described?
A. C3a C. y-lnterferon
B. C5a D. lnterleukin-1
75. A32-year-oldmanworkingonaconstructioncrewfalls from the scaffolding and develops
multiple long bone fractures. Four hours after presenting to the emergency department, he
complains of increased shortness of breath, hypoxemia, and has diffuse infiltrates on his
chest radiograph, consistent with development of acute respiratory distress syndrome. Which
of the following findings is almost always present in patient's who present with ARDS?
A. Normal oxygenation with impaired minute ventilation
B. Reduced lung compliance
C. lncreased arterial PCOz
D. Pulmonary embolism

Continued on Page 12
PHYSICIAN Licensure Examination I Page 12
Saturday, September 16,2017 - O2:OO p.m. - 04:00 p.m.
MEptcrNE q+a
76. Chronic obstructive pulmonary disease (COPD) is classified as emphysematous or bronchitic,
depending on the pathologic changes that occur in the lung. Although these two COPD
syndromes rarely exist as pure entities, they may be differentiated on the basis of their clinical
presentation. Which of the following clinical features is common to both the emphysematous
and bronchitic types?
A. Polycythemia
B. Chronic cough
C. Dyspnea
D. Hypercapnia
77. A middle-aged manwhohas recently begun taking captopril and hydrochlorothiazide for
hypertension develops an itchy rash in sun-exposed areas. The most likely cutaneous
diagnosis is:
A. Urticarla
B. Vasculitis
C. Photosensitivity reaction
D. Erythema multiforme
78. A 73-year-old man attends a family dinner celebration and has a grand evening- As he leaves
the restaurant, he collapses. Which of the following diagnoses is most likely?
A. Seizure
B. Cardiac arrhythmia
C. Carotid sinus hypersensitivity
D. Postprandial syncope
79. A dysfunctional myocardial endothelium underlies one form of heart disease. ln patients with
this common disease process, there is a lack of autoregulatory coronary artery vasodilation
needed to provide increased blood flow in states of increased physical exertion or emotional
stress. The pathogenesis is most often from severe narrowing of atherosclerotic coronary
vessels and typically manifests as chest pain, relieve by rest or nitroglycerine tablets. Which
of the following is a soluble metabolite that mediates the compensatory coronary artery
vasodilation during periods of increased myocardial oxygen demand?
A. Acetylcholine C. Carbon dioxide
B. Adenosine D. Norepinephrine
80. A 22-year-old man with schizophrenia is being treated as an inpatient with a first-generation
anti-psychotic medication. Several hours after his initial dose, he develops involuntary muscle
spasms, particularly in his facial muscles. To which class of drugs does the indicated
treatment for this patient's adverse effects belong?
A. Beta blockers
B, Anticholinergic agents
C. Dopamine agonists
D. Mood stabilizers
81. A patient complains of excess drooling, tremors, and multiple joint pains. Physical and
laboratory reveal that the patient has hepatitis. Which of the following is the most likely
diagnosis?
A. Rheumatic fever
B. Wilson's disease
C. Whipple's disease
D. Gout
82. The most frequent location for spider angiomata in cirrhosis is?
A. Abdomen
B. Face and Neck
C. Back
D. Upper extremities

Continued on Page 13
PHYSICIAN Licensure Examination Page 13
Saturday, September 16,2017 - O2:O0 p.m. - 04:00 p.m.
MEptclNE
83. A S2-year-old obese man is evaluated in your office during a routine annual visit. =3 He
denies any fatigue or recent weight changes. He has normal appetite and physically active.
He had a colonoscopy 1 year ago that was normal. On physical examination, he is obese
with a BMI of 34. His skin is getting darker and thicker in the armpits. Which of the following
is most useful in diagnosing the underlying condition?
A. Serum lnsulin levels

C. Hemoglobin AIC
D. CT Scan of the Chest, Abdomen and Pelvis

84. Foot care for patient diagnosed with NIDDM:


A. Soak foot daily in tap water
B. Avoid using moisturizer
C. Cut toenails straight across and smooth nails with an emery board
D. Cut corns and calluses with nail cufter
85. A 42-year-oldwoman comes to the clinic complaining of blurry vision. She states that for the
past 3 weeks her eyes have been very dry and itchy, and she is unable to make tears. She
also states that she has had a very dry mouth despite drinking adequate fluids. Physical
examination reveals bilateral dry, ulcerated corneas and fissures on the sides of her lips. ln
addition, both of her knees are erythematous and swollen. When asked about her knees, she
says "Yes, my knees and wrists tend to be swollen and stiff in the morning, but my mom had
arthritis". Testing for several autoantibodies reveals she is rheumatoid factor-positive and
antibody-SS-B (La)-positive. Which of the following is the most likely diagnosis?
A. Cystic fibrosis
B 3i!tr#3it?[;"
o. CnESr syndrome
,

86. A 6o-year-old woman presents to the physician because of red-brown streaks of blood in her
stool. She reports no pain with bowel movements. lnitial laboratory tests show: Hematocrit:
33oh, Hemoglobin: 11 g/dl, Mean corpuscular volume: 73 ;:m", WBC count: 8,000/mms,
Platelet count: 200,000/mm". Which of the fotlowing is the most important next step in
management?
A. Abdominal plain film
B. Colonoscopy
C. CT scan of the abdomen
D. Pelvic ultrasound
87. Which one of the following is a reason to interrupt the initial assessment?
A. Cardiac arrest
B. Multiple open (compound) fractures
C. Severe head injury with brain tissue visible
D. Severe shock
88. Which of the following conditions can precipitate repeated attacks of acute pancreatitis?
A. Hyperuricemia
B. Hypertriglyceridemia
C. Hyperglycemia
D. Flypercholesterolemia
89. The most common protozoal cause of septic shock is:
A. Toxoplasma
B. Cryptosporidium
C. Malaria
D. Amoeba

Continued on Page 14
PHYSICIAN Licensure Examination Page 14
Saturday, September 16,2017 - O2:OO p.m: - O4:OO
MEDICINE SET A
90. An 19-year-old female presents with a twelve hour history of troublesome cough followed by
wheezing and breathlessness unresponsive to albuterol. She has had troublesome asthma
since the age of 18 months. She has been poorly compliant with her preventive medication.
On PE, she is extremely distressed. She appears slightly cyanotic, and her oxygen saturation
is 84%o. What type of immune reaction causes this?
A. Type I hypersensitivity reaction
B. Type ll hypersensitivity reaction
C. Type lll hypersensitivity reaction

91. Which of the following is not a screening tool for coagutopathy?


count
a: Elx'f,il
C. APTT
D. Hemoglobin
92. A S5-year-old man who is a chronic alcoholic develops severe memory loss with marked
Confabulation and ataxia. Deficiency of which of the following vitamins would likely to
contribute to the neurologic damage underlying these symptoms:
A. Vitamin 812
B. Vitamin 81
C. Riboflavin
D. Niacin
93. What is the definitive treatment choice for ST elevation Ml?
A. lV nitroglycerine
B. lV beta blockers
C. Unfractionated heparin administration
D. Timely reperfusion using fibrinolytics or PCI
94. Which of the following is an absolute indication for hemodialysis?
A. Hypervolemia
B. Hyperkalemia
C. Respiratory acidosis
D. Uremic pericarditis

95. The cerebrospinal fluid abnormality suggestive of meningitis:


A. lncreased pressure
B. lncrease white blood cell count
C. lncrease protein
D. Decrease sugar
96. What is the most common cause of acute
A. Alcohol
B. Drugs
C. Gallstone
D. Hypertriglyceridemia
97. Patient is taking Vancomycin. Which of the following must be assessed closely?
A. Pulse rate
B. Have eye check-up
C. Creatinine level
D. Transaminase studies

Continued on Page 15
PHYSICIAN Licensure Examination
Saturday, September 16,2C17 - O2:OO p-nl. - O4:OO p.m.
MEDICINE SET A

98. Which of the following is considered the hallmark sign of thyroid storm?
A. Hypotension
B. Hypothermia
C. Uncontrolled fever (37.8 to 41 oC)
D. Enlargement of the neck
99. What is the most common cause of cardiopulmonary arrest in the trauma patient?
A. Brain injury
B. Hypoxemia
C. Myocardial contusion
D. Ventricular arrhythmia
100. Which group of syndromes appears to be closely related to Sudden Unexplained
Nocturnal Death Syndrome (SUNDS)?
A. Brugada syndrome
B. Broken heart syndrome
C. Takotsubo syndrome
D. Holiday heart syndrome
*** END ***

WARNING: Failure to submit your Test Questions (Complete) set will cause the cancellation
of your Test-Result for this subject.
MEDICINE 2017
ANSWER KEY

1. A 26. A 51. D 76. C


2.C 27. A 52. D 77. C
3.E} 28. C 53. C 78. D
4.8 29. B 54. C 79. B
5.B 30. B 55. C 80. B
6.C 31. D 56. A 81. C
7.D 32. D 57. C 82. B
8.A 33. C 58. A 83. C
9.8 34. B 59. D 84. C
10. c 35. B 60. A 85. C
11. B 36. B 61. A 86. B
12. A 37. C 62. B 87. A
13. C 38. B 63. A 88. B
14. D 39. D 64. B 89. A
15. D 40. B 65. B 90. A
16. A 41. C 66. B 91. D
17. C 42. B 67. C 92. B
18. D 43. A 68. A 93. D
19. B 44. C 69. D 94. B
20. A 45. C 70. c 95. B
21. A 46. D 71. C 96. C
22. B 47. C 72. A 97. C
23. B 48. B 73. B 98. C
24. B 49. B 74. A 99. B
25. C 50. A 75. B 100. A
Seat No.
Republic of the Philippines
PROFESSIONAL REGULATION COMMISSION
Mariila
BOARD OF MEDICINE
PHYSICIAN Licensure Examination
Sunday, September 17, 2017
OBSTETRICS AND GYNECOLOGY sEr B
INSTRUCTION: Select the correct answer for each of the following questions. i

Mark onlv one answer for each item by marking the box corresponding to the letter of your choice
on the answer sheet provided. STRICTLY NO ERASURES ALLOWED. Use pencil no. 2 only.
MULTIPLE CHOICE:

1. Supine hypotension syndrome in the pregrrant patient is caused by:


A. Urine obstruction of venous blood flow
B. Atelectasis (collapse of small airways) of the lungs
C. Uterine pressure on the vagal nerye
D. Gastric reflux

2. Physiologic changes secondary to pregnancy include all of the following EXCEPT?


A. lncreased plasma volume C. Decreased PaCOz
B. lncreased cardiac output D. Decreased fibrinogen level
3. What is a positive Hegar sign?
A. Ballottement of the fetus
B. Cervical nlucous ferning
C. Softening of the uterine cervix
D. Vasocongestion of the vaginal mucosa

4. Awoman who is 31-weeks pregnant comes to the emergency room with symptoms of
pre-eclampsia. The presence of which of the following signs would change this initial
diagnosis?
A. Hyperreflexia C. Proteinuria
B. Edema D. Seizure

5. Which of the following patterns noted on continuous monitoring of fetal heart rate is most
indicative of fetal distress?
A. Baseline variability with periodic acceleration
B. lncreasing baseline variability
C. Early deceleration without baseline variability
D. Late deceleration without baseline variability

6. The virus of Rubeola (Measles):


A. Produces congenital abnormalities exactly like those of Rubella
B. Produces severe congenital abnormalities
C. Has a relatively mild effect on the fetus
D. Attack the fetal cardiovascular system selectively

7. A S5-year-otd G2PO woman has been referred to an obstetrician after being recf,ntly
diagnosed with cervical carcinoma with parametrial infiltration. She and her husband walrt to
know what to expect in terms of management and prognosis. She is particularly concerined
about the treatment options she will need. The obstetrician explain to her that thid will
depend on the stage of her cancer. M/hat is the treatment option for this patientlwith
stage ll-B cervical CA?
A. Simple (total) hysterectomy
B. Radical hysterectomy )

C. Surgery and radiation


D. Chemoradiation

Continued on Page 2
PHYSICIAN Licensure Examination Page22
Page
__
Sunday, September 17, 2017 - O8:OO\ a.m. ,^.n^ ^ _
-. 1O:OO a.m.
OBSTETRICS AND GYNECOLOGY SET B

B. A female patient being treated for congestive heart failule is being treated well maintained
with Digoxin therapy. Furosemide is adcjed to the patient's drug regimen. The patient
subsequently develops signs of Digoxin, toxicity, as manifested by cardiac arrhythmia.
lndicate the most likely explanation for the development of digoxin toxicity:
A. Furosemide inhibits digoxin metabolism
C. Furosemide stimulates vagal input to th,e heart
D. Furosemide lowers intracellular cardiac calcium levels
i

9. A newborn girl presents to the neonatologist with an inability to swallow, coughing, choking
and vomiting when fed. Work-up reveals a tracheoesophaleal fistula, Which of the following
may have been present during ihe mothefs pregnancy with tnis child? , I

A. Abnormal vaginal bleeding


B. Oligohydramnios
C. Polyhydramnios
D. Pre-eclampsia
10.A4-year-oldgirl isbroughttotheemergencyroom by ambulance late at night. Her nanny
reports that the child refuses to bear weight on her left leg and reports that she fell down
the stairs this morning. X-ray of the leg shows spiral fracture of her left femur. Which of the
following is the most likely diagnosis?
A. Child abuse C. Osteosarcoma
B. Normal play injury D. Vitamin'D deficiency

11. Which one of the following findings is most likely to be present a patient with severe
magnesium deficiency?
A, Respiratory depression C. Tetany
B. Bradycardia D. Loss of patellar reflex

12. Twelve hours after an open hysterectomy, a 45 year old woman develops a fever of 39.5"C.
On examination, she has foul-smelling "dishwatef pus draining from her wound. A gram
stain reveals gram-positive rods. Which oflthe following statements regarding this condition
is true?
A. The causative organism is an aerobe
B. Emergency operitive debridement of th,e wound is indicated
C. High-dose flagyl (metronidazole) is the antibiotic of choice
D. The organism produces an endotoxin

l3.Apregnantwomancomestothephysicia,nforacheck-upbefore the beginning of her third


trimester. lt is learned that she has been exposed to an infectious disease. Fortunately,
the infections disease cause no morbidity to the fetus, and the resulting pregnancy is
uncomplicated. The woman later gives birth to a healthy child. To which of the following
pathogens was the woman most likely exposed?
A. Cytomegalovirus
B. Epstein-Barr virus
C. Herpes simplex virus
D, Toxoplasmosis

14. lmmediately following a difficult 3-hour open total abdominal hysterectomy and bilateral
salpingoophorectomy for ovarian cancer, a 35 year old woman could not extend her left
knee. Clinical examination is consistent with a femoral nerve palsy. Which of the following
statements about this condition is TRUE?
A. Robotic-assisted laparoscopic surgery increases the risk of this injury
B. lt is associated with external compression or leg positioning
C. Most patients are permanently disabled
D. Hand held retractors can minimize the incidence of this injury

Continu6d on Page 3
PHYSICIAN Licensure Examination Page 3
Sunday, September 17,2017 - O8:O0 a.m. - 1O:OO a.m.
OBSTETRICS AND GYNECOLOGY SET B

1 5. A 27-year-old woman presents with complains of foul-smelling vaginal discharge. On exam


she has a grey/green discharge and the cervix has punctate epithelial papillae, giving it a
"strawberry" appearance. On the wet prep a unicellular organism with flagella can be seen.
This organism is most likely:
A. Candida albicans
B. Trichomonas vaginalis
C. Gardnerella vaginalis
D. Bacteroides fragilis

16. At 22-weeks gestation where would you expect to find the uterine fundus?
A. Umbilicus C. Symphysis pubis
B. Xiphisternum D. Halfway between umbilicus and xiphisternum

17. A32-year-oldwoman,gravida 1, para O, with a history of infertility underwent ovulation


induction resulting in a twin pregnancy, now at 31 weeks gestation. An early obstetric
sonogram showed dichorionic placentation. She has a positive group B B-hemolytic
streptococcus vaginal culture. Because of epigastric pain, vaginal bleeding, and uterine
contraction, she is evaluated at the maternity clinic. An obstetric sonogram shows twin A to
be a female fetus in breech presentation and twin B to be a male in transverse lie with the
back down. The sonogram also shows a marginal anterior placenta previa. Her initial vital
signsareas follows: temperature,37.2"C: blood pressure 1651115 mm Hg. Her urine
dipstick test shows !+ glucose and 3+ albumin. Which of the following is contradiction to
tocolysis in this case?
A. Multiple gestation C. Severe pre-eclampsia
B. Marginal placental previa D. Early g'estational age

18. The normal pH of the adult vagina:


A. 3.8 - 4.5 c. 5.8 - 6.5
B. 4.8 - 5.5 D. 6.8 - 7.5
'19. Hypertonic dysfunctional labor generally can be expected to:
A. Be associated with rapid cervical dildtion
B. Cause little pain
C. Occurs in active phase of labor
D. Response to sedation

20. An 18-year-old primigravida atterm is completely dilated for2 hours with an L.O.T. (Left
Occiput Transverse) at a station plus 2. Treatment consist of:
A. Cesarean section C. Await spontaneous delivery
B. Forceps delivery D. Oxytocin stimulation
21. The embryonic origin of the vagina is from the:
A. Urogenital sinus and Mullerian duct I

B. Mullerian ducf and Wolffian duct


C. Mesonephric duct and Metanephric duct

22. A2L-year-oldfemaledrugaddict presenis with the following history of slight burning on


urination frequency and leukorrhea of 5 days duration. The discharge was noted to be yellow-
green and was irritating to the vulvar tissues. Examination reveals a reddened mucosa 6f the
urethral meatus and adjacent tissues. Pus can be expressed from the urethra and Skene's
glands. The cervix is inflamed. There is no history of pain or bleeding. The simplest laboqatory

A. Cervical biopsy :

B. Blood serology
C. Culture of the vaginal flora

Continued on Page 4
PHYSICIAN Licensure Examination
Sunday, September 17,2017 - O8:OO a.m. - 1O:OO a.m.
OBSTETRICS AND GYNECOLOGY SET B

23. A44-year-old caucasian female, married, presents herself to her physician complaining of
increase in sweating and frequent flushing of the face. Further history reveals that she is
Para 2-O-O-2 and ha! fraa normal menses all her life until three months ago when she skipped
a period. Her last two menses came at the right time but were scanty. General physical
examination including pelvic was negative. What is most likely diagnosis?
A. Carcinoid syndrome :

B. Cushing's disease
C. Menopause
D. Ovarian masculinizing tumor
, ,

24. A23-year-old primigravida in the third trimester of pregnancy is examined and found to hAvei
rapid weight gain, e]evated blood pressure, protein in tne urine, edema and headache' The
diagnosis-of [re-eclampsia is established. if tf,e patient's blood pressure suddenly rose to
above 140 mm Hg, initial management should include:
A. Bed rest, mild iedation, copi6us bowel evacuations, diet restriction
B. Phlebotomy, anti-hypertensive drugs, bed rest, sedation
C. Bed rest, sedations, Cesarean section within 24 hours
D. lV magnesium sulfate, bed rest, sedation; phlebotomy

25. A26-year-old multipara has delivered a 2.95 Kg infant after four hours of labor. The infant
was delivered from the vertex presentatit-rn by low forceps over a median episiotomy. The
patient received nitrous oxide anesthesia. The patient had little medication during labor.
The placenta was manually expressed. Following delivery the patient has postpartum
hemorrhage. The most likely base for the excessive bleeding is:
A. Duration of labor C. Type of anesthesia
B. Afibrinogenemia D. Retained placental fragments

26. A 24-year-old primigravida is admitted to the hospital at 32-weeks gestation with a history of
painless vaginal spotting for one hour. The patient describes the bleeding as being bright
red, and states that she has not bled previously. The most important immediate step to be
taken upon admission is:
A. Vaginal examination
B. Observation and evaluation with special attention to vital signs
C. Preparation for immediate Cesarean sPction
D. Determination of hematocrit

27. Allof the following statements concerning neural tube defects are true, EXCEPT
A. Elevated AFP in the amniotic fluid is a reliable but nonspecific marker for open neural
tube defects prior to the 26th week of pregnancy
B. Turner syndrome and duodenal atresia also results in elevated alpha-feto protein
C. Anencephaly is the most common and more severe type of neural tube defect
D. Type 1 maternal diabetes carries a higher risk of neural tube defect

28. During midline episiotomy, a cut is made through which of the following muscles?
A. Superficial transverse perineal and bulbocavernous
B. Pectineus and puborectalis
C. Coccygeus and deep transverse perineal
D. Bulbocavernous and external analsphincter
:

29. A 17-year-old girlwho is 6 weeks pregnant presents to the emergency room with abdominal
pain Lnd vaginal bleeding. Ultrasound shOws no fetal heartbeat and incomplele lfetal
developmentland the preglrancy is terminated by aspiration to remove the fetat tissue. On
questioning, the patient reports that she has been taking:high doses of her mother's i:lcer
medication for her own heartburn. Which of the f6llowing medications did this patient most
likely take?
A. Cimetidine C. Misoprostol
B. Magnesium hydroxide D. Omeprazole

Continued on Page 5
PHYSICIAN Licensure Examination Page 5
Sunday, September 17,2017 - O8:00 a.m. - 1O:OO a.m.
oBSTETRTCS AND GYNECOLOGY sEr B
30. 4 38-year-old woman presents to the physician with a 2-week history of aching pain in her
left calf that is made worse by dorsiflexion of her foot. On physical examination, her left calf
is found to be erythematous, warm, and swollen. Which of the following measures should:she
take to decrease similar problems in the future?
A. Begin taking statin
B. Begin taking low-dose oral contraceptives
C. Exercise 30 minutes three times per week
D. Quit smoking

31, A 26 - year - old woman visits her physician with complaints of vaginal bleeding after
sexual intercourse. She started menses at age 14 years and has 32-day cycles. She
acknowledges having unprotected sex with multiple partners. Cytologic specimens ,are
taken from the cerviiand vagina. On microscopy, cervical cells have large nucleiwith open
chromatin; several cells have mitotic figures. What would most likely be present in tthe
specimens that account for these findings? ,

A. Double-stranded DNA virus


B. Gram-negative diplococci ,

C. Gram-positive cocci
D. Single-stranded DNA virus

32. A65'year-old woman visits her physician because of increasing difficult urination. She has
trouble initiating a stream and experiences posfuoid dribbling. She wakes from sleep three
times per night to ufinate. Her baseline creatinine level was 1.0 mg/dl, and it is now 1.5
mg/dl. Which treatment is most feasible to immediately improve this patient's creatinine
level?
A. Administration of fluid bolus
B. Dialysis
C. Placement of a Foley catheter
D. Treatment with terazosin

33. A 27-year-old woman, gravida3, paraO, aborta2,comes to the outpatient office at


'15 weeks gestation by dates with complaints of exquisite vulvar pain and blisters. She
states she has had similar episodes prior to the pregnancy for the past 5 years. Her vital
signs are as follows:37 "C, pulse 95/min, respiration 2Olmin, blood pressure 124n5 mm Hg.
On examination, you find exquisite painfulvesicles on her left labia minora. lnguinal nodes
are negative bilaterally. She has a past history of a right Bartholin's abscess, which was
marsupialized. She had a positive cervical chlamydia culture on her first. prenatal visit.
Which of the following statement is true?
A. She should undergo a cesarean section to protect her infant from infection
B. Transplacental transmission to her fetus is a significant concern
C. Her fetus has an increased risk of ccrgenital malformation
D. Decisions regarding route of delivery are best made at the onset of labor

34. A 2O-year-old woman, gravida 3, para 2, lras a history of a seizure disorder since 10 years of
age. She is currently at 15weeks gestation and is being treated with phenytoin orally
three times per day. Her prenatal course was complicated by first-trimester bleeding that
spontaneously resolved with conservative management. A complete blood count (CBC)
was obtained as part of her routine prenatal laboratory tests. The following values are noted:
hemoglobin (Hgb) 9.3, hematocrit (Hct) 29, mean corpuscular volume (MCV) 105. Which of
the following is the most likely diagnosis?
A. Sickle celltrait
B. lron deficiency
C. Folate deficiency
D. Physiologic anemia

Continued on Page 6
PHYSICIAN Licensure Examination Pa$e 6
Sunday, September 17,2017 - 08:00 a.m. - 10:00 a.m.
OBSTETRICS AND GYNECOLOGY SET B

35. At a routine prenatal visit, a 28-year-old, gravida 5, para 4, at 28 weeks gestation reports
that she has not felt the baby move tor 2 days. Her pregnancy has been complicated by
chronic hypertension for which she is being treated with twice daily tablets of q-rnethyldopa.
On examination, you measure the fundal height at 30 cm, and Leopold's maneuvers reveal
the fetus in transverse lie. Her blood pressure is 145/85 mm Hg. You are unable to obtain
fetal heart tones with Doppler fetoscope. Which of the following is the most appropriate next
step in the management of this patient?
A. Perform a nonstress test
B. Obtain a real-time ultrasound assessmgnt for cardiac motion
C. Obtain a maternal abdominal x-ray assessment of the fetus
D. Perform an amniocentesis

36. A 64-year-old woman presents to the physician because of vaginal bleeding. Ultrasonography
reveals a small mass in the left adnexa, along with a thickened endometrial stripe. A biopsy
of her left ovary reveals the presence of Call-Exner bodies. Which of the following is the most
likely diagnosis?
A. Endometroid tumor
B. Granulosa cell tumor
C. Serous cystadenocarcinoma
D. Teratoma

37. An obstetric sonogram is performed on a 22-year-old woman, gravida 2, para 1, who is at 3O


weeks gestation by dates and confirmed by early trimester ultrasound fetal measurements.
Her pregnancy is cbmplicated by gestationil diabetes that is being managed by diet alone.
Her home blood glucose monitoring values have been within the target range. The sonogram
reveals a single fetus with the head in the right upper quadrant, back to the mother's left.
Both fetal thighs are flexed and both legs are extended. Which of the following is the correct
fetal presentition? '

A. Frank breech
B. Complete breech
C. lncomplete breech
D. Transverse breech

38. Growth hormone is essentialto normal human growth and development, and its secretion is
tightly regulated via a feedback control system involving the hypothalamus, the pituitary
gland,andtheperipheraltissues.Whichofthefollowing is a stimulus for the secretion of
growth hormone?
A. Hypoglycemia
B. Obesity
C. Pregnancy
D. Somatostatin therapy

39. A 22-year-old woman, gravida 5, para 2, aborta 3, has a history of substance abuse
prenatally. At 37 weeks gestation she underwent a vaginal delivery of a small-for-gestational
age male neonate with short palpebral fissures, epicanthal folds, flat midface, hypoplastic
philtrum, and thin vermillion border. These findings are characteristic in offspring born to
mothers who prenatally abused which of the following substances?
A. Tobacco
B. Alcohol
C. Marijuana

40. A woman is concern about her risks for developing cervical cancer. Which of the following
factors poses the largest risk for developing cervical cancer?
A. Alcoholism
B. Early sexual activity
C. Nulliparity
D. Prolonged estrogen use

Continued on Page 7
PHYSICIAN Licensure Examination
Sunday, September 17,2017 - 08:00 a.m. - 1O:OO a.m.
oBSTETRTCS ANp GYNECOLOGY SET B

41 . A21-year-old woman, gravida 1,para O, presents at 39 weeks gestation in active labor.


Clinical pelvimetry shows her pelvic dimensions as follows: pelvic sidewalls are straight,
ischial spine are not prominent, pubic arch is wide, sacrum is hollow, and sacrosciatic notch
is well rounded. Based on general bony architecture, the characteristics of this woman's
pelvis identify it as which of the following common female bony shapes?
A. Gynecoid
B. Platypelloid
C. Obstetroid
D. Anthropoid

42. A1 S-yearold girl presentstothe physician with underdeveloped breast and hirsutism. Her
medical histor! includes early appeirance of axillary and pubic hair and amenorrhea. On
examination her blood pressure is 90/55 mm Hg. The en4yme deficient in this patient is
needed for the production of which of the following hormones?
A. Aldosterone
B. Estradiol l

C. Progesterone
D. Testosterone :

43. A 37-year-old woman requests diagnostic testing before her frozen embryos are implanted.
The physician requests a work-up for cystic fibrosis, as both the patient and her husband are
carriers. The laboratory isolates some of the DNA from each 8-cell to 1O-cell embryo prior to
implantation and proceeds to run tests to determine the genotype of the embryos. Whiih of
the following procedures is the first step performed in determining the genotype of the
embryos?
A. Enzyme-linked immunosorbent assay
B. Ligase chain reaction
C. Northern blot
D. Polymerase chain reaction

44. Assess the safety of drug with establishment of dose level at which sign of toxicity appears:
A. Phase I studies C. Phase lll studies
B. Phase ll studies D. Phase lV studies
45. All of the following contribute to wrong site surgery, EXCEPT?
A. Administering sedation immediately before induction
B. lncorrect positioning
C. Anesthetic nerve blocks performed before time-out procedure
D. Not verifying consents
46. Which of the following is the key hormone in the luteal phase?
A. Estrogen
B. Progesterone :

c. T3&T4
D. Prolactin
47. Which of the following is the most common side effect of the long acting Depo-Provera?i
A. Dysmenorrhea
B. Menstrual irregularities
C. Euphoric mood
D. Weigh loss
48. Treatment of endocervical polyps include all of the foltowing EXCEPT
B. Dilation and curettage
C. Follow-up recurrence
D. Pathologic evaluation

Continued on Page 8
PHYSICIAN Licensure Examination Page 8
Sunday, September 17,2017 - 08:OO a.m. - 1O:0O a.m.
OBSTETRTCS AND GYNECOLOGY SET B

49. A 29-year-old nulligravida complains of severe pain with menses and inability to conceive
alter24 months of unprotected intercourse. On pelvic examination, her uterus is found to be
retroverted and there is nodularity of the uterosacral ligaments on rectovaginal examination.
Which of the following will be the most helpful in arriving at a diagnosis for this patient?
A. Laparoscopy
B. Physical examination
C. Hysterosalphingography
D. Culdocentesis

50. Pressure sores are BEST prevented by?


A- Routine use of air-flow mattresses
B. Frequent re-positioning
C. Aggressive nutritional support
D. Drying powders to moist skin surfaces

51. With a face presentation, the presenting part would be:


A. Occiput
B. Chin
C. Forehead
D. Nose

52. A woman who has been pregnant only one time and delivered twins at term by Cesarean
section is a:
A. Multipara C. Primipara
B. Nulli para D. Nulli gravida

53. Factors predisposing to endometrial aderrocarcinoma include the following, EXCEPT


A. Nulliparity
B. Endometrial hyperplasia
C. Oral contraceptives
D. Prolonged esirogen stimulation
54. Which of the following statements is NOT true?
A. Ovulation following delivery can occur as early as two weeks postpartum in a non-lactating
woman
B. Nursing mothers can ovulate before the fourth postpartum month
C. Contraception is not important until after the first menses postpartum
D. Contraception should begin immediately in a non-lactating woman

55. The most valuable clinicaltest for ovulation timing is:


A. The quantity, quality and spinnbarkeit of the cervical mucus
B. Daily vaginal smears

D. Fern phenomenon

56. At what stage of gestation would you expect a nulliparous women to begin to feel fetal
movements?
A. 18-20 weeks
B. 22-24 weeks
C. 10-12 weeks
D. 14-16 weeks
57. Marker of open neural tube defect:
A. Amniotic fluid acetylcholine esterase
B. Amniotic fluid acetylcholine kinase
C. Amniotic fluid HCG
D. Amniotic fl uid pseudoacetylcholine esterase

Continued on Page 9
PHYSICIAN Licensure Examination : Page 9
Sunday, September 17, 2017 - 08:00 ?.m. 'T 10:00 a.m.
OBSTETRICS AND GYNECOLOGY SET B

58. A67-year-old patientcomesto seeyou complaining of a bulging mass 18 months after a


vaginal hysterectomy. On examination, the vaginal mucosa is prolapsed. The most likely
diagnosis is:
A. Enterocele
B. Rectocele
C. Cystocele
D. Ureterocele

59. A 23-year-old woman has a blood sample drawn at 16 weeks'gestation by dates. The sample
is sent for maternal serum alpha-fetoprotein (MSAFP) screening. The result comes back as
Positive Low. Which of the following conditions is most likely associated with this findings?
A. Spina bifida
B. Anencephaly
C. Down syndrome
D. Turner syndrome

60. A Brenner tumor of the ovary is classified as:


A. Epithelial C. Germ cell
B, Stromal D. Mixed

61. Excessive intake (hypervitaminosis)of which of the following vitamin is associated with
increased risk of congenital malformations:
A. Vitamin A C. Folic acid
B. Biotin D. Vitamin K
I

62. A S2-year-old patient complains of loss of urine when she laughs, coughs or sneezes. The
patient had four vaginal deliveries and is in good health. All of the following statements are
true EXCEPT
A. Perineal exercises and estrogen therapy can alleviate these symptoms
B. Cystometrogram evaluation
C. Urine analysis and culture and sensitivity should be performed
D. Retropubic urethropexy surgery has a 5Oo/o success rate

63. How long does it take for carcinoma-in-situ to develop into invasive cervical carcinoma?
A. 1O years
B. 3 years
i

C. 7 years f

D. Unknown
64. A 3O-year-old female has had two viable pregnancies. Her children are two and four years of
age. This woman would be described as a:
A. Multipara
B. Parturient
C. Gravida
D.BothA&C
65. The most important pre - operative test to assess the risk of abnormal intra - operative
bleeding is?
A. Bleeding time
B. Activated partial thromboplastin time (aPTf)
C. lnternational normalized ration (lNR)
D. History and physical examination
i

66. All of the following are true of HCG, except


A. Has luteotrophic action
B. Acts on the same receptors as LH
C. Secreted by cytotrophoblast
D. lt is a glycoprotein ,

Continued on Page 1O
Sunday, September 17,2017 - 08:O0 a.m. - 1O:0O a.m.
OBSTETRICS AND GYNECOLOGY SEIT B

67. Which of the following anti-tuberculosis drugs is contraindicated in pregnancy?


A. Rifampicin
B. INH
C. Streptomycin
D. Ethamb,utol

68. Oxytocin is a drug widely used in obstetrics. Potential adverse side effects of oxytocin include
all of the following EXCEPT
A. Water intoxication :

B. Uterine atony
C. Hypertension ;, i
:

D. Fetal distress
,

69. All of the following are usually found in molar pregnancies, EXCEPT
A. Absence of fetal heart tones
B. Eclampsia
C. High gonadotrophic hormones levels
D. Proteinuria
70. A case of 3S-weeks pregnancy with hydramnios and marked respiratory distress is best
treated by:
A. lntravenous furosemide C. Amniocentesis
B. Saline infusion D. Artificial rupture of membrane

71. Macrosomia is NOT seen in: l

maturity
A. Post C. Obesity
B. Hydrocephalus D. Diabetes mellitus

72. A 23-year-old sexually active woman presents with fever, pain, and effusion in her left knee,
as well as scattered erythematous popular lesions on her fingers, toes and extremities. Which
of the following is the most likely diagnosis?
A. Secondary syphilis
B. Disseminated lymphogranuloma venereum
C. Disseminated gonococcemia
D. Reiter's syndrome

73 Chronic villus biopsy is done in all of the following, EXCEPT


A. Neural tube defects
B. Sickle cell disease
C. Myotonic dystrophy
D. Down syndrome

74. Gold standard test for beta-HCG levels?


A. Radioimmunoassay
B. ELISA
C. Latex agglutination test
D. Western blot

75. Which of the following infection control strategies has been shown to reduce methicillin
resistant staphylococcus aureus (MRSA) infection in surgical patients?
A. Universaladmission screening with standard cultures
B. Routine nasal decolonization
C. Routine pre-operative chlorhexidine showers
D. Routine hand washing

Continued on Page 11
PHYSICIAN Licensure Examination Page 1 1

Sunday, September 17,2017 - 08:00 a.m. - 10:00 a.m.


OBSTETRICS AND GYNECOLOGY . SET B

76. A second-time mother is in first stage of labor. After how many hours will the labor be
considered "prolonged"?
A. 8 hours
B. 12 hours
C. 14 hours
D. 18 hours
77. A pregnant woman is found to have excessive accumulation of amniotic fluid. Such
polyhydramnios is likely to be associated with all of the following conditions, EXCEPT
A. Twinning
B. Macro anencephaly
C. Esophageal atresia
D. Bilateral renal agenesis

78. Presumptive signs of pregnancy:


A. Fetal heart tones
B. Chadwick's sign
C. Skeleton on x-ray
D. Palpation of a fetus

79. Which animals are used in the Friedman Test?


A. lmmature mice
B. Rabbits
C. Toads
D. Frogs

80 The major natural estrogens produced by, women are the following, EXCEPT:
A. Estradiol (Estradiol-'178, Ez)
B. Estron (Er)
C. Ethinyl estradiol
D. Estriol (Es)

81. The most important factor in the diagnosis of puerperal infection is the existence of:
A. Temperature elevation
B. Foul-smelling lochia
C. Abdominal pain

82. ln instructing an expectant mother about potential danger signals during pregnancy, which of
the following should the woman be told to report immediately?
A. Blurred vision :

B. Facial swelling ',

C. Escape of fluids from the vagina


D. All of the above
)

83. The third stage of labor:


A. Begins with complete dilatation of the cervix and ends with the birth of the shoulders
B. Begins with the birth of the shoulders and ends with the birth of the placenta
C. Begins with the birth of the baby and ends with the expulsion of the placenta
D. Begins with the expulsion of the placenta and ends six hours later
84. tn Sheehan's syndrome, the most effective drug is:
A. Corticosteroid
B. Prolactin
c. Estrogen
D. Gonadotrophins

Continued on Page 12
PHYSICIAN Licensure Examination Pagd 12
Sunday, September 17, 2017 - O8:00 a.m. - 1O:0O a.m.
OBSTETRICS AND GYNECOLOGY SE'IT B

85. Least vital capacity is seen in:


A. Prone position
B. Left lateral position
C. Trendelenburg position
D. Lithotomy position
86. A pregnant female is diagnosed as heving abruption placentae. Which of the following
findings might be considered typical of severe abruptio placentae?
A. Painless vaginal bleeding
B. Decreased uterine tone
C. Tenderness of the uterus

87. All of the following are signs of fetal distress, EXCEPT?


A. Meconium with face presentation
B. Persistent irregularity of fetal heart
C. Meconium with vertex presentation
D. Meconium with breech presentation
88. X-ray pelvimetry is indicated in all of the following, EXCEPT
A. Severe CPD
B. Breech presentation in vaginal delivery
C. Outlet obstruction
D. Osteomalacia l

89. Respiratory function test were determined in a 24-year-old pregnant female. A significant
decline would be expected for which of the following?
A. Minute ventilation C. Minute oxygen uptake
B. Tidalvolume D. Residualvolume
90. A 24-year-old female presents with lower abdominal pain and tenderness, and vaginal
bleeding. A palpable mass is noted in the adnexal area of the uterus. She is tachycardic and
hypotensive. The patient likely has:
A. Polycythemia
B. Mittelschmerz
C. Ectopic pregnancy
i

91. A patient with a diagnosis of polycystic ovaries is LEAST likely to have: :

A. Diminished hair growth :

B. lnfertility
C, Amenorrhea
D. Functional bleeding

92. A 34-year-old female is diagnosed as having endometriosis. Which of the following symptoms
would be LEAST likely to be related to this disorder?
A. Abnormal uterine bleeding
B. Dyspareunia
C. Rectal pain
D. Constipation when menstruating :

93 A young female who is attempting to detect her tie of ovulation has been measuring her basal
body temperature. About how many days prior to expected ovulation would her basal
temperature shift upward?
A. 6-8 days
B. 9-10 days :

C. 14-15 days
D.24-28 days

Continued on Page 13
PHYSICIAN Licensure Examination Page 13
Sunday, September 17,2017 - 08:00 a.m. - 10:00 a.m.
OBSTETRICS AND GYN ECOLOGY SET B

94. A young female who recently found out that she was pregnant asked her physician when it
would be necessary for her to start taking supplemental iron. The correct response would:be:
A. During the first month of pregnancy
B. During the second month of pregnancy
C. During the third month of pregnancy
D. lt is not usually necessary for a pregnant to take supplemental iron during the first three
months because giving iron during this period could aggravate nausea & vomiting

95. A 25-year-old pregnant mother sees you regularly for pre-natal check-up. Her last menstrual
period was September 9, 2017 . When is her expected date of delivery?
A. May 16,2018
B. June 16, 2018
C. July 16, 2O18
D. August 18, 2018

96. Which of the following can be the etiology of either primary or secondary amenorrhea?
B. Anorexia nervosa .

C. Sheehan's syndrome
D. Kallmann's syndrome
i

97. Most fetal infections or damage commonly occur during this period:
A. Pre-implantation

D. After birth

98. Definitive management for pre-eclampsia:


A. Magnesium sulfate
B. Anti-hypertensives
C. Diuretics
D. Delivery of baby

99. A 21-year-old woman presentswith complaints of painful intercourse on vaginal entry since
she got married, as a virgin, 1 month ago. Which of the following conditions is the most likely
cause of her pain?
A. Narrow hymen
B. Psychogenic problem
C. Endometriosis
D. Pelvic inflammatory disease (PlD)
:

100. The potential cancer risk associated with using talcum powder:
A. Cervical cancer :

B. Ovarian cancer
C. Breast cancer
,

D. Endometrial cancer :

*** END ***

WARNING: Failure to submityour TestQuestions(Complete)set will cause the cancellation


of your Test-Result for this subject.
OBSTETRICS AND GYNECOLOGY 2017
ANSWER KEY

1. A 26. B 51. B 76. D


2.D 27. C 52. C 77. D
3.C 28. A 53. C 78. B
4.D 29. C 54. C 79. B
5.D 30. c 55. A 80. c
6.C 31. A 56. A 81. A
7.D 32. C 57. A 82. D
8.B 33. D 58. C 83. C
9.C 34. C s9. c 84. A
10. A 35. B 60. A 85. C
11. C 36. B 61. A 86. C
12. B 37. A 62. D 87. D
13. B 38. A 63. A 88. A
14. D 39. B 64. D 89. D
15. B 40. B 65. D 90. c
16. A 41. B 66. C 91. A
17. C 42. A 67. C 92. D
18. A 43. D 68. C 93. C
19. D 44. A 69. B 94. D
20. B 45. A 70. c 95. B
21. C 46. B 71. B 96. B
22. D 47. B 72. C 97. B
23. C 48. B 73. A 98. D
24. A 49. A 74_ A 99. A
25. D 50. B 75. D 100. B
Seat No.
Republic of the Philippines
PROFESSIONAL REGULATION COMMISSION
Manila
BOARD OF MEDICINE

PHYSICIAN Licensure Examination


Sunday, September'17, 2017
PEDIATRICS AND NUTRITION SET B
lNSTRUCTloN:Selectthecorrectanswerforeachofthefollowingquestions.
Mark onlv one answer for each item by marking the box corresponding to the letter of your choice
on tlre answer sheet provided. STRICTLY NO ERASURES ALLOWED. Use pencil no. 2 only.
MULTIPLE CHOICE:
1. An outbreak of sepsis caused by Staphylococcus aureus has occurred in the newborn
nursery. What is the most likely source of the organism?
A. Colon
B. Throat
C. Hand
D. Nose

2. The first clinical marrifestation of tetanus neonatorum usually is:


A. Fever
B. Vomiting
C. Spasm
D. Difficulty sucking and swallowing

A 3-year-old girl is stung by a bee while playing in the yard, and her mother finds her
unconscious and cyanotic. She is taken to the emergency department where epinephrine is
administered and the girl recovers. Which of the following mediated the girl's reaction to;the
sting?

D. lgM
,

4. Oral Rehydration Therapy uses:


A. Potassium to drive a sodium-potassium ATPase, leading to potassium and water re-
absorption
B. Glucose to drive an apical sodium-glucose cotransporter, leading to sodium and water
re-absorption I

C. Bicarbonate to counteract chloride losses via the cystic fibr.osis transmembrane


conductanceregulator(cFTR)'withnetwaterretention
D. ATP to ac-tively drive sodium and water re-absorption across the apical membrane
,

5. The following vaccine is contraindicated in a child who has an immunocompromised house-


hold contact:
A. Diptheria-Tetanus-Pertusis (DTP/DTaP)
B. Hemophilus influenzae, type b vaccine
C. Pneumococcal vaccine
D Oral polio vaccine (OPV)
6. Routinely recommended childhood vaccines should not be given in which of the following
clinical situations?
A. Minor upper respiratory tract infection
B.Child,smotherisinfirsttrimesterofpregnancy
C. History of possible seizure
D High fever with severe vomiting and diarrhea

Continued on Page 2
PHYSICIAN Licensure Examination Pagle z
Sunday, September 17,2017 - 1 1:OO ?.ffi. : O1:OO p.m.
PEDIATRICS AND NUTRITION SET B

7. The agent most commonly implicated as the cause of bronchiolitis is:


A. lnfluenza B
B. H. influenzae type b
C. Parainfluenza group of virus
D. Respiratory syncytial virus
8. ln Kernicterus (Neonatal Jaundice):
A. Phototherapy converts bilirubin to a nonpotar, water insoluble configuration
B. Phototherapy converts bilirubin to a polar, water soluble configuration
C. Phototherapy performs the same molecular action as UDP-glucuronosyltransferase
D. Conjugated hyperbilirubinemia causes injury to the basal ganglia and spinal cord
A contentious hypothesis focus on clinical and laboratory research though remain unproven
shows that non - genetic, environmental, post - infectious and psychological factors ban
influence the severity of Tourette's Syndrome. Which of the following infectious autoimmune
process can exacerbate obsessive<ompulsive disorder (OCD) and tic disorders cause; by
Tourette?
A. Post-streptococcal infection
B. Waterhouse-Friderichsen syndrome
C. Zika encephalitis
D. Hib meningitis '

10. The diagnostic facie seen in MPS-I Hurler syndrome and MPS-IV Morquio syndrome with
characteristic th ickening and coarsening facial features caused by subcutaneous deposi[ion
of mucopolysaccharide:
A. Bird facie C. Gargoyle facie
B. Adenoid facie D. Chiprnunk facie
11. A 3-year-old boy comes to the pediatrician with fever, conjunctivitis, erythema in the oral
mucosa, and cervical lymphadenopathy. The boy suddenly becomes hypotensive and Eioes
into cardiac arrest and dies shortly after. Autopsy shows aneurysmal dilations of the left
circumflex and right coronary arteries. The boy's disease is characterized as a self-limiting
disease that most commonly affects the coronary arteries. Which of the following diseases is
the correct diagnosis?
A. Buerger's disease C. Takayasu's arteritis
B. Kawasaki's disease D. Polyarteritis nodosa
12. A7-year-old boy is broughttothe emergency room after falling from a tree. An x-ray film
shows that he has a midshaft fracture of the humerus. Which of the following structures
could be injured with this type of fracture?
A. Axillary nerve
B. Median nerve
C. Radial nerve
D. Ulnar nerve
13, A 6-year-old boy is evaluated for kidney disease. The only significant past medical history is
streptococcal throat infection 3 weeks ago. Light microscopic evaluation of a kidney biopsy
is negative for any glomerular pathotogy. Blood tests demonstrate elevated levels of
cholesterol but decreased level of albumin. Urine test are positive for protein. On physical
examination, blood pressure is 11OI7O nrm Hg and there is pitting edema in the lower
extremities, as well as a small amount of periorbital swelling. The patient denies any pain in
urination. Which of the following is the most likely diagnosis?
A. Acute nephritic syndrome
B. Minimal change disease
C. Post infectious glomerulonephritis
D. Rapidly progressive glomerular nephritis

Continued on Page 3
PHYSICIAN Licensure Examination Page 3
Sunday, September 17,2017 - 11:00 a.m. - 01:00 p.m.
PEDIATRICS AND NUTRITION SET B

14. A2-week-old girl is brought to the pediatrician for her first doctor appointment. Physical
examination reveals a tuft of hair on her lower back. Ultrasound shows no herniation of any
kind. Which of the following conditions does this baby have?
A. Meningocele
B. Meningomyelocele :

C. Spina UiRaa cystica


D. Spina bifida occulata
; :

15. A 7 -year-old boy is brought to his pediatrician by his mother after a troubling parent-teaCher
conference. The teacheiinformed the mother that the child is disruptive in class and genedally
does not finish his homework assignments. The mother reports that the child's room is always
messy and that he has difficulty completing chores in a timely fashion. ln the office the ihild
is resiless and interrupts his mother often. Which of the following is the most likely diagno5is?
A. Antisocial personality disorder
B. Attention deficiUhyperactivity disorder
C. Conduct disorder
D. Separation anxiety disorder
16. What site is the first choice for intraosseous infusion?
A. Proximaltibia
B. Distal humerus
C. Proximalfemur
D. Distal fibula
17. A 6-year-old boy presents with multiple oval-shaped, coffee-colored macules on his right lower
abdomen. His mother also has a7.5 cm caf6-au-lait patch on her right hip and smaller spots
on her back that she refers to as freckles. Which of the following is the most likely diagnosis
in this boy?
A. Sturge-Weber disease
B. Tuberous sclerosis
C. Neurofibromatosis
D. Tinea versicolor
18. A3-year-old child presentswith convulsions followed by coma. Cerebrospinal fluid (CSF)
finding s include a total white blood cell WBC) count of 250 cells/prl with predominantly
lymphocytes and monocytes, glucose of 2O mg/dl. A Gram's stain of a spun-down sediment
is negative. A computed tomography (CT)'scan with contrast shows enhancement at the base
of the brain. Which of the following is the most likely diagnosis?
A. Subacute sclerosing panencephalitis
B. Tuberculous meningitis
C. Cryptococcal meningitis
D. Meningococcal meningitis
.1.
19. A pediatric resident is doing a thorough assessment on a 3-month-old infant expenenclng
increased intracranial pressure (lCP). Which of the following is a clear alarming sign of lbP?
A. Pinpoint pupil
B. High-pitched cry
C. Decrease in blood pressure
D. Tachycardia
20. Which of the following would alert the doctor to a possible developmental delay?
A. An 18-month-old who is crawling but not walking
B. A toddler who can't ride a bicycle
C. A 6-month-old infant who doesn't have a pincer grasp
D. An infant who doesn't sit up alone at 1 month

Continued on Page 4
PHYSICIAN Licensure Examination Page 4
Sunday, September 17,2017 - 1 1:OO a.m. - 01:O0 p.m.
.

PEDIATRICS AND NUTRITION SET B

21 . A lO-year-old boy is brought to the emergency room after becoming less responsive following
several bouts of nausea and vomiting. The patient is tachycardic and is breathing deeply
and slowly. Laboratory studies are remarkable for a serum pH of 7.21 , a serum glucose
level of 700 mg/dL. A serum bicarbonate level of 16 mEq/L, and a serum anion gap
of 22(normalT-'16). lntravenous fluids and insulin are administered. Measurement and
management of which of the following electrolytes are most critical in this patient?
A. Bicarbonate
B. Chloride
C. Potassium
D. Sodium

22. A parent of a child suffering from disturbed sleep and restlessness calls the family pediatrician
and states that her child is once again infected with pinworms and asks if she can administer
the same medicine that was used to cure an earlier infection. After the pediatrician is
convinced that the pinworm infection is the problem, she advises on giving the same treatment
and provides direction on how to clean up the environment to prevent further.reinfection. The
pediatrician should have been convinced by the fact that the parent:
A. Knew that reinfection was a possibility
B. Had collected worms from bed linen and accurately described them
D. Noted that the family's pet cat continued to sleep on the child's bed

23. A newborn has small whitish pinpoint spots over the nose which the birth attendant knows
are caused by retained secretions. This is called:
A. Milia C. White heads
B. Lanugo D. Mongolian spots
24. A previously healthy 1O-year-old boy develops excruciating abdominal pain and protracted
emesis of 3 days duration. The emesis is originally clear, but the day of admission the em'esis
becomes blood colored. The most likely diagnosis is:
A. Peptic ulcer disease C. Gastroenteritis
B. Hepatitis D. Pancreatitis
25. A 6-year-old child presents to their pediatrician with symptoms of fever, fatigue, and swollen
glands. Which of the following patient information would confirm a diagnosis of infection with
the mumps virus?
A. A history of exposure to mumps ,

B. Clinical evidence of orchitis


D. Resolution of fever foliowed by signs of encephalitis
i

26. A 1S-year-old known asthmatic presents to the emergency room with shortness of brdath.
He hjs used his inhaler three times in the past hour.Ilis respiratory rate is 34 with a lulse
oxygenation measurement of 92Yo. However, no wheezing is heard on exam. What isi the
most appropriate initial pharmacologic intervention?
A. Oral bronchodilators
B. Nebulized bronchodilators
C. Nebulized cromolyn
D. lntravenous steroids
27. A 9-year-old girl has a 2-week history of restlessness, episodes of excessive crying, and
increased difficulty writing her name. Magnetic resonance imaging of her head is normal.
Which of the following is the most likely diagnosis?
A. Complex partial seizure
B. Sydenham chorea
C. Hydrocephalus with increased intracranial pressure
D. Tourette syndrome

Continued on Page 5
PHYSICIAN Licensure Examination
Sunday, September 17,2017 - 1 1:00 a.m. - 01:00 p.m.
PEpIATRJCS AND NUTRITION SET B

28. The majority of non-immunized patients infected with poliovirus would be expected to
experience:
A. Flu-like illness
B. Aseptic meningitis
C. Muscle spasm and pain
D. Asymptomatic infection
29. The most common systemic fungal infectiop in immunocompromised patients with leukel'nia
is:
A. Aspergillosis
B. Candidiasis
C. Histoplasmosis
D. Coccidioidomycosis
3O. An eight-year-old girl has an abdominopelvic mass. The presence of alpha-protein in the
serum strongly suggest: r

A. Dermoid cyst
B. Endodermal sinus tumor
C. Embryonic carcinoma

31.Strawberryskullonfetatultrasoundpointsto:
A. Trisomy 13 C. Trisomy 21
B. Trisomy 18 D. All trisomies

32. A 2-year-old boy is brought to the pediatrician because his mother discovered a
large abdominal mass while bathing him. Examination show that the child has marked
hypertension and dark circles around his eyes. Urine catecholamine levels are elevated. A
biopsy of the mass reveals small round cells with hyperchromatic nuclei, often forming a
pseudorosette pattern around central primitive nerve fibers. Amplification of which of tthe
following oncogenes is associated with this tumor?
A. bcl-2 C. ras
B. n-myc D. c-myc
33. A 6-week-old boy is brought to the office in September with a 2-day history of cough and
increased work of breathing . Past medicarl history is significant with vaginal delivery without
complications at 38 weeks. At 1O days of age, he was treated with erythromycin ointment for
conjunctivitis. On physical examination, the patient is afebrile. He is coughing rapidly and has
a respiratory rate of 54 breaths/min. Diffuse wheezes are present throughout the lung fields
along with mild subcostal retractions. Which of the following is the most likely cause of the
patient's signs and symptoms?
A. Mycoplasma pneumoniae
B. Chlamydia trachomatis
C. Respiratory syncytial virus
D. Streptocoicui pneumoniae
i

34. An infant presents at the emergency room following a seizure that resolved without
intervention. She has a history of fever and two days of diarrhea. Her white blood count is
elevated, and a stool sample is full of mucus and streaked with blood, but she appears
nontoxic and is well hydrated. The most likely pathogen responsible for this child's disease is
Shigella dysentariae and is also associated with which of the following complications? l

A. Erythema nodosum
B. Failure to thrive
C. Cholera
D. Hemolytic uremic disease

Continued on Page 6
PHYSICIAN Licensure Examination
Sunday, September 17,2017 - 11:00 a.m. -'01:OO p.m.
PEDIATRICS AND NUTRITION SET B

35. All are associated with Down syndrome, except


A. Acute megakaryoblastic leukemia
B. Ventricular septal defects
C. Hirschsprung's disease
D. Chorea and mental regression
36. An B-year-old white male with chronic respiratory disease, pancreatic deficiency, and high
concentration of sweat electrolytes is most likely to have which of the following? :

A. Celiac disease C. Cystic fibrosis


B. Malabsorption syndrome D. Chronic pancreatiti"
I

37. A 12-year-old boy with classic hemophilia would most likely have an abnormal value for which
of the following tests?
A. Prothrombin time C. Bleeding Time
B. PTT D. Clot retraction time
38. A healthy premature infant who weighs 95O grams is fed undiluted breastmilk to provide
12O allKg per day. Over ensuing weeks the baby is most apt to develop: i

A. Hypernatremia i

B. Hypocalcemia
C. Blood in the stool
D. Metabolic acidosis I

39. A 2-year old male infant was noted to have intermittent episodes of cyanosis. He was nft"O
to assume a squatting position while playing. The mother claimed he was not a "blue baby" at
birth and he has no recurrent episodes of cough. A chest x-ray of the patient was done, Winicfr
of the following finding would be seen in this [atient? " i

A. Boot shaped heart i

B. Normal heart but with increased pulmonary vascularity


C. Heart described as egg on the side
D. Left ventricular hypertrophy
40. A newborn girl has been diagnosed as having phenylketonuria through newborn screening
test. Which of the following consequences will early initiation of dietary treatment protect this
child from?
A. Seizure disorder
B. Hearing loss
C. Hypoglycemia
D. Mental retardation
41 . Term infant 4-hours-old with excessive mucus, choking and cyanotic'episodes, hoarse cry
and regurgitation with feeding. The most likely diagnosis is:
A. Malrotation C. Esophagealatresia
B. Pyloric stenosis D. Hirsch6prung's disease
42. A7-year-old child is referred to your clinic because of declining school performance. There is
no known change in the child's life stressors. The teacher reports that he has falling asleep in
his classes. The grandmother notes that she begun sleeping in the same room with: him
because he snores so badly that he frequently stops breathing in his sleep and begins to
gasp. The tonsils appear quite large but not erythematous on exam, and the child does not
complain of throat pain. Obstructive sleep apnea is confirmed. What treatment is most likely

A. Continuous positive airway pressure


B. Oxygen therapy
C. Removal of tonsils and adenoids
D. Prophylactic antibiotic therapy

Continued on Page 7
PHYSICIAN Licensure Examination Page 8
E
Sunday, September 17,2017 - 11:00 a.m. - 01:00 p.m.
PEDIATRICS AND NUTR SEtr B

50. A 12-year-old boy is playing in the park near a trash can. He is stung by what he believes is
a yellow-colored bee. He immediately has symptoms of urticaria and wheezing. These
symptoms are treated in a local emergency department. He follows up in his primary care
provider's office 1 week later. Which of the following findings provides the best evidence of
insect sting hypersensitivity? :

A. Positive prick or intradermal skin test '

B. Extensive local reaction lasting 5-7 days


C. Documented evidence of a syitemic ailergic reaction
D. Generalized urticaria in this patient
,

51. A 1O-year-old boy presents with fever, hypotension, diarrhea, and diffuse erythroderma of
the skin. Nikolsky sign is absent. Laboratory studies reveal thrombocytopenia an( an
elevated creatinine kinase. Culture of his blood, urine and cerebrospinal fluid are negative.
Which of the following is the most likely cause of infection?
A. Group A B-hemolytic streptococcus
B. Staphylococcus aureus
C. Kawasaki disease
D. Escherichia coli 0157:H7
52. During a well-baby visit, the mother asks tlre pediatrician when she should start giving her
infant solid foods. The doctor should instruct her to introduce solid food first?
A. Egg whites
B. Rice cereals
C. Applesauce
D. Yogurt
53. When performing a physical examination on an infant, the doctor in charge notes abnormally
low-set ears. This findings is associated w,ith:
A. Tracheoesophageal fistula C. Renal anomalies
B. Congenital heart defects D. Otogenous tetanu"
,

54. A child is diagnosed with Wilm's tumor. During assessment, the doctor in charge expects to
detect:
A. Gross hematuria C. Nausea and vomiting
B. Dysuria D. An abdominal mass
55. Which of the following is the most important procedure in the diagnosis of subacute bacterial
endocarditis?
A. Complete blood count ,l

B. Erythrocyte sedimentation rate i


C. Blood culture I
D. Electrocardiogram
I
I

56. The doctor is examining a newborn who had undergone vaginal delivery. Which of the
I

following findings is least likely to be observed in a normal newborn? l

A. Uneven head shape


B. Respirations are irregular, abdominal, 3O-60 bpm
C. (+) moro reflex
D. Heart rate is 80 bpm
57. Neonatal infections that are partially or completely preventable include all of the following,
except
A. Cytomegalovirus
B. Group B streptococci
C. Rubella
D. Toxoplasmosis

Continued on Page,9
PHYSICIAN Licensure Examination
Sunday, September 17,2017 - 1 1:O0 a.m. -l O1:00 p.m.
PEDIATRICS AND NUTRITION set e
i

58, A child with seizure disorder is having recurrent seizures. She is currently arousable and is
spontaneously breathing but has gurgling upper airway sound. Appropriate measures would
include all of the following, except
A. lnsertion of an oropharyngeal airway
B. Providing oxygen with a face mask
C. Monitoring of oxygen saturation
D. Suctioning of the oropharynx
59. A S-year-old male chitd comes to the pediatric clinic for the first time with choreoatheloid
movements. The mother states that the child has becomes clumsier over the past few we6ks,
and there has been marked deterioration in the child's handwriting. The physician suspects
that the patient has rheumatic fever. ln addition to Sydenham's chorea, which of the following
is another major criteria for the diagnosis of acute rheumatic fever? :

A. lncreased anti streptolysin O (ASO) tiier


B. Arthralgia
C. Erythema migrans

60. What is the MOST common long-term sequela of bacterial meningitis in children?
A. Seizure disorder
B. lntellectual disability
C. Hearing loss
D. Hemiparesis .

61. A mother brings her 4-year-old child to the emergency department for evaluation of
persistent fevers. For the last week the yor:ng girl has suffered from spiking fevers and
malaise. The mother is concerned that she may have caught something while visiting family
in Japan lmonth prior, though the child h:is no known sick contacts. Past medical history
and family medical history are unremarkahle. Aside from the usual childhood illnesses, the
patient has been well. She is up to date on all appropriate immunizations. Phy$ical
examinations shows bilateral conjunctival injection and dry, cracked mucous membranes.
Her lips are fissured and her tongue is ert'rhematous and swollen. She has marked cervical
lymphadenopathy, with at least one tender node of approximately 2 cm. Additionally, across
her chest is a pleomorphic, erythematous rash. Erythema is also present on the soles of her
feet and palms of her hands, as is some mild desquamation. Laboratory studies show a
normocytic anemia, a thrombocytosis, and a markedly elevated erythrocyte sedimentation
rate. Blood and urine cultures are unrevealing. Given this patient's presentation, which of
the following is the most appropriate treatment?
A. Acetaminophen
B. Aspirin and intravenous imrnunoglobulin
C. Plasmapheresis
D. Prednisone
62. A 3-year-old child is brought to the emergency room by DSWD representatives for suspected
physical abuse. The physician on duty performs a thorough history and physical examination
to determine evidence of maltreatment. Which of the following findings is NOT consistent
with physical abuse?
A. Patterned markings resembling slap marks, grasp marks and human bites
B. Linear buttock bruising
C. Bruises at many stages of healing
D. Bruises on the shins, knees and elbows
63. A 19-year-old adolescent comes to the clinic because of a painful outbreak on his genitals
that started 2 days earlier. He also complains of malaise and low-grade fever that is partially
responsive to aspirin. Reluctantly, he admits that about a week before this illness he had
sexual intercourse with a woman he met at a localfraternity party. He used a condoni but
noticed after their encounter that it had ripped at the tip. He is in good overall health and works

Continued on Page 1O
PHYSICIAN Licensure Examination Page 10
Sunday, September 17,2017 - 11:OO a.m. -' O1:OO p.m.
PEDIATRICS AND NUTRITION SET B

out on a regular basis. On physical examination, he is a well-developed, fit young man with a
temperature of 37.9C and a pulse of 9O/mirr. His respiratory rate and blood pressure are with-
in normal limits, lnspection of the genitals reveals multiple tense, grouped vesicles with an
erythematous base on the glans and shaft rrf the penis. There are also several painful small
ulcerations. The inguinal lymph nodes are symmetrically enlarged bilaterally, and tender to
palpation. Which of the following diagnostic studies is indicated at this time?
A. Bacterial culture
B. Skin biopsy
D. Tzank preparation ,

64. An obese teenager was brought to the emergency room complaining of chest pain. The
doctor on duty ferforms a thorough examination of the patient. Which of the following
symptoms would be MOST characteristic Of an acute myocardial infarction?
A. Colic-like epigastric pain
B. Sharp, well-localized, unilateral chest pain
C. Severe substernal pain radiating down the left arm i

D. Sharp, burning chest pain moviig from place to place ,

65. A pediatric patient was brought to the clinic for evaluation of acute onset of seizured. A
thorough history and physical examination is performed. The doctor should request which of
the following diagnostic tests FIRST?
A. Magnetic resonance imaging (MRl)
B. Complete cerebral angiography (CCA)
C. Electroencephalogram (EEG) :

D. Electromyogram (EMG) ,

66. The laboratory results of a patient with disseminated intravascular coagulation are likely to

[:'Bt?"rsed tevels of fibrin split products


B. lncreased levels of fibrinogen
C. lncreased platelets counts
D. Prolonged prothrombin and partial thromboplastin times
67. A 5 - year - old boy has dysuria , fever, leukocytosis, and right flank pain. On physical
examination he is tender to fist percussion over the right costovertebral angle. Urinalysis
shows grossly infected urine. He is treaterd with appropriate antibiotics and the infection
clears. This is verified by repeated negative urinary cultures. The family indicates that he
has had two previous similar episodeJ that were aiso successfully treated with antibiotics.
No further workup was done following those two episodes. The reason for this recul-:rent
problem is most likely to be diagnosed with which of the following studies?
A. Cystoscopy C. Renal ultrasound
B. lntravenous pyelogram D. Voiding cystourethrogram
68. A 2-year-old boy is brought to the emergency department because of noisy breathing
for the last 2 days. He was well until 3 days ago when he developed a fever up to 38.5 "C,
cough, and runny nose. One day later, the breathing became noisy with a loud sound
during inspiration, which is more prominent during night. He is 'able to eat and drink
fluids and has no vomiting. There is no previous history of similar episodes or chronic
cough or choking. On eiamination, the child has mild respiratory distress with a'loud
inspiratory stridoi. He is alert and afebrite, and orygen saturation is 98% in rooni air.
Examination reveals good breath sounds on both sides and no wheezing. There is no
cardiac murmur and abdominal examination is normal. Chest x-ray shows that the subglottic
space is narrowed and resembles a pencil point. Which of the following is the most likely
causal organism?
A. Haemophilus influenzae C. Respiratory syncytial virus
B. Parainfluenza virus D. Streptococcus group A

Continued on Page 11
PHYSICIAN Licensure Examination Page 11
Sunday, September 17,2017 - '11:00 a.m. -- O1:0O p.m.
PEDIATRICS AND NUTRITION SET B

69. A diabetic 33-year-old G1P1 type O+ mothier delivered 5 weeks earlier than her expected
date of confinement. The baby was delivered in a lying-in clinic and was noted to be limp,
cyanotic face and pale body, gasping, puls'es weak and no response on suctioning. lf you
are present on the delivery of this baby. The Apgar score at 1 minute is:
A.O
B. 1
c.2
D.3
i

70. A 16-year-old girl is brought to the emergency department by paramedics on a Satuiday


night. She is not accompanied by a family member, and is lacking available history. Physical
examination reveals delirium, dilated pupils, dry and flushed skin and mucous membraries,
peculiar myoclonus-like twitching, tachycardia, and fever. Her clinical condition appears to
be deteriorating during a period of 15 minutes of observation. The attending physidian
suspects that the patient is having a reaction to an ingested drug. Which of the following
medications would be most appropriate to treat this patient?
A. Edrophonium C. Phenothiazine
B. Phenobarbital D. PhySostigmine
71 . A dietary risk factor for iron deficiency anemia is:
A. Less than 3 months of breastfeeding
B. Late institution of solids
C. Exclusive breastfeeding for more than 1 year
D. A soya-rich diet
72.AS-year-oldboyisbroughttothephysicianfor evaluation. The parents complain that he
has trouble maintaining his balance and has started walking in a peculiar, waddling way as
of lately. He pushes his knees when he wants to stand up and has developed large calf
muscles that seem to stand out from the rest of his body. His medical history is siQnifidant
for several episodes of pneumonia. He is on no medications at this time and has no allelrgies
to medication. The famiiy history is unremarkable. On physical examination, the child is in
no acute distress. His vital signs are within normal limits. He has difficulty generating a
forceful cough for chest auscultation. lnspection of the trunk and extremities reveals very
pronounced calf muscles. Deep tendon reflexes are significantly diminished. There is mild
kyphoscoliosis secondary to paraspinal muscle weakness. Which of the following studies is
most likely to aid in confirming the diagnosis?
A. Electrocardiogram
B. Electromyography
C. Muscle biopsy
D. Serum creatinine phosphokinase
l

73. A 14-year-old girl is brought to the clinic for a routine school physical examination.
The mother is concerned that her daughter has shown no signs of breast developrhent
and has not started menstruating. Past history does not reveal any significant illnesses.
Review of the patient's growth reveals she was always just below the 5th percentile until
she was 1O years old. However, hercurrent height iswell belowthe Sth percentile with an
absence of pubertal groMh spurt. Physical examination reveals a short girl with ndrmal
facial appearance. There is a prominent cubitus valgus deformity. The neck is short andi the
posterior hairline is low. There is no breast development, although some pubic and axillary
hair is present. Examination of the heart, lungs, and abdomen reveald no
abnormality. Which of the following studies is most likely to reveal the etiology of this
patient's short stature?
A. Abdominal ultrasound i

B. Gonadotropin level :

C. Growtlr hormone level


D. Karyotype

Continued on Page 12
PHYSICIAN Licensure Examination Yage 12
Page
Sunday'September17,2o17-11:00a.m'-01:00p.m.
PEDIATRIcS AND NUTRITION SET B

74. A 9-month-old boy is brought to the physician because the mother noticed "orange sand" in
his diapers over the previous several days. The child has been in good health, although a
little slow to develop his milestones. He has no significant past medical history and takesl no
medication. The family history is unremarkable. On physical examination, the patient is in no
acute distress. His temperature is 36.7 "C, pulse is 95/min, and respirations are 2Olmin. The
weight and length are in the 20th and 1Oth percentiles, respectively. He sits with assistance
and lifts his upper body when in a prone position, but does not attempt to crawl or stand,
lnvoluntary movements indicative of extrapyramidal dysfunction are noted. There lare
prominent calluses on both hands and you noi6 tn.t tne ifrita repeatedly gnaws on, thlose
areas. A complete blood count and chemistry panel reveals macrocytic anemia '5nd
hyperuricemia. in addition, which of the following studies is most likely to be abnormall in
patients with this condition?
1

A. Cerebrospinal fluid analysis


B. Computerized tomography of the brain
C. Plain abdominal radiograph
D. Ultrasound of the abdomen j

75. You are caring for a 14-year-old adolescent boy with trisomy 21. Which of the following test
is MOST appiopriate to order on an annual basis, as part of the regular health supervision for
this adolescent?
A. ECG
B. Thyroid function
C. Sleep oximetry
D. X-rays of the cervical spine
76. A patient was diagnosed with Tetralogy of Fallot. His chest x-ray reveals the classic "Le Couer
en Sabot". Which of the following components of TOF is responsible for this shape?
A. Pulmonary stenosis C. RVH
B. VSD D. Overriding of the aorta
77. One of the following drug is ABSOLUTELY contraindicated to breastfeeding:
A. Diuretics
B. Prednisone
C. Tetracycline
.

D. Hydralazine i

78. A S-year-old girl developed fever, coryza, and conjunctivitis. After 5 days, still with fAver,
she was noted to have maculo-papular rashes and cervical lymphadenopathy. What is your
diagnosis?
A. Rubella
B. Roseola
D. Rubeola
79. A 1S-year-old malewith Peutz-Jegher syridrome complained of severe crampy abdominal
pain with visible peristaltic waves noted over the abdomen and a palpable mass at the
epigastric area. He most likely developed:
A. Volvulus
B. Gastric perforation
C. lntussusception

80. All of the following are associated with Klinefelter syndrome,


A. 47 XXY karyotype
B. Mental retardation
C. Smalltestes and penis
D. Precocious puberty

Continued on Page 13
PHYSICIAN Licensure Examination ragel rr
Sunday, September 17,2017 - 11:00 a.m' - 01:00 p.m. :

PEDIATRICS AND NUTRITION


:

81. Large numbers of spherocytes on the blood film are found in all of the following disorders,
EXCEPT
A. lmmune hemolYtic anemia i
B. Wilson's disease

D. Sickle cell anemia

82. A2-year-old 12 Kg child has dengue hemorrhagic fever stage 2 and has isotonic dehydration.
Which of the following fluids should be administered?
A. Ds LR C. Ds NSS
B. Ds 0.45Nacl D. Plain NSS
83. What is the most common cause of death in measles?
A. Encephalitis
B. Acute otitis media
C. Myocarditis
D. Pneumonia
84. A 4-year-old child, whose family follows a strict vegetarian diet, undergoes a-prekindergarten
physical examination. The school physician is alert to the child's potential deficiency of: i

A. Niacin C. Vitamin 86
B. Thiamine D. Vitamin B12
l

85. Solanine intoxication follows ingestion of:


A. Mushroom of the genus Amanita
C. Oysters
D. Peanut butter
86. The physical manifestations of fetal alcohol syndrome include:
A. Cleft lip and palate
B. Hepatomegaly, hypotonia and microphthalmia
C. Hyperbilirubinemia, jaundice and failure to thrive
D. Microcephaly, short philtrum and prenatal groMh retardation
87. An 8-year-old child complains of recurrent vomiting some time associated with fever for last
three years each episode lasts for trn,o to three days. There is no metabolic abnormality or
neurologic deficit found. Choose the right answer:
A. Endoscopy should be done
B. H. pylori is the cause
C. Propranolol is treatment of choice l

D. Drug poisoning may be the cause


:

88. A child who suddenly develops difficulty of breathing, urticarial rashes, abdominal ,pain
5 minutes after eating crabs need immediate administration of:
A. lntramuscular antihistamine
B. lntravenous antihistamine
C. lntramuscular corticosteroids
D. lntramuscular epinephrine
89. You're evaluating a 4-month-old infant who presents with fever of 39.4 "C and tonic-Clonic
seizure. There'sio prior history of seizures or epilepsy. A lumbar puncture is unreve{ling.
Which one of the following is the likely cause of this infant's seizures?
'

B. Encephalitis :

C. Malignancy
:

D. Fever

Continur:d on Page 14
PHYSICIAN Licensure Examination Page 14
Sunday, Septernber 17, 2017 - 1 1:OO a.m. - O1:0O p.m.
PEDIATRICS AND NUTRITION SET B

90. What is the most significant serious complication arising from Kawasaki disease?
A. Coronary aneurysm :

B. Kidney failure
C. Arthritis
D. Gastrointestinal bleeding
91 . An infant is brought to the pediatrician clinic because the mother states that the baby ha! a
rash, which is extremely pruritic. Further questioning reveals that the mother also has a
pruritic rash. The physician determines the diagnosis to be scabies. Which of the folloWing
best describes the manner in which infantile scabies differs from adult scabies?
A. lt has no burrows
B. It is not limited to the intertriginous areas
C. lt is often presents with red-brown nodules in the groin i

92. Which of the fotlowing statements about administering Vitamin K to a newborn is correcti
A. lnfant need Vitamin K to develop immunity ',

B. Vitamin K will protect the infant from being jaundiced


C. Newborn infants have sterile bowel and Vitamin K promotes the growth of bacteria in the
bowel
D. Newborn infants are deficient in Vitamin K and thus needed to prevent infant from
abnormal bleeding
93. The following is not included in the Newborn Screening Test in the Philippines:
A. Glucose-6-phoshate dehydrogenase deficiency
B. Homocystinuria
C. Galactosemia
D. Maple syrup urine disease
94. Bedtime ritual are particularly important to which of the following age group?
A. Preschooler C. School-age
B. Toddler D. Adolescence
95. lf polio infection is suspected, the virus can be isolated in what type of specimen:
A. Blood
B. Urine :

C. Stool )

D. CSF fluid
96. A 4-month-old infant was brought to the hospital because of diarrhea. He passed out weitery
yellow stools with low grade fever. The most likely cause of the acute diarrhea is: l

A. ETEC
B. Rotavirus
C. Cholera
D. Shigella
97. The most important index of protein malnLrtrition at present is:
A. Edema
B. Wasting
C. Negative nitrogen balance
D. Slow growth and development
98. Most common foreign body seen in the esophagus:
A. Pins
B. Walnuts
C. Candy
D. Coins

Continued on Page 15
PHYSICIAN Licensure Examination
Sunday, September 17,2017 - 1 1:00 a.m. - O1:00 p.m.
PEDIATRICS AND NUTRITION
99. ln the Philippines, the leading cause of fatal injuries for school age groups is:
A. Motor vehicle accidents
B. Poisoning/Overdoses
C. Gunshot wounds
D. Drowning
1OO. The universal language of children: i
A. Play
B. Music
C. Animation
D. "Mama", "Dada"
**r END ***

WARNING: Failure to submit your Test Questions (Complete) set will cause the cancellation
of your Test-Result for this subject.
PEDIATRIC AND NUTRITION 2017
ANSWER KEY

1.C 26. B 51. B 76. C


2.D 27. B 52. B 77. C
3.8 28. D 53. C 78. D
4.8 29. B 54. D 79. C
5.D 30. B 55. C 80. D
6.D 31. B 56. D 81. D
7.D 32. B 57. A 82. C
8.8 33. B 58. A 83. D
9.A 34. D 59. D 84. D
10. c 35. D 60. c 85. B
11. B 36. C 61. B 86. D
12. C 37. B 62. D 87. C
13. B 38. B 63. D 88. D
14. D 39. A 64. C 89. D
15. B 40. D 65. C 90. A
16. A 41. C 66. D 91. A
17. C 42. C 67. D 92. D
18. B 43. C 68. B 93. B
19. ts 44. B 69. D 94. A
20. A 45. C 70. D 95. C
21. C 46. A 71. C 96. B
22. B 47. C 72. C 97. A
23. A 48. B "< 73. D 98. D
24. D 49. A 74. D 99. A
25. C 50. c 75. B 100. A
Republic of the Philippirres
PROFESSIONAL REGULATION COMMISSION
Manila
BOARD OF MEDICINE

PHYSICIAN Licensure Examination


Sunday, September 17, 2017
PREVENTIVE MEDIC]NE AND PUBLTC HEALTH STT B

INSTRUCTTON: Select the correct answer for each of the following questions. r

Mark onlv one answer for each item by marking the box corresponding to the letter of your choice
on the answer sheet provided. STRICTLY NO ERASURES ALLOWED. Use pencil no. 2 only.
f
CHOICE:
I

MULTIPLE
.

1 . Close supervision of contacts for prompt recognition of illness without restricting movemeint
A. quarantine
B. surveillance I

C. detention !

D. segregation
,

2. When a food product is sold fraudulently under the name of another it is said to be .

A. misbranded
B. adulterated i

C. trademarked
D. blackmarketed
3. The distance between the high and low scores in statistics
A. range
B. mode
C. median
D. standard deviation
4. Observation of events in the community are presented in the form of tables/graphs. Which of
the following graphs is best to use in presenting trends of births and death rates over period
of time?
A. pie chart C. line graph
B. bar graph D. frequency polygon
5. lnfection of this disease usually confers permanent immunity
A. amebiasis C. malaria
B. leprosy D. diptherra
6. lf infant mortality rate is high, emphasis should be given to
A. communicable disease control
B. nutrition program
C. maternal and child health
D. immunization program
7. ln evaluating the success of your environmental sanitation program, the most significant
criterion is
A. number of toilets built
B. amount of money donated
C. reduced incidence rate of diarrhea among 0-6 years old
D. number of families involved in the program

8. Suspected relationship between customs and habits of people and the incidence of spe-cific
types of cancer exclude i

A. chewing betel nut (Philippines) and oral cancer ,

B. application of cosmetics (Hollywood, USA) and risk of skin cancer


C. circumcision ritual (lsrael) and decrease incidence of penile cancer
D. wearing kangri basket (lndia) and Bowen's disease

n^-^ -
^--ri-..^J --
PHYSICIAN Licensure Examination Page 2
Sunday, September 17,2017 - O2:OO p.m. - O4:OO p.m.
PREVENTIVE MEDICINE AND PUBLIC HEALTH SET B
9. Pasteurization of milk is what level of prerention
A. Primordial level
B. Primary level
C. Secondary level
D. Tertiary level
10. During the polio virus scare in the late 1930s and 194Os, panic arose over the public's
fears that children could be exposed to polio virus in
A. schools
B. churches
C. theaters
D. swimming pools
1 1. The occurrence in a community of a disease clearly in excess of the normal expectancyl
A. endemic
B. pandemic
C. sporadic
D. epidemic
12. Which of the following is not an example of nominal data?
A. name C. sex
B. age D. race
13. Most frequently acquired injury at the workplace
A. muscle strain
B. abrasion or cuts
C. fracture
D. sprain
14. Which of the following vaccines is a form of passive immunization?
A. Typhoid C. Hepatitis B
B. Varicella D. Tetanus antitoxin
15. Officially known asTransforming OurWorld 2030. This 17 sets of goals adopted by UN
member countries envisioned to end poverty, protect the planet and ensure prosperity frbr all:
A. Millennium Development Goal
B. High lmpact Goal
C. Sustainable Development Goal
D. Universal Healthcare Goal
16. Poor absorption and low dietary intake are relative causes of iron deficiency anemia as,per
findings of the National Nutrition Survey. Who among these indigenous ethnic groups has
the highest prevalence of iron deficiency?
A. Aetas C. Mangyan
B. Lumad D. Tausug
17. lnjuries or illnesses such as cancer cases requiring chemotherapy or radiotherapy are
Philhealth classified under
A. .intensive cases
B. catastrophic cases
C. ordinary cases

18, The 3 distinct Vpes of polio virus


A. Salk, Sabin, Landsteiner-Popper
B. Brunhilde, Lansing, Leon
C. TgPVR 1, TgPVR21, TgPVR 5
D. Piolo Type l, Piolo Type ll, Piolo Type lll

Conlinrrad on Paoa 3
PHYSICIAN Licensure Examination
Sunday, September 17,2017 - O2:OO p.m. - O4:OO p.m.
.--:-
I

PREVENTIVE MEDICINE AND PUBLIC HEALTH SET B


i

19. The industry with the highest accidental death rate, and long held to be the rnost dangeroLrs
occupation
B. construction sector
C. mining and quarrying sector
D. service sector
20. The delivery of health care services in the municipal and barangay level is the responsibility
of the
A. Department of Health
B. Provincial Health Offices
C. Local Government Units
D. Respective Communities
21. Which of the following is not an A-Zprograrn o:f the Departrnent.of Health
A. 3'#,iJ3 r"Hl""": l,?';?[,..
C. government procurement program
-
D. woman and children protection program
i

22. The mosquito that transmit dengue feve: may bite anytime during the day but it is peak hours
of biting is
A. early morning and late afternoon
B. night time
C. early morning ;

D. late afternoon l

23. The most important indicator of the state of the world's children since it reflects maternal
health, level of immunization, ORT use, availability of maternal & child health services, inbome
& food availability in the family, availability of clean water & safe sanitation and the overall
safety of the child's environment
)

B. specific mortality rate


C. child mortality rate
D. under five mortality rate
24. Which one of the following scale of measurement is a mismatch to its variable?
A. nominal- blood type
B. ordinal - response to treatment
C. interval - family size
D. ratio - height
25. Balantidiasis in man has been recorded from most parts of the world and is caused by infection
with the ciliated protozoan Balantidium coli. Which of the following is a common reservoir of

A. wild boars C. burrowing rats


B. domestic pigs D. grazing sheeps
26. Which is not the true reason for cyclic trend of a disease
A. antigenic variation
B. buildup of susceptible
C. herd immunity variation
D. environmental conditions
i

27. A man who has undergone vasectomy at a local Family Planning Clinic is considered sferile
A. after 25 ejaculations l

C. imm6diately after the procedure


D. after a zero sperm count

Continued on Page 4
PHYSICIAN Licensure Examination
Sunday, September 17,2017 - O2'.OO p.m. - O4:OO p.m. i

PREVENTIVE MEDICINE AND PUBLIC HEAI-TH SET B

28. Herbal plant which the families from far flurrg barrios can use for swollen gums and toothache
A. herba buena
B. tsaang gubat
C. ampalaya leaves
D. lagundi
29. ln the iceberg phenomenon of morbidity assessment, which of the following treatment
modality can be seen at the "tip" of the pyramid?
A. treatment by quacks and charlatans
C. state-of-the-art treatment center :

D. treatment by traditional healers


30. A given set of observation are 1O,7 , 5, 9, 2 and 1 1. What is the median of this given set,of
observation? l

A. 10.5 c. 11.5
B. 11 D. 10
31. ln control of communicable diseases, the period of quarantine in respectof a disease is'
determine by +

B. carrier state I

C. incubation period )

D. duration of illness
32. Violence and injury prevention programs of the D.O.H. includes the following, except
A. zero fireworks related injuries
B. stop bullying in schools promotion
C. end mental health stigma & discriminiation
D. suicide prevention hotline
33. Which of the following is the backbone in disease prevention?
A. epidemiology
B. demographics
C. vital statistics
D. health statistics
l

34. The intensity of noise causing immediate damage to the ear is at the sound level of
A. 80 decibels
B. 100 decibels
C. 12O decibels l

D. 60 decibels :,i

35. The most common method of community diagnosis that accounts for the bulk of data isr
A. census
B. records review
C. interview
D. sample suruey
36. Which strain is used for BCG vaccine?
A. Yamagata-1 strain
B. Jeryl Lynn strain
C. La Sota strain
D. Danish 1331 strain
37. Which of the following is a negative health behavior that must be avoided to prevent illrJess?
A. immunizalion
B. adequate sleep pattern I

C. self-medication
D. regular check-up
(lnnfinrrori an Pana 6
,,x.i

tii:,:irl

PHYSICIAN Licensure Examination Page 5


Sunday, September 17,2017 - 02:00 p.m. - O4:OO p.m.
PREVENTIVE.MEDICINE AND PUBLIC HEALTH SET B
38. Few diseases are subject to notification on the basis of internationalagreements. The
internationally notifiable diseases known also as quarantinable or convention diseases are
governed by lnternational Sanitary Regr.:lators. The diseases currently notifiable to the World
Health Organization are the following, except
A. plague
B. cholera
C. malaria
D. yellow fever
39. A more or less complete count of individuals within a given geographic area at specified in stant
of time
A. census
B. survey
C. demography
D. population register
., of dehydration in diarrhea is : )

f.J:isl=,:r,:
B. thirst
C. loss of consciousness
D. positive skin fold test
41. Chimney sweep's cancer is the first cancer linked to occupational exposure and development
of scrotal squamous cell CA. The cancer primarily affect chimney sweepers who had been in
long contact with what substance?
A. tar
C. soot i

42. Which program is an example of secondary prevention?


A. A community-wide nutrition program at fast-food establishments
B. A program to immunize persons exposed to infectious disease with immunoglobulin G
within 2 weeks of exposure
C. Alcoholics Anonymous
D. An exercise program for persons who have had a stroke
43. lodine rich food
A. squash
B. seaweed :

C. carrot
D. meat
44. Which of the following tell us how many babies born alive would die before reaching one year
of age in a given place and time
A. specific child death rate
B. infant mortality rate
C. neonatal mortality rate
D. crude death rate
45. To establish the existence of a relationship between a risk factor and a disease, the
investigator str-rdied those who are sick and those that are not sick. The study is called
A. retrospective C. cross-sectional
B. prospective D. experimental
46. Which of the following micronutrients is given to a child at g months of age, preschoolers and
newly-delivered others?
A. iron C. iodine
B. fluoride D. vitamin A

Contir;ued on Page 6
PHYSICIAN Licensure Examination
Sunday, September 17,2017 - O2:OO p.m. - O4:OO p.m.
PREVENTIVE MEDICINE AND PUBLIC HEALTH SEtr B

47. This is considered as the first link in the chain of infection wherein the patient's symptoms are
infectious in origin
A. etiologic agent
B. reservoir
C. susceptible host
D. portal of entry
,

48. Hypertension and diseases of the heart are among the 10 leading causes of illnesses bach
year. These are collectively known as Lifestyle Related Non-Communicable Diseases (NCD)
because these diseases have common risk factor which are to large extent related to
unhealthy lifestyle. As a health promoter, what should you suggest to a patient on a low
cholesterol diet?
A. egg pie, boiled liver diet
B. fried chicken, potatoes diet
C. hamburger, french fries diet
D. grilted fish, eggplants diet
49. An infectious disease not considered communicable
A. scabies
B. tetanus
C. pneumonia
D. mumps
5O. The RHU physician documented a case of chickenpox in his center. What does the rash from
this viral exanthem look like?
A. maculo-papular rash
B. small irregular red spots with minute bluish-white centers
C. round or oval erythematous scaling patches
D. petechiae
51. A population pyramid with triangular shape and broad base indicates the following, except
A. high death rates C. young population
B. poverty D. more fernales l

52. A religious leader wants all sect members and their respective families to live with him jnd
his wife until the time of his death, is an example of what family structure? i

A. nuclear
B. extended
C. blehded
D. communal
53. The most effective way to assess the health care needs of older adults in a community is to
A. conduct personal interviews with local health care and social services professionals.
B. rely on intuitive impressions obtained from working with older persons.
C. review demographic data from the most recent census.
D. survey a random sample of adult children who do not reside in the community.
54. ln Smilkstein Family ToolAssessment, what is the APGAR component that measures family
capabilitytoutilizeandshareinherentresources
A. affection
B. resolve
C. adaptation
D. partnership
:

55. Uses weight for age as the index and includes the occurrence of edema to assist in
distinguishing between marasmus and kwashiorkor :

A. Wellcome classification

E, [*::*ra+;m
Continued on Page 7
PHYSICIAN Licensure Examination PaiTe 7
Sunday, September 17,2017 - O2:OO p.m. - O4:OO p.m. l

pREVENTtvE MEptCtNE ANp pUBLtC HEALTH


--T--
"#"
56. A ravaging typhoon strikes a depressed community in the coast town of Leyte. Approxim]ately
30 families are left without food and shelter. Which best reaction demonstrates the
community's preparedness i

A. community leaders activate the community-based disaster risk reduction and management
council
B. community leaders contact the national government for disaster relief
C. relief volunteers help in sorting and packing canned sardines, instant noodles and rice
D. typhoon victims are sent to public elementary schools

57. A type of disability benefits for work-connected injury or sickness


A. Temporary Total Disability CffD)
B. Permanent Partial Disability (PPD)
C. Permanent Total Disability (PTD)
D. None of the above
58. Also known as "brown lung disease" or Monday fever
A. Byssinosis
B. Coal workers' pneumoconiosis
C. Thresher's lungs
D. Bagassosis i

59. When promoting the health of the individual, family and community, the first step to take by
the RHU physician is
A. to establish a registry of families

C. to do a health assessment

60. The basic principle of pulmonary TB infection and sexually transmitted infection control
program rs
A. health education C. early case findings
B. drives for funds D. statistical predictions of epidemics
61. With a normal B-hour working day schedule, the most cornmon period for accidental injury is
the
A. 2nd hour C. 8th hour
B. 4th hour D. evenly distributed
62. Which scales refers to levels of magnitude or ranking?
A. interval C. ordinal
B. nominal D. ratio
63. lmproving of housing conditions is aimed at
A. planning the future development of co,mmunities
B. reduction of health and accident hazards
C. control of new construction and remodeling
D. all are correct
64. Water from a mountain stream is
A. generally safe to drink because of filtering action of the sand
B. generally has an excellent taste
C. hardness of water is primarily due to sulfur content
D. all are correct
65. Emerging and re-emerging infection that encode the antigenic shift variants that cause
epidemic to human threatening countries all-over the world
A. SARS
B. Retrovirus
C. Avian influenza
D. Ebola
Continued on Page 8
PHYSICIAN Licensure Examination Page 8
Sunday, September 17,2017 - O2:OO p.m. - O4:OO p.m.
PREVENTIVE MEDICINE AND PIJBLIC HEALTH SET B

66. Which of the following statements about retrospective and prospective studies is/are true?
A. Retrospective studies usually involve record review and are planned to obtain information
on events that will occur in the future ,

B. Prospective studies infrequently involve record reviews and are planned to obtain
information on past occurrences :

C. Both statements are true


i

D. Both statements are false


;

, i

67. ln any form of disaster, this is the least likely inadequacy to be reported in the managernent
of relief measures
A. identification of resources
B. transportation of resources
C. presence of volunteers
D. appraisal of damage
68. Which among the following has no field association with under 6 program of the Department
of Health?
A. tMCt C. UHC-H|-5
B. KOT D. RPIOD
69. The modified Third's solution for bedsores is composed of the following, except
A. Zonrox bleach C. Hydrochloric acid
B. Cane vinegar D. NSS
7O. To be effective in prevention and control of spread of communicable diseases, physicians
should strengthen skills and knowledge on
A. data analysis
B. community diagnosis
C. rehabilitation of patient
D. health education
71. A total of 97 neonatal cases were admittdO in St. Anthony Hospital from January to June of
2016. 71o/o were males and 29%o were females. There were 37 deaths. What was the case
fatality ratio of neonatal tetanus? :

A. 3OV. C. TOVI
B. 38% D. 72o/o

72. Children in difficult situations may resort to substance abuse. They include l
A. orphan"
B. sexually abused children ;

C. children of OFWs
D. all of the above l

73. A couple has 6 children,4 boys and2 girls. The most appropriate family planning method for
them is
A. permanent method l

C. method recommended by the doctor


D. method they decide and practice
74. Some 1 ,45O persons are in the population. 46 persons died. 640/o of those who died were age
55 years and above. What is the crude death rate?
A. 2.O3yo c. 3.790/o
B. 3.170h D. 4.4104
75. Hospital waste management program is er requirement before construction of a new facility
The hospital personnel are required to train in waste management to prevent which of the
following?
A. communicable disease C. cross-infection
B. nosocomial infection D. transmission of infection l

t^^^+ih'ia-l D-^A O
^^
PHYSICIAN Licensure Examination Page 9
Sunday, September 17,2017 - 02:00 p.m. - 04:00 p.m.
PREVENTIVE MEDICINE AND PUBLIC HEALTH seT B

76. Which of the following is the most effective measures to prevent or reduce high density flies
in the community?
A. prevent breeding places through sanitary refuse disposal
B. use insect sprays
C. screen houses
D. use fly.swatters/papers
77. Oral Rehydration Solution (ORS) should be discarded after
A. 72 hours C. 24 hours
B. 48 hours D. '12 hours
78. Newborn babies have limited ability to produce antibodies hence immunization should not be
given earlier than
A. 2 mos. C. 3 mos.
B. 1 mo. D. 6 weeks
79. Vaccination of the live attenuated form of ,this organism is contraindicated if the woman may
become pregnant within 3 months
A. Diphtheria C. Rubella I

B. Tetanus D. Hepatitis
80. The most important objective of an Acute Respiratory lnfection (ARl) case management is to
A. treat patient immediately thus preventing infection of the lower respiratory tra-t
B. prevent cross-infection with other members of the family
C.preventpatientfromtriggeringasthmaticattacks
D. recognize and treat pneumonia
I

81. Among men aged 21 to 35 in a community, weight is normally distributed with a mean of
59 Kg and a standard deviation of 8.5 Kg. What oZ of men will have a weight over 67.5 (g?
A.4% C.32% -: i-
B. 160/o D. no sufficient data
82. Hazardous exposure inherent to deteriorating American ancestral housing units
A. organophosphate toxicity
B. lead paint poisoning
C. asbestos inhalation
D. moldy molasses exposure
83. Technical term used for sickness which also measures prevalence and incidence
A. mortality C. life expectancy
B. morbidity D. disabitity
84. Specificity refers to
A. identification of those infected among those exposed
B. also known as the true positive rate
C. if high means can identify the presence of disease among those tested
D. ability of the test to identify those who do not have the disease after a negative result
85. Under the Paternity Leave Act of 1996 (R.A. 8187), married male employees regardless of
the nature of employment is entitled to -
A. 5 days of leave for the first 4 deliveries of his wife
B. 7 days of leave for the first 4 deliveries of his wife i

C. 10 days of leave for the first 4 deliveries of his wife


D.15daysofleaveforthefirst4deliveriesofhiswife
86. Substances may be mixed with food which may reduce its food value or render it injurious to
health
A. misbranded C. trademarked
B. adulterated D. blackmarketed

Continued on Page 10
PHYSICIAN Licensure Examination
Sunday, September 17,2017 - O2:OO p.m. -O4:00 p.m.
PREVENTIVE MEDICINE AND PUBLIC HEALTH SET B
87. A combination of liquid wastes or water-carried discharges conducted away from residehces
and other institution together with such ground surface or rain water as may be present is
known as
A. sewage
B. garbage
C. iefuse 1

D. rubbish
88. This type of well requires some type of pump either motor or hand-powered and makes use
of a high velocity stream of water to excavate the hole and to carry the excavated material out
of the hole
A. driven well
)

B. jetted well
C. drilled well
D. dug well
89. The sampling method used for a reliable communigr diagnosis is
A. simple random :

B. multi stage I

C. cluster
D. systematic random
:

90. ln developing capability of people to assume more responsibility for their health, which of the
following would you consider most important?
A. organizing community health
B. lead lag in the health activity
C. identifying health problems with the people
D. making medicine available
91. Vagabond's Disease is due to
A. mosquitos C. spiders
B. lice D. rodents
92. The author of a research study states tha! the lack of statistical significance in the result may
be related to the sample size and suggests that the study be replicated with a larger sample.
Which statement is accurate?
A. An exact replication may produce the same results.
B. Replication of a well-designed study is appropriate.
C. Sample size is not important if the appropriate statistical techniques are applied.
D. Sample size is not related to statistical significance, so the author's suggestion is illogical.
93, According to the Sanitation Code of the Philippines, a septic tank should be
A. At least 15 meters from any source of water supply
B. At least 25 meters from any source of water supply
c.Atleast50metersfromanySourceofwatersupply
D. At least 100 meters from any source,of water supply
94. Major techniques to be used in the prevention of dental caries, except
A. The observation of good mouth hygiene ,

B. The restriction of fermentable-carbohydrates in the diet


C. Use of fluoride solutions
D. Chlorination of drinking water
95. Chemical asphyxiants interfere with the transportation or absorption of oxygen in the body.
One of the best example is
A. methane
B. hydrogen sulfide
C. nitrogen
D. carbon dioxide

Continued on Paoe 11
PHYSICIAN Licensure Examination Page 1 1

Sunday, September 17,2017 - O2:OO p.m. - 04:00 p.m.


PREVENTIVE MEDICINE AND PUBLIC HEALTH SET B

96. As a resource person, you discussed about the contraceptive methods. Which of these is
least effective?
A. oral e,ontraceptives C. condom
B. tubal ligation D. IUD
97. Home accidents causes the largest number of injuries because
A. people are more careless at home
B. people spend more time at home tha;t elsewhere
C. homes are basically unsafe
D. intentional accidents are common :

98. A type of survey or study that determines the norms and limits of the distribution of variables,
including the characteristic of the events and population under study, is referred to as ,

A. descriptive
B. explanatory
i

C. analytical
D- cross-sectional i

99.Methodswidelyusedtodetectthepresenceofmicrobesinpasteurizedmilk,except
A. methylene blue reduction test i

C. hot filter test '

D. coliform bacteria count


1OO. The operating unit of the Department of l{ealth at the local level is
A. Barangay Health Station
B. Puericulture Center
C. Rural Health Unit
D. Regional Hospital
*** END ***

WARNING: Failure to submit your Test Questions (Gomplete) set will cause the cancellation
of your Test-Result for this subject.
PREVENTIVE MEDICINE & PUBLIC HEALTH 2017
ANSWER KEY

1. B 26. D 51. D 76. A


2.A 27. D 52. D 77. C
3.A 28. A 53. A 78. D
4.C 29. C 54. C 79. C
5.D 30. c 55. A 80. D
6.C 31. A 56. A 81. B
7, C 32. C 57. B 82. B
8.8 33. A 58. A 83. B
9.8 34. C 59. C 84. D
10. D 35. D 60. c 85. B
11. D 36. D 61. A 86. B
12. B 37. C 62. C 87. A
13. A 38. C 63. D 88. B
14. D 39. A 64. B 89. D
5.() 40. B 65. C 90. c
16. A 41. C 66. D 91. B
17. A 42. C 67. D 92. B
18. B 43. B 68. D 93. B
19. C 44. B 69. C 94. D
20. c 45. A 70. D 95. B
21. C 46. D 71. B 96. C
22. C 47. A 72. D 97. B
23. D 48. D 73. C 98. A
24. C 49. B 74. B 99. C
25. B 50. A 75. D 100. c

You might also like